Site Loader

Содержание

Электромагнитная индукция. — Закон электромагнитной индукции

Электромагнитная индукция — явление возникновения электрического тока в замкнутом контуре при изменении магнитного потока, проходящего через него.

Электромагнитная индукция была открыта Майклом Фарадеем 29 августа 1831 года. Он обнаружил, что электродвижущая сила, возникающая в замкнутом проводящем контуре, пропорциональна скорости изменения магнитного поток через поверхность, ограниченную этим контуром. Величина электродвижущей силы (ЭДС) не зависит от того, что является причиной изменения потока — изменение самого магнитного поля или движение контура (или его части) в магнитном поле. Электрический ток, вызванный этой ЭДС, называется индукционным током.

Зако́н электромагни́тной инду́кции Фараде́я является основным законом электродинамики, касающимся принципов работы трансформаторов,дросселей, многих видов электродвигателей и генераторов. Закон гласит:

  • Для любого замкнутого контура индуцированная электродвижущая сила (ЭДС) равна скорости изменения магнитного потока, проходящего через этот контур.

или другими словами:

  • Генерируемая ЭДС пропорциональна скорости изменения магнитного потока.

Согласно закону электромагнитной индукции Фарадея: 

где

 — электродвижущая сила, действующая вдоль произвольно выбранного контура,
  — магнитный поток через поверхность, натянутую на этот контур.

Знак «минус» в формуле отражает правило Ленца, названное так по имени русского физика Э. Х. Ленца:

Индукционный ток, возникающий в замкнутом проводящем контуре, имеет такое направление, что создаваемое им магнитное поле противодействует тому изменению магнитного потока, которым был вызван данный ток.

Для катушки, находящейся в переменном магнитном поле, закон Фарадея можно записать следующим образом:

где

 — электродвижущая сила,
 — число витков,
 — магнитный поток через один виток,
 — потокосцепление катушки.

Информация взята с http://ru. wikipedia.org/wiki/Закон_электромагнитной_индукции_

Закон электромагнитной индукции Фарадея, колебательный контур

§ 3. Электродинамика

3.1. Основные понятия и законы электростатики Закон Кулона:
сила взаимодействия двух точечных неподвижных зарядов в вакууме прямо пропорциональна произведению модулей зарядов и обратно пропорциональна квадрату расстояния между ними:

Коэффициент пропорциональности в этом законе

В СИ коэффициент k записывается в виде

где ε0 = 8, 85 · 10−12 Ф/м (электрическая постоянная).

Точечными зарядами называют такие заряды, расстояния между которыми гораздо больше их размеров.
 Электрические заряды взаимодействуют между собой с помощью электрического поля. Для качественного описания электрического поля используется силовая характеристика, которая называется «напряжённостью электрического поля» (E). Напряжённость электрического поля

равна отношению силы, действующей на пробный заряд, помещённый в некоторую точку поля, к величине этого заряда:

 Направление вектора напряжённости совпадает с направлением силы, действующей на положительный пробный заряд. [E]=B/м. Из закона Кулона и определения напряжённости поля следует, что напряжённость поля точечного заряда

где q — заряд, создающий поле; r — расстояние от точки, где находится заряд, до точки, где создаётся поле.
 Если электрическое поле создаётся не одним, а несколькими зарядами, то для нахождения напряжённости результирующего поля используется принцип суперпозиции электрических полей: напряжённость результирующего поля равна векторной сумме напряжённостей полей, созданных каждым из зарядов — источников в отдельности:

Работа электрического поля при перемещении заряда: найдём работу перемещения положительного заряда силами Кулона в однородном электрическом поле. Пусть поле перемещает заряд q из точки 1 в точку 2:


 В электрическом поле работа не зависит от формы траектории, по которой перемещается заряд. Из механики известно, что если работа не зависит от формы траектории, то она равна изменению потенциальной энергии с противоположным знаком:

Отсюда следует, что

Потенциалом электрического поля называют отношение потенциальной энергии заряда в поле к этому заряду:

 Запишем работу поля в виде

Здесь U = ϕ1 − ϕ2разность потенциалов в начальной и конечной точках траектории. Разность потенциалов называют также напряжением

 Часто наряду с понятием «разность потенциалов» вводят понятие «потенциал некоторой точки поля». Под потенциалом точки подразумевают разность потенциалов между данной точкой и некоторой заранее выбранной точкой поля. Эту точку можно выбирать в бесконечности, тогда говорят о потенциале относительной бесконечности.

Потенциал поля точечного заряда подсчитывается по формуле

 Проекция напряжённости электрического поля на какую-нибудь ось и потенциал связаны соотношением

3.2. Электроёмкость. Конденсаторы. Энергия электрического поляЭлектроёмкостью тела называют величину отношения

 Формула для подсчёта ёмкости плоского конденсатора имеет вид:

где S — площадь обкладок, d — расстояние между ними.
 Конденсаторы можно соединять в батареи. При параллельном соединении ёмкость батареи C равна сумме ёмкостей конденсаторов:

Разности потенциалов между обкладками одинаковы, а заряды прямо пропорциональны ёмкостям.

 При последовательном соединении величина, обратная ёмкости батареи, равна сумме обратных ёмкостей, входящих в батарею:

 Заряды на конденсаторах одинаковы, а разности потенциалов обратно пропорциональны ёмкостям.
 Заряженный конденсатор обладает энергией. Энергию заряженного конденсатора можно подсчитать по любой из следующих формул:

3.3. Основные понятия и законы постоянного токаЭлектрический ток — направленное движение электрических зарядов. В разных веществах носителями заряда выступают элементарные частицы разного знака. За положительное направление тока принято направление движения положительных зарядов. Количественно электрический ток характеризуют его силой. Это заряд, прошедший за единицу времени через поперечное сечение проводника:

Закон Ома для участка цепи имеет вид:

Коэффициент пропорциональности R, называемый электрическим сопротивлением, является характеристикой проводника [R]=Ом. Сопротивление проводника зависит от его геометрии и свойств материала:

где l — длина проводника, ρ — удельное сопротивление, S — площадь поперечного сечения. ρ является характеристикой материала и его состояния. [ρ] = Ом·м.
 Проводники можно соединять последовательно. Сопротивление такого соединения находится как сумма сопротивлений:

 При параллельном соединении величина, обратная сопротивлению, равна сумме обратных сопротивлений:

 Для того чтобы в цепи длительное время протекал электрический ток, в составе цепи должны содержаться источники тока. Количественно источники тока характеризуют их электродвижущей силой (ЭДС). Это отношение работы, которую совершают сторонние силы при переносе электрических зарядов по замкнутой цепи, к величине перенесённого заряда:

 Если к зажимам источника тока подключить нагрузочное сопротивление R, то в получившейся замкнутой цепи потечёт ток, силу которого можно подсчитать по формуле


Это соотношение называют законом Ома для полной цепи.

 Электрический ток, пробегая по проводникам, нагревает их, совершая при этом работу

где t — время, I — сила тока, U — разность потенциалов, q — прошедший заряд.

Закон Джоуля-Ленца:

3.4. Основные понятия и законы магнитостатики  Характеристикой магнитного поля является магнитная индукция ➛B. Поскольку это вектор, то следует определить и направление этого вектора, и его модуль. Направление вектора магнитной индукции связано с ориентирующим действием магнитного поля на магнитную стрелку. За направление вектора магнитной индукции принимается направление от южного полюса S к северному N магнитной стрелки, свободно устанавливающейся в магнитном поле.
 Направление вектора магнитной индукции прямолинейного проводника с токам можно определить с помощью правила буравчика:
если направление поступательного движения буравчика совпадает с направлением тока в проводнике, то направление вращения рукоятки буравчика совпадает с направлением вектора магнитной индукции.

 Модулем вектора магнитной индукции назовём отношение максимальной силы, действующей со стороны магнитного поля на участок проводника с током , к произведению силы тока на длину этого участка:

Единица магнитной индукции называется тесла (1 Тл)

Магнитным потоком Φ через поверхность контура площадью S называют величину, равную произведению модуля вектора магнитной индукции на площадь этой поверхности и на косинус угла между вектором магнитной индукции ➛B и нормалью к поверхности ➛n:


Единицей магнитного потока является вебер (1 Вб).
 На проводник с током, помещённый в магнитное поле, действует сила Ампера

Закон Ампера:
на отрезок проводника с током силой I и длиной l, помещённый в однородное магнитное поле с индукцией ➛B , действует сила, модуль которой равен произведению модуля вектора магнитной индукции на силу тока, на длину участка проводника, находящегося в магнитном поле, и на синус угла между направлением вектора ➛B и проводником с током:

 Направление силы Ампера определяется с помощью правила левой руки:
если левую руку расположить так, чтобы перпендикулярная проводнику составляющая вектора магнитной индукции входила в ладонь, а четыре вытянутых пальца указывали бы направление тока, то отогнутый на 90° большой палец укажет направление силы Ампера.
 На электрический заряд, движущийся в магнитном поле, действует сила Лоренца. Модуль силы Лоренца, действующей на положительный заряд, равен произведению модуля заряда на модуль вектора магнитной индукции и на синус угла между вектором магнитной индукции и вектором скорости движущегося заряда:

 Направление силы Лоренца определяется с помощью правила левой руки: если левую руку расположить так, чтобы составляющая магнитной индукции, перпендикулярная скорости заряда, входила в ладонь, а четыре пальца были направлены по движению положительного заряда, то отогнутый на 90° большой палец покажет направление силы Лоренца, действующей на заряд. Для отрицательно заряженной частицы сила Лоренца направлена против направления большого пальца.

3.5. Основные понятия и законы электромагнитной индукции  Если замкнутый проводящий контур пронизывается меняющимся магнитным потоком, то в этом контуре возникает ЭДС и электрический ток. Эту ЭДС называют ЭДС электромагнитной индукции, а ток — индукционным. Явление их возникновения называют электромагнитной индукцией. ЭДС индукции можно подсчитать по основному закону электромагнитной индукции или по закону Фарадея:

Знак «−» связан с направлением индукционного тока. Оно определяется по правилу Ленца:
индукционный ток имеет такое направление, что его действие противодействует причине, вызвавшей появление этого тока.
 Магнитный поток, пронизывающий контур, прямо пропорционален току, протекающему в этом контуре:

Коэффициент пропорциональности L зависит от геометрии контура и называется индуктивностью, или коэффициентом самоиндукции этого контура. [L] = 1 Гн

Энергию магнитного поля тока можно подсчитать по формуле

где L — индуктивность проводника, создающего поле; I — ток, текущий по этому проводнику

3.6. Электромагнитные колебания и волныКолебательным контуром называется электрическая цепь, состоящая из последовательно соединённых конденсатора с ёмкостью C и катушки с индуктивностью L (см. рис. 7).

 Для свободных незатухающих колебаний в контуре циклическая частота определяется формулой

 Период свободных колебаний в контуре определяется формулой Томсона:

 Если в LC-контур последовательно с L, C и R включить источник переменного напряжения, то в цепи возникнут вынужденные электрические колебания. Такие колебания принято называть переменным электрическим током
 В цепь переменного тока можно включать три вида нагрузки — конденсатор, резистор и катушку индуктивности.

 Конденсатор оказывает переменному току сопротивление, которое можно посчитать по формуле

 Ток, текущий через конденсатор, по фазе опережает напряжение на π/2 или на четверть периода, а напряжение отстаёт от тока на такой же фазовый угол.

 Катушка индуктивности оказывает переменному току сопротивление, которое можно посчитать по формуле

 Ток, текущий через катушку индуктивности, по фазе отстаёт от напряжения на π/2 или на четверть периода. Напряжение опережает ток на такой же фазовый угол.

Трансформатором называется устройство, предназначенное для преобразования переменных токов. Трансформатор состоит из замкнутого стального сердечника, на который надеты две катушки. Катушка, которая подключается к источнику переменного напряжения, называется первичной обмоткой, а катушка, которая подключается к потребителю, называется вторичной обмоткой. Отношение напряжения на первичной обмотке и вторичной обмотке трансформатора равно отношению числа витков в этих обмотках:

Если K > 1, трансформатор понижающий, если K

Закон электромагнитной индукции (закон Фарадея — Максвелла — Ленца) — З — Русский алфавит — Словарь-справочник электрика

Закон электромагнитной индукциизакон Фарадея — Максвелла — Ленца — закон, устанавливающий взаимосвязь между магнитными и электрическими явлениями. Эдс электромагнитной индукции, в контуре численно равна и противоположна по знаку скорости изменения магнитного потока сквозь поверхность, ограниченную этим контуром. Пусть в однородном магнитном поле B помещен прямолинейный отрезок проводника. При передвижении его перпендикулярно к силовым линиям заряды под действием магнитной силы начнут передвигаться вдоль проводника перпендикулярно к направлению движения и вектору поля B. Направление тока легко установить по правилу правой руки. Ток будет течь до тех пор, пока на концах проводника не образуются заряды противоположного знака, которые создадут электрическое поле E, направленное против силы, действующей на заряд. Электрические и магнитные силы уравновесят друг друга, и движение зарядов прекратится. В этот момент E=-q[VB]/q=-[VB]. В рассматриваемом случае заряды накапливаются на концах проводника. Если поместить в магнитное поле замкнутый контур и двигать его так, чтобы плоскость рамки была перпендикулярна к линиям однородного магнитного поля B, то ток пойдет по стороне ab (от a к b) и по dc (от d к c). Две другие стороны можно не рассматривать, так как сила, действующая на заряды, расположенные в проводнике, перпендикулярна к проводу и движение зарядов прекращается. Если токи в сторонах ab и dc текут навстречу друг другу, что приводит к накоплению заряда, замкнутый ток по рамке не пойдет. Величина эдс вдоль рамки равна нулю ΣElΔl=Eablab+Ebclbc+Ecdlcd+Edalda=(E+0-E+0)Δl=0. При движении рамки в том же направлении сторона dc выйдет из области, занимаемой магнитным полем, и поток последнего через плоскость рамки начнет уменьшаться. В этом случае сила, действовавшая на заряды, принадлежащие стороне dc, исчезнет и заряды, движущиеся от a к b, ничто уравновешивать не будет. По рамке пойдет замкнутый ток. То же самое будет наблюдаться при движении, например, соленоида относительно неподвижно лежащей рамки. Величина эдс в этом случае равна ΣEΔl=Eablab+0+0+0=Eablab=VBΔl. Таким образом, наведенная в проводе эдс электромагнитной индукции пропорциональна величине магнитной индукции поля, в котором движется проводник, длине провода и скорости его движения в направлении, перпендикулярном к магнитным силовым линиям.

Если обозначить скорость V=Δx/Δt где Δx — смещение рамки за время Δt, то ΣEΔl=-BΔxΔl/Δt. Знак минус в правой части равенства объясняется законом Ленца. Так как произведение ΔxΔl равно изменению площади ΔS, пронизываемой магнитным потоком, а ΔSB=ΔΦ — изменение самого магнитного потока за время t, то получим ΣEΔl=-ΔΦ/Δt. Таким образом, при изменении магнитного потока через контур начинает идти ток (индукционный или наведенный), который обязан своим возникновением электрическому полю. Величина эдс поля зависит от скорости изменения магнитного потока.

Закон индукции Фарадея и закон Ленца

Физика > Закон индукции Фарадея и закон Ленца

 

Изучите действие закона электромагнитной индукции Фарадея и закона Ленца – явление, формулировка и формулы: создание электродвижущей силы в магнитном поле.

Созданная переменой магнитного потока 

Задача обучения

  • Перевести закон индукции Фарадея в формулу.

Основные пункты

  • Минус говорит о том, что созданные ток и магнитное поле вступают в противостояние изменению потока – закон Ленца.
  • Закон индукции выступает вторым главным принципом функционирования множества электрических приборов.
  • Согласно закону Фарадея, созданная переменой магнитного поля ЭДС зависит от изменения потока, времени и количества витков на катушке.

Термины

  • Поток – скорость передачи энергии.
  • Соленоид – катушка с намотанным проводом, функционирующая по принципу магнита, когда сквозь нее проходит электрический ток.
  • Электродвижущая сила (ЭДС) – созданное батареей или магнитной силой напряжение.

Закон индукции Фарадея

Это главный закон в электромагнетизме, отображающий принципы контакта магнитного поля и электрической цепи, что приводит к формированию электродвижущей силы (ЭДС). На этом принципе работает много различных механизмов, вроде генераторов, соленоида, индукторов, трансформаторов и т.д.

Фарадей в своих экспериментах смог показать, что созданная при перемене магнитного потока ЭДС зависит от нескольких факторов. Начнем с того, что она выступает в прямой пропорциональности перемене потока, а своего максимума достигает, если изменение времени было незначительным. Если у катушки есть N оборотов, то ЭДС в N раз больше, чем у одиночной катушки. Высчитывается по формуле:

Это выражение закона индукции Фарадея, где единицей служит вольт.

Закон Ленца

Отметьте в формуле знак минус, потому что он играет важную роль. С его помощью мы понимаем, что созданные ЭДС ток и магнитное поле вступают в противостояние с переменой потока – закон Ленца. Заданное минусом направление было именовано в честь Генриха Ленца, который в одиночку занимался исследованием аспектов индукции. Конечно, Фарадей то же знал о направлении, но Ленц заявил об этом первым.

(а) Когда магнитный стержень входит в катушку, сила магнитного поля возрастает. Ток создает еще одно поле, но в противоположном направлении к магниту, чтобы вступить в противостояние увеличению. Это один из аспектов закона Ленца. (b) и (c) – Другие примеры ситуаций. Главное проверить, чтобы направление показывало противостояние перемене магнитного потока и отвечало правилу правой руки

Энергосбережение

Закон Ленца отображает проявление энергетического сбережения. ЭДС создает ток, противостоящий перемене потока. Энергия способна войти или выйти, но это не происходит мгновенно. Закон Ленца выступает следствием. Как только начинается изменение, индукция выступает против. Если бы была положительная обратная связь, то индуцированная ЭДС оказывалась в том же направлении, что и измененный поток.


Урок 5. электромагнитная индукция — Физика — 11 класс

Физика, 11 кл

Урок 5. Электромагнитная индукция

Перечень вопросов, рассматриваемых на этом уроке

  1. Знакомство с явлением электромагнитной индукции.
  2. Изучение законов, описывающих явление электромагнитной индукции.
  3. Решение задач, практическое использование электромагнитной индукции.

Глоссарий по теме

Явление электромагнитной индукции заключается в возникновении электрического тока в проводящем контуре, который либо покоится в переменном во времени магнитном поле, либо движется в постоянном магнитном поле таким образом, что число линий магнитной индукции, пронизывающих поверхность, ограниченную этим контуром, меняется со временем. Магнитный поток Ф – графически величина пропорциональная числу линий магнитной индукции, пронизывающих поверхность площадью S.

Единица измерения магнитного потока: магнитный поток в один вебер создаётся однородным магнитным полем с индукцией 1 Тл через поверхность площадью 1 м2, расположенную перпендикулярно вектору магнитной индукции.

Правило Ленца: возникающий в замкнутом контуре индукционный ток своим магнитным полем противодействует тому изменению магнитного потока, которым он вызван.

Сила индукционного тока пропорциональна скорости изменения магнитного потока через поверхность, ограниченную контуром.

ЭДС индукции в замкнутом контуре равна по модулю скорости изменения магнитного потока через поверхность, ограниченную контуром:

Основная и дополнительная литература по теме:

Мякишев Г.Я., Буховцев Б.Б., Чаругин В.М. Физика.10 класс. Учебник для общеобразовательных организаций М.: Просвещение, 2017стр. 107-112

Рымкевич А.П. Сборник задач по физике. 10-11класс. — М.: Дрофа,2009. Стр. 28-29

ЕГЭ 2017. Физика. 1000 задач с ответами и решениями. Демидова М.Ю., Грибов В.А., Гиголо А.И. М.: Экзамен, 2017.

Теоретический материал для самостоятельного изучения

Электрические и магнитные поля создаются одними и теми же источниками – электрическими зарядами. Отсюда естественнее было предположить, что между этими полями имеется связь. Экспериментально это предположение было доказано в 1831 г. английским учёным М. Фарадеем, открывшим явление электромагнитной индукции. Все опыты Фарадея по изучению явления электромагнитной индукции объединял один признак – магнитный поток пронизывающий замкнутый контур проводника менялся. При всяком изменении магнитного потока через замкнутый контур, в нем возникал индукционный ток.

Сила индукционного тока пропорциональна ЭДС индукции.

Направление индукционного тока менялось в зависимости от направления движения магнита относительно катушки. Это направление тока, можно найти используя правило Ленца.

М. Фарадеем экспериментально было установлено, что при изменении магнитного потока, в проводящем контуре возникает электродвижущая сила индукции, которая равна скорости изменения магнитного потока через поверхность, ограниченную контуром, взятой со знаком минус:

Знак минус в этой формуле отражает правило Ленца.

Закон электромагнитной индукции формулируется для ЭДС индукции.

ЭДС индукции в замкнутом контуре равна по модулю скорости изменения магнитного потока через поверхность, ограниченную контуром:

ЭДС индукции в движущихся проводниках:

Ɛ_i = Вlvsinα.

Джеймс Максвелл в 1860 году сделал вывод что переменное со временем магнитное поле всегда порождает вихревое электрическое поле, а переменное во времени электрическое поле в свою очередь порождает магнитное поле. Следовательно, существует единая теория электромагнитного поля.

Разбор типового контрольного задания

1.

На рисунке изображен момент демонстрационного эксперимента по проверке правила Ленца, когда все предметы неподвижны. Южный полюс магнита находится внутри сплошного металлического кольца, но не касается его. Коромысло с металлическими кольцами может свободно вращаться вокруг вертикальной опоры. При выдвижении магнита из кольца влево кольцо будет

1) оставаться неподвижным

2) перемещаться вправо

3) совершать колебания

4) перемещаться вслед за магнитом

При выдвижении магнита из кольца влево магнитный поток от магнита через кольцо будет уменьшаться. В замкнутом кольце возникает индукционный ток. Направление этого тока по правилу Ленца такое, что создаваемое им магнитное поле препятствует изменению магнитного потока. Так как коромысло вокруг вертикальной оси может свободно вращаться, и магнитное поле магнита неоднородно, коромысло под действием сил Ампера начнёт двигаться так, чтобы препятствовать изменению магнитного потока. Следовательно, коромысло начнёт перемещаться вслед за магнитом.

Ответ:4) перемещаться вслед за магнитом.

2.

Проводник МN с длиной активной части 1м и сопротивлением 2 Ом находится в однородном магнитном поле индукцией 0,2 Тл. Проводник подключён к источнику тока с ЭДС 4 В (внутренним сопротивлением источника и сопротивлением подводящих проводников пренебречь). Какова сила тока в проводнике, если:

№1 проводник покоится;

№2 проводник движется в право со скоростью 6 м/с.

Дано:

ℓ= 1м

R = 2 Ом

В = 0,2 Тл

Ɛ = 4 В

I =?

Решение:

№1: Ток в неподвижном проводнике течёт от N к М

v = 0; Закон Ома для полной цепи I = Ɛ/R = 4В/2Ом = 2А

№2: Если проводник движется в право со скоростью 6 м/с, то по правилу правой руки индукционный ток потечёт от точки N к точке М:

Ответ: №1 2А

№2 2,6А

Парадокс Фарадея — Faraday paradox

Парадокс Фарадея или парадокс Фарадея является любым экспериментом , в котором Майкл Фарадей закон «s из электромагнитной индукции появляется предсказать неправильный результат. Парадоксы делятся на два класса:

  • Похоже, что закон Фарадея предсказывает, что ЭДС будет нулевая, но есть ненулевая ЭДС.
  • Похоже, что закон Фарадея предсказывает, что ЭДС будет ненулевой, но нулевой ЭДС.

Фарадей вывел свой закон индукции в 1831 году, после изобретения первого электромагнитного генератора или динамо-машины , но так и не удовлетворился своим собственным объяснением парадокса.

Закон Фарадея в сравнении с уравнением Максвелла – Фарадея

Закон Фарадея (также известный как закон Фарадея-Ленца ) гласит, что электродвижущая сила (ЭДС) определяется полной производной магнитного потока по времени t :

E знак равно — d Φ B d т , {\ displaystyle {\ mathcal {E}} = — {\ frac {d \ Phi _ {B}} {dt}},}

где — ЭДС, а Φ B — магнитный поток . Направление электродвижущей силы задается законом Ленца . Часто забывают факт, что закон Фарадея основан на полной производной, а не на частной производной магнитного потока. Это означает, что ЭДС может возникать, даже если общий поток через поверхность постоянен. Чтобы решить эту проблему, можно использовать специальные методы. См. Ниже раздел об использовании специальных техник с законом Фарадея . Однако наиболее распространенная интерпретация закона Фарадея такова: E {\ displaystyle {\ mathcal {E}}}

Индуцированная электродвижущая сила в любой замкнутой цепи равна отрицательной скорости изменения магнитного потока, заключенного в цепи.

Эта версия закона Фарадея строго соблюдается только тогда, когда замкнутая цепь представляет собой петлю из бесконечно тонкой проволоки, и недействительна в других обстоятельствах. Он игнорирует тот факт, что закон Фарадея определяется полной, а не частной производной магнитного потока, а также тот факт, что ЭДС не обязательно ограничивается замкнутым контуром, но также может иметь радиальные компоненты, как обсуждается ниже. Другая версия, уравнение Максвелла – Фарадея (обсуждается ниже), действительна во всех обстоятельствах, и при использовании в сочетании с законом силы Лоренца это согласуется с правильным применением закона Фарадея.

Схема доказательства закона Фарадея из уравнений Максвелла и закона силы Лоренца.
Рассмотрим производную по времени потока через возможно движущийся контур с площадью : Σ ( т ) {\ Displaystyle \ Sigma (т)}
d Φ B d т знак равно d d т ∫ Σ ( т ) B ( т ) ⋅ d А {\ displaystyle {\ frac {d \ Phi _ {B}} {dt}} = {\ frac {d} {dt}} \ int _ {\ Sigma (t)} \ mathbf {B} (t) \ cdot г \ mathbf {A}}

Интеграл может изменяться со временем по двум причинам: подынтегральное выражение может измениться или область интегрирования может измениться. Таким образом, они складываются линейно:

d Φ B d т | т знак равно т 0 знак равно ( ∫ Σ ( т 0 ) ∂ B ∂ т | т знак равно т 0 ⋅ d А ) + ( d d т ∫ Σ ( т ) B ( т 0 ) ⋅ d А ) {\ displaystyle \ left. {\ frac {d \ Phi _ {B}} {dt}} \ right | _ {t = t_ {0}} = \ left (\ int _ {\ Sigma (t_ {0}) } \ left. {\ frac {\ partial \ mathbf {B}} {\ partial t}} \ right | _ {t = t_ {0}} \ cdot d \ mathbf {A} \ right) + \ left ({ \ frac {d} {dt}} \ int _ {\ Sigma (t)} \ mathbf {B} (t_ {0}) \ cdot d \ mathbf {A} \ right)}

где t 0 — любое заданное фиксированное время. Покажем, что первое слагаемое в правой части соответствует ЭДС трансформатора, второе — ЭДС движения. Первый член в правой части можно переписать, используя интегральную форму уравнения Максвелла – Фарадея:

∫ Σ ( т 0 ) ∂ B ∂ т | т знак равно т 0 ⋅ d А знак равно — ∮ ∂ Σ ( т 0 ) E ( т 0 ) ⋅ d ℓ {\ displaystyle \ int _ {\ Sigma (t_ {0})} \ left. {\ frac {\ partial \ mathbf {B}} {\ partial t}} \ right | _ {t = t_ {0}} \ cdot d \ mathbf {A} = — \ oint _ {\ partial \ Sigma (t_ {0})} \ mathbf {E} (t_ {0}) \ cdot d {\ boldsymbol {\ ell}}}
Площадь, заметаемая векторным элементом d кривой ∂Σ за время dt при движении со скоростью v .

Далее мы анализируем второй член в правой части:

d d т ∫ Σ ( т ) B ( т 0 ) ⋅ d А {\ displaystyle {\ frac {d} {dt}} \ int _ {\ Sigma (t)} \ mathbf {B} (t_ {0}) \ cdot d \ mathbf {A}}

Это самая трудная часть доказательства; более подробную информацию и альтернативные подходы можно найти в справочных материалах. По мере того как петля перемещается и / или деформируется, она выметает поверхность (см. Рисунок справа). Магнитный поток через эту вытянутую поверхность соответствует магнитному потоку, который либо входит, либо выходит из контура, и, следовательно, это магнитный поток, который вносит вклад в производную по времени. (На этом шаге неявно используется закон Гаусса для магнетизма : поскольку силовые линии не имеют начала и конца, они могут попасть в петлю, только будучи разрезанной проволокой.) Поскольку небольшая часть петли движется со скоростью v на короткое время. время , он выметает вектор площади . Следовательно, изменение магнитного потока через петлю здесь равно d ℓ {\ displaystyle d {\ boldsymbol {\ ell}}} d т {\ displaystyle dt} d А знак равно v d т × d ℓ {\ displaystyle d \ mathbf {A} = \ mathbf {v} \, dt \ times d {\ boldsymbol {\ ell}}}

B ⋅ ( v d т × d ℓ ) знак равно — d т d ℓ ⋅ ( v × B ) {\ displaystyle \ mathbf {B} \ cdot (\ mathbf {v} \, dt \ times d {\ boldsymbol {\ ell}}) = — dt \, d {\ boldsymbol {\ ell}} \ cdot (\ mathbf {v} \ times \ mathbf {B})}

Следовательно:

d d т ∫ Σ ( т ) B ( т 0 ) ⋅ d А знак равно — ∮ ∂ Σ ( т 0 ) ( v ( т 0 ) × B ( т 0 ) ) ⋅ d ℓ {\ displaystyle {\ frac {d} {dt}} \ int _ {\ Sigma (t)} \ mathbf {B} (t_ {0}) \ cdot d \ mathbf {A} = — \ oint _ {\ partial \ Sigma (t_ {0})} (\ mathbf {v} (t_ {0}) \ times \ mathbf {B} (t_ {0})) \ cdot d {\ boldsymbol {\ ell}}}

где v — скорость точки на петле . ∂ Σ {\ Displaystyle \ partial \ Sigma}

Собирая их вместе,

d Φ B d т | т знак равно т 0 знак равно ( — ∮ ∂ Σ ( т 0 ) E ( т 0 ) ⋅ d ℓ ) + ( — ∮ ∂ Σ ( т 0 ) ( v ( т 0 ) × B ( т 0 ) ) ⋅ d ℓ ) {\ displaystyle \ left. {\ frac {d \ Phi _ {B}} {dt}} \ right | _ {t = t_ {0}} = \ left (- \ oint _ {\ partial \ Sigma (t_ { 0})} \ mathbf {E} (t_ {0}) \ cdot d {\ boldsymbol {\ ell}} \ right) + \ left (- \ oint _ {\ partial \ Sigma (t_ {0})} ( \ mathbf {v} (t_ {0}) \ times \ mathbf {B} (t_ {0})) \ cdot d {\ boldsymbol {\ ell}} \ right)}

Между тем, ЭДС определяется как энергия, доступная на единицу заряда, которая проходит один раз по проволочной петле. Следовательно, по закону силы Лоренца ,

E M F знак равно ∮ ( E + v × B ) ⋅ d ℓ {\ displaystyle \ mathrm {EMF} = \ oint \ left (\ mathbf {E} + \ mathbf {v} \ times \ mathbf {B} \ right) \ cdot {\ text {d}} {\ boldsymbol {\ ell }}}

Комбинируя их, d Φ B d т знак равно — E M F {\ displaystyle {\ frac {d \ Phi _ {B}} {dt}} = — \ mathrm {EMF}}

Уравнение Максвелла – Фарадея является обобщением закона Фарадея, согласно которому изменяющееся во времени магнитное поле всегда сопровождается изменяющимся в пространстве неконсервативным электрическим полем, и наоборот. Уравнение Максвелла – Фарадея:

∇ × E знак равно — ∂ B ∂ т {\ displaystyle \ nabla \ times \ mathbf {E} = — {\ frac {\ partial \ mathbf {B}} {\ partial t}}}

(в единицах СИ ), где — оператор частной производной , — оператор ротора, и снова E ( r , t ) — электрическое поле, а B ( r , t ) — магнитное поле . Эти поля обычно могут быть функциями положения r и времени t . ∂ {\ displaystyle \ partial} ∇ × {\ Displaystyle \ набла \ раз}

Уравнение Максвелла – Фарадея является одним из четырех уравнений Максвелла и поэтому играет фундаментальную роль в теории классического электромагнетизма . Его также можно записать в интегральной форме по теореме Кельвина – Стокса .

Парадоксы, в которых закон индукции Фарадея, кажется, предсказывает нулевую ЭДС, но на самом деле предсказывает ненулевую ЭДС

Эти парадоксы обычно разрешаются тем фактом, что ЭДС может создаваться изменением потока в цепи, как объяснено в законе Фарадея, или движением проводника в магнитном поле. Это объясняется Фейнманом, как указано ниже. См. Также A. Sommerfeld, Vol III Electrodynamics Academic Press, стр. 362.

Оборудование

Рисунок 1: Дисковый электрогенератор Фарадея. Диск вращается с угловой скоростью ω, перемещая проводящий диск по кругу в статическом магнитном поле B, создаваемом постоянным магнитом. Магнитная сила Лоренца v × B перемещает ток в радиальном направлении через проводящий диск к проводящему ободу, и оттуда контур цепи завершается через нижнюю щетку и ось, поддерживающую диск. Таким образом, ток возникает в результате механического движения.

Для эксперимента требуется несколько простых компонентов (см. Рис. 1): цилиндрический магнит , проводящий диск с проводящим ободом, проводящая ось, немного проводов и гальванометр . Диск и магнит установлены на небольшом расстоянии друг от друга на оси, на которой они могут свободно вращаться вокруг своих осей симметрии. Электрическая цепь образуется подключением скользящих контактов: один к оси диска, другой к его ободу. В схему можно вставить гальванометр для измерения силы тока.

Процедура

Эксперимент проходит в три этапа:

  1. Магнит удерживают, чтобы предотвратить его вращение, в то время как диск вращается вокруг своей оси. В результате гальванометр регистрирует постоянный ток . Поэтому устройство действует как генератор , по- разному называется генератором Фарадея, на диске Фарадея , или гомеополярные (или) однополярный генератор .
  2. Диск удерживается неподвижно, в то время как магнит вращается вокруг своей оси. В результате гальванометр не регистрирует ток.
  3. Диск и магнит вращаются вместе. Гальванометр регистрирует ток, как это было на шаге 1.

Почему это парадоксально?

Эксперимент описывается некоторыми как «парадокс», поскольку на первый взгляд кажется, что он нарушает закон электромагнитной индукции Фарадея, потому что поток через диск кажется одинаковым независимо от того, что вращается. Следовательно, прогнозируется, что ЭДС будет равна нулю во всех трех случаях вращения. Обсуждение ниже показывает, что эта точка зрения проистекает из неправильного выбора поверхности для расчета потока.

Парадокс появляется немного отличается от линий потока точки зрения: в модели Фарадея электромагнитной индукции, А магнитное поле , состоит из воображаемых линий от магнитного потока , аналогичны линиям , которые появляются , когда железные опилки посыпают на бумаге , и выдерживают около магнита. Предполагается, что ЭДС пропорциональна скорости резки линий потока. Если представить себе, что силовые линии возникают в магните, то они будут стационарными в рамке магнита, и вращение диска относительно магнита, будь то вращение магнита или диска, должно вызывать ЭДС, но вращение они оба вместе не должны.

Объяснение Фарадея

В модели электромагнитной индукции Фарадея цепь получает индуцированный ток, когда она разрезает линии магнитного потока. Согласно этой модели, диск Фарадея должен был работать, когда вращался либо диск, либо магнит, но не то и другое вместе. Фарадей попытался объяснить несогласие с наблюдениями, предположив, что поле магнита вместе с его магнитными линиями оставалось неподвижным при вращении магнита (полностью точная картина, но, возможно, не интуитивно понятная в модели магнитных линий). Другими словами, у линий потока есть своя собственная система отсчета. Как мы увидим в следующем разделе, современная физика (с момента открытия электрона ) не нуждается в картине потоковых линий и рассеивает парадокс.

Современные объяснения

Принимая во внимание обратный путь

На этапе 2 , поскольку ток не наблюдается, можно сделать вывод, что магнитное поле не вращалось вместе с вращающимся магнитом. (Действует или не действует эффективно или относительно, сила Лоренца равна нулю, поскольку v равно нулю относительно лабораторной системы отсчета. Таким образом, измерения тока из лабораторной системы отсчета отсутствуют.) Использование уравнения Лоренца для объяснения этого парадокса привело к В литературе обсуждается вопрос о том, вращается ли магнитное поле вместе с магнитом. Поскольку сила, действующая на заряды, выражаемая уравнением Лоренца, зависит от относительного движения магнитного поля (т. Е. Лабораторной рамы) к проводнику, в котором расположена ЭДС, было высказано предположение, что в случае, когда магнит вращается вместе с диском, но напряжение все еще развивается, магнитное поле (т.е. лабораторная рамка) не должно поэтому вращаться вместе с магнитным материалом (конечно, поскольку это лабораторная рамка), в то время как эффективное определение рамки магнитного поля или «эффективное / относительное вращение поля» поворачивается без относительного движения относительно проводящего диска.

Тщательное размышление показало, что если предположить, что магнитное поле вращается вместе с магнитом, а магнит вращается вместе с диском, ток все равно должен создаваться, а не за счет ЭДС в диске (нет относительного движения между диском и магнитом). но во внешней цепи, соединяющей щетки, которая фактически находится в относительном движении относительно вращающегося магнита. (Кисти находятся в лабораторной раме.)

Этот механизм согласуется с наблюдениями, касающимися обратных путей: ЭДС генерируется всякий раз, когда диск движется относительно обратного пути, независимо от вращения магнита. Фактически было показано, что до тех пор, пока токовая петля используется для измерения наведенных ЭДС от движения диска и магнита, невозможно сказать, вращается ли магнитное поле вместе с магнитом или нет. (Это зависит от определения, движение поля может быть определено только эффективно / относительно. Если вы придерживаетесь точки зрения, что поток поля является физическим объектом, он вращается или зависит от того, как он генерируется. Но это не меняет что используется в формуле Лоренца, особенно v , скорость носителя заряда относительно системы отсчета, в которой происходит измерение, и напряженность поля изменяется согласно теории относительности в любой точке пространства-времени.)

Было предложено несколько экспериментов с использованием электростатических измерений или электронных лучей для решения этой проблемы, но, по-видимому, ни один из них до сих пор не был успешно проведен. {2}}} \ mathbf {v} \ times \ mathbf {E}}

На самом нижнем уровне полная сила Лоренца является совокупным результатом электрических полей E и магнитных полей B каждого заряда, действующих на каждый другой заряд.

Когда магнит вращается, но силовые линии неподвижны, а проводник неподвижен

Рассмотрим частный случай, когда цилиндрический проводящий диск неподвижен, а цилиндрический магнитный диск вращается. В такой ситуации средняя скорость v зарядов в проводящем диске изначально равна нулю, и поэтому магнитная сила F = q v × B равна 0, где v — средняя скорость заряда q контура относительно системы отсчета. где производятся измерения, а q — заряд электрона.

Когда магнит и магнитопроводы неподвижны, а проводник вращается

После открытия электрона и влияющих на него сил стало возможным микроскопическое разрешение парадокса. См. Рис. 1. Металлические части устройства являются проводящими и ограничивают ток из-за электронного движения внутри металлических границ. Все электроны , которые двигаются в опыте магнитного поля силы Лоренца из F = д V × B , где V есть скорость электронов относительно рамы , где проводят измерения, и д -заряд на электроне. Помните, не существует такой рамки, как «рамка электромагнитного поля». Кадр устанавливается на конкретную точку пространства-времени, а не на расширяющееся поле или линию потока как математический объект. Это другая проблема, если вы рассматриваете поток как физическую сущность (см. Квант магнитного потока ) или рассматриваете эффективное / относительное определение движения / вращения поля (см. Ниже). Эта заметка помогает разрешить парадокс.

Сила Лоренца перпендикулярна как скорости электронов, которая находится в плоскости диска, так и магнитному полю, которое перпендикулярно ( нормали к поверхности ) к диску. Электрон, покоящийся в рамке диска, движется по кругу вместе с диском относительно B-поля (то есть оси вращения или лабораторной рамки, помните примечание выше), и, таким образом, испытывает радиальную силу Лоренца. На рисунке 1 эта сила (действующая на положительный заряд, а не на электрон) направлена ​​наружу к ободу в соответствии с правилом правой руки.

Конечно, эта радиальная сила, которая является причиной тока, создает радиальную составляющую скорости электронов, генерируя, в свою очередь, свою собственную составляющую силы Лоренца, которая противодействует круговому движению электронов, стремясь замедлить вращение диска, но электроны сохраняют компонент кругового движения, который продолжает управлять током за счет радиальной силы Лоренца.

Использование специальных приемов с законом Фарадея

Поток через часть пути от щетки на ободе, через внешнюю петлю и ось к центру диска всегда равен нулю, потому что магнитное поле находится в плоскости этого пути (не перпендикулярно ему), нет независимо от того, что вращается, поэтому интегрированная ЭДС вокруг этой части пути всегда равна нулю. Поэтому внимание сосредоточено на участке пути от оси через диск до щетки на ободе.

Закон индукции Фарадея можно выразить словами:

Индуцированная электродвижущая сила или ЭДС в любой замкнутой цепи равна скорости изменения магнитного потока через цепь во времени.

Математически закон формулируется:

E знак равно — d Φ B d т знак равно — d d т ∬ Σ ( т ) d А ⋅ B ( р ,   т )   , {\ displaystyle {\ mathcal {E}} = — {\ frac {d \ Phi _ {B}} {dt}} = — {\ frac {d} {dt}} \ iint _ {\ Sigma (t)} d \ mathbf {A} \ cdot \ mathbf {B} (\ mathbf {r}, \ t) \,}

где Φ B — поток, а d A — векторный элемент площади движущейся поверхности Σ ( t ), ограниченной петлей, вокруг которой должна быть найдена ЭДС.

Рисунок 2: Два возможных цикла для поиска ЭДС: геометрически простой путь прост в использовании, но другой обеспечивает такую ​​же ЭДС. {2}} {2}} \ omega \,}

при ω = / dt угловая скорость вращения. Знак выбирается на основании закона Ленца : поле, создаваемое движением, должно противодействовать изменению потока, вызванному вращением. Например, схема с радиальным сегментом на Рисунке 2 согласно правилу правой руки добавляет к приложенному B-полю, стремясь увеличить потокосцепление. Это говорит о том, что поток через этот путь уменьшается из-за вращения, поэтому / dt отрицательно.

Этот результат отсечения магнитного потока для ЭДС можно сравнить с расчетом работы, совершаемой на единицу заряда, заставляющую бесконечно малый пробный заряд пересечь гипотетическую линию, с использованием силы Лоренца на единицу заряда на радиусе r , а именно | v × B | = Bv = Brω :

E знак равно ∫ 0 р d р B р ω знак равно р 2 2 B ω   , {\ displaystyle {\ mathcal {E}} = \ int _ {0} ^ {R} drBr \ omega = {\ frac {R ^ {2}} {2}} B \ omega \,}

что и есть тот же результат.

Вышеупомянутая методология нахождения потока, отсекаемого цепью, формализована в законе потока путем правильной обработки производной по времени ограничивающей поверхности Σ ( t ). Конечно, производная по времени интеграла с ограничениями, зависящими от времени, — это не просто производная по времени одного подынтегрального выражения, о чем часто забывают; см. интегральное правило Лейбница и силу Лоренца .

При выборе поверхности Σ ( t ) ограничения заключаются в том, что (i) она должна быть ограничена замкнутой кривой, вокруг которой должна быть найдена ЭДС, и (ii) она должна захватывать относительное движение всех движущихся частей схема. Категорически не требуется, чтобы ограничивающая кривая соответствовала физической линии протекания тока. С другой стороны, индукция — это относительное движение, и траектория явно должна улавливать любое относительное движение. В случае, подобном Рисунку 1, где часть пути тока распределена по области в пространстве, ЭДС, управляющая током, может быть найдена с использованием множества путей. На рисунке 2 показаны две возможности. Все пути включают очевидную петлю возврата, но на диске показаны два пути: один геометрически простой путь, другой извилистый. Мы можем выбирать любой путь, который нам нравится, но часть любого допустимого пути фиксируется в самом диске и вращается вместе с диском. Поток вычисляется через весь путь, обратный контур плюс дисковый сегмент и определяется скорость его изменения.

Рисунок 3: Отображение диска Фарадея в пример скользящего проводящего прямоугольника. Диск рассматривается как кольцо; он разрезается по радиусу и сгибается, чтобы получился прямоугольник.

В этом примере все эти пути приводят к одинаковой скорости изменения магнитного потока и, следовательно, к одной и той же ЭДС. Чтобы дать некоторое представление об этой независимости пути, на рисунке 3 диск Фарадея развернут на полосу, что делает его похожим на задачу скользящего прямоугольника. В случае скользящего прямоугольника становится очевидным, что характер протекания тока внутри прямоугольника не зависит от времени и, следовательно, не имеет отношения к скорости изменения магнитного потока, соединяющего цепь. Нет необходимости точно учитывать, как ток проходит через прямоугольник (или диск). Любой выбор пути, соединяющего верх и низ прямоугольника (ось-щетку на диске) и перемещающийся с прямоугольником (вращающийся вместе с диском), обеспечивает одинаковую скорость изменения магнитного потока и предсказывает ту же ЭДС. . Для диска эта оценка скорости изменения магнитного потока такая же, как и оценка, сделанная выше, на основе вращения диска мимо линии, соединяющей щетку с осью.

Конфигурация с обратным путем

«Движется» ли магнит, не имеет значения в этом анализе из-за магнитного потока, индуцированного в обратном пути. Важнейшее относительное движение — это движение диска и обратного пути, а не диска и магнита. Это становится более ясным, если использовать модифицированный диск Фарадея, в котором обратным путем является не провод, а другой диск. То есть установите два токопроводящих диска рядом друг с другом на одной оси и позвольте им иметь скользящий электрический контакт в центре и по окружности. Ток будет пропорционален относительному вращению двух дисков и не зависит от вращения магнита.

Конфигурация без обратного пути

Диск Фарадея также не может работать ни с гальванометром, ни без обратного пути. Когда диск вращается, электроны собираются вдоль обода и оставляют дефицит около оси (или наоборот). В принципе возможно измерить распределение заряда, например, с помощью электродвижущей силы, создаваемой между ободом и осью (хотя это не обязательно легко). Это разделение зарядов будет пропорционально относительной скорости вращения диска и магнита.

Парадоксы, в которых закон индукции Фарадея, кажется, предсказывает ненулевую ЭДС, но на самом деле предсказывает нулевую ЭДС

Эти парадоксы обычно разрешаются путем определения того, что кажущееся движение контура на самом деле является деконструкцией контура с последующим восстановлением контура на другом пути.

Дополнительное правило

В случае, когда вращается только диск, поток через цепь не изменяется, однако возникает электродвижущая сила, индуцированная вопреки закону Фарадея. Мы также можем показать пример, когда есть изменение потока, но нет индуцированного напряжения. На рисунке 5 (справа) показана установка, использованная в эксперименте Тилли. Это схема с двумя петлями или сетками. Гальванометр подключен к правому шлейфу, магнит в центре левого шлейфа, переключатель в левом шлейфе и переключатель между шлейфами. Начнем с того, что переключатель слева открыт, а переключатель справа — закрыт. Когда переключатель слева замкнут, а переключатель справа разомкнут, поле магнита не изменяется, но изменяется площадь цепи гальванометра. Это означает, что есть изменение потока. Однако гальванометр не отклонялся, что означает отсутствие наведенного напряжения, и закон Фарадея в этом случае не работает. Согласно А.Г. Келли, это предполагает, что индуцированное напряжение в эксперименте Фарадея возникает из-за «перерезания» цепи магнитными линиями, а не из-за «магнитной связи» или фактического изменения магнитного потока. Это следует из эксперимента Тилли, поскольку силовые линии в цепи не перемещаются и, следовательно, не индуцируется ток, хотя поток через цепь изменяется. {2}}}}

Таким образом, мы можем переписать силу на проводе 1 как:

F 21 год знак равно я 1 ∮ C 1 d л 1   × B 2 {\ displaystyle \ mathbf {F} _ {21} = I_ {1} \ oint _ {C_ {1}} d \ mathbf {l_ {1}} \ \ mathbf {\ times} \ mathbf {B} _ {2 }}

Теперь рассмотрим отрезок проводника, перемещаемый в постоянном магнитном поле. Проделанную работу можно найти по ссылке: d л {\ displaystyle d \ mathbf {l}} d р {\ displaystyle d \ mathbf {r}}

d W знак равно d F ⋅ d р {\ displaystyle dW = d \ mathbf {F} \ cdot d \ mathbf {r}}

Если мы подключим то, что нашли ранее, мы получим: d F {\ displaystyle d \ mathbf {F}}

d W знак равно ( я d л × B ) ⋅ d р {\ displaystyle dW = (идентификатор \ mathbf {l} \ mathbf {\ times} \ mathbf {B}) \ cdot d \ mathbf {r}}

Площадь смещения проводника составляет:

d S знак равно d р × d л {\ displaystyle d \ mathbf {S} = d \ mathbf {r} \ mathbf {\ times} d \ mathbf {l}}

Следовательно:

d W знак равно я B ⋅ d S знак равно я d Φ B {\ displaystyle dW = I \ mathbf {B} \ cdot d \ mathbf {S} = Id \ Phi _ {B}}

Дифференциальная работа также может быть выражена в терминах заряда и разности потенциалов : d q {\ displaystyle dq} V {\ displaystyle V}

d W знак равно V d q знак равно V я d т {\ displaystyle dW = Vdq = VIdt}

Уравнивая два уравнения для дифференциальной работы, мы приходим к закону Фарадея.

d Φ B знак равно V d т {\ displaystyle d \ Phi _ {B} = Vdt}

Более того, теперь мы видим, что это верно, только если не обращается в нуль. Это означает, что закон Фарадея действителен только в том случае, если выполняется работа по изменению потока. d W {\ displaystyle dW}

Математический способ подтвердить закон Фарадея в подобных ситуациях — это обобщить определение ЭДС, как в доказательстве закона индукции Фарадея :

E M F знак равно ∮ ( E + v × B ) ⋅ d ℓ {\ displaystyle \ mathrm {EMF} = \ oint \ left (\ mathbf {E} + \ mathbf {v} \ times \ mathbf {B} \ right) \ cdot {\ text {d}} {\ boldsymbol {\ ell }}}

Гальванометр обычно измеряет только первый член в ЭДС, который вносит вклад в ток в цепи, хотя иногда он может измерять включение второго члена, например, когда второй член вносит вклад в часть тока, который гальванометр измеряет как ЭДС движения, например, в эксперимент с диском Фарадея. В приведенной выше ситуации первый член равен нулю, и только первый член ведет ток, который измеряет гальванометр, поэтому наведенное напряжение отсутствует. Однако закон Фарадея по-прежнему остается в силе, поскольку очевидное изменение магнитного потока относится ко второму члену в приведенном выше обобщении ЭДС. Но гальванометром это не измеряется. Помните, что это локальная скорость точки в цепи, а не носитель заряда. В конце концов, обе / все эти ситуации согласуются с заботой об относительности и микроструктуре материи и / или полнотой уравнения Максвелла и формулы Лоренца, или их комбинации, гамильтоновой механики . v {\ displaystyle \ mathbf {v}}

Смотрите также

Рекомендации

дальнейшее чтение

  • Майкл Фарадей, «Экспериментальные исследования электричества», том I, первая серия, 1831 г. в «Великих книгах западного мира», том 45, Р.М. Хатчинс, изд., Encyclopædia Britannica, Inc., Чикагский университет, 1952 г. [1]
  • «Электромагнитная индукция: физика и воспоминания» (PDF) Джузеппе Джулиани — детали силы Лоренца в диске Фарадея
  • «Униполярное электрическое динамо» — содержит вывод уравнения для ЭДС диска Фарадея.
  • Колонка Дона Ланкастера «Tech Musings», февраль 1998 г. — о практической неэффективности диска Фарадея.
  • «Последняя загадка Фарадея: вращается ли поле с помощью магнита?» (PDF) — противоположная теория, но содержит полезные ссылки на эксперименты Фарадея.
  • PJ Scanlon, RN Henriksen и JR Allen, «Подходы к электромагнитной индукции», Am. J. Phys. 37, 698–708 (1969). — описывает, как применить закон Фарадея к диску Фарадея
  • Хорхе Гуала-Вальверде, Педро Маццони, Рикардо Ахиллес «Униполярный двигатель: настоящий релятивистский двигатель», Am. J. Phys. 70 (10), 1052–1055 (октябрь 2002 г.). — утверждает, что только сила Лоренца может объяснить диск Фарадея, и описывает некоторые экспериментальные доказательства этого
  • Фрэнк Манли, вызовы правилу потока Фарадея, Am. J. Phys. 72, 1478 (2004). — обновленное обсуждение концепций в ссылке Scanlon выше.
  • Ричард Фейнман, Роберт Лейтон, Мэтью Сэндс, «Лекции Фейнмана по физике, том II», глава 17 — В дополнение к «парадоксу» Фарадея (когда связанный поток не изменяется, но индуцируется ЭДС), он описывает «качающиеся пластины». «эксперимент, в котором связанный поток изменяется, но не индуцируется ЭДС. Он показывает, что правильная физика всегда дается комбинацией силы Лоренца с уравнением Максвелла – Фарадея (см. Рамку для цитат), и создает эти два собственных «парадокса».
  • Вращение магнитного поля Ваня Янезич — описывает простой эксперимент, который может сделать каждый. Поскольку в нем участвуют только два тела, его результат менее неоднозначен, чем эксперименты Фарадея, Келли и Гуала-Вальверде с тремя телами.
  • У. Ф. Хьюз и Ф. Дж. Янг, Электромагнитодинамика жидкостей, John Wiley & Sons (1965) LCCC # 66-17631. Главы 1. Основы специальной теории относительности и 2. Электродинамика движущихся сред. Из этих глав можно проработать все проблемы наведенной ЭДС и объяснить все связанные с ними парадоксы, обнаруженные в литературе.

§23. Закон электромагнитной индукции Фарадея

В 1831 г. Фарадей экспериментально открыл явление электромагнитной индукции. Суть явления состояла в том, что если через замкнутый контур происходило изменение магнитного потока, то в контуре возникала электродвижущая сила, приводящая к возникновению замкнутого тока. Этот ток был назван индукционным током. Правило, устанавливающее направление индукционного тока было сформулировано в 1833г. Э. Х. Ленцем (1804 — 1865) и называется правилом Ленца. Оно гласит: индукционный ток направлен так, что создаваемый им магнитный поток стремится компенсировать изменение магнитного потока, вызывающего данный ток.

Опыты Фарадея состояли в следующем: катушка индуктивности подключалась к чувствительному гальванометру и в катушку вдвигался и выдвигался постоянный магнит.

Из опытов следовало, что

. Но сила тока зависит еще и от сопротивления контура. Поэтому закон электромагнитной индукции формулируется не для индукционного тока, а для причины, вызывающий этот ток, т. е. для . В 1845г. Ф. Э. Нейман (1799 — 1895) дал математическое определение закона электромагнитной индукции в современной форме: (23.1)

Хотя внешне формулы (22.6) и (23.1) одинаковы, между ними существует принципиальное различие. Возникновение

в (22.6) связано с движением проводников в магнитном поле и с действием на заряды силы Лоренца. Тогда как в (23.1) на заряды в контуре действует электрическое поле, причем сам контур лишь только инструмент или прибор, который может обнаружить это изменяющееся электрическое поле, которое возникает в пространстве. Следовательно закон Фарадея отражает новое физическое явление, а именно: изменяющееся магнитное поле порождает изменяющееся электрическое поле. А это означает, что электрическое поле порождается не только зарядами, но и изменяющимся магнитным полем. Закон электромагнитной индукции является фундаментальным законом природы.

Дифференциальная формулировка закона

, а тогда магнитный поток , а ..

К левой части применим формулу Стокса. Тогда

. После того как перенесем все слагаемые в одну сторону получим:

В силу произвольности

можно заключить, что подынтегральная функция равна нулю, а значит (23.2)

Уравнение (23.2) является дифференциальной формой закона электромагнитной индукции. В переменных магнитных полях

, а значит и следовательно, в отличие от электростатического поля, порождаемого неподвижными зарядами, переменное электрическое поле не является потенциальным и работа при перемещении заряда по замкнутому контуру не равна нулю:.

Так как закон электромагнитной индукции не затрагивает закона порождения магнитного поля, то уравнение (18.6)

остается в силе, а значит в силе остается и выражение (19.2): .

Если подставить (19.2) в (23.2), то

, а значит. (23.3)

Отсюда следует, что в переменных полях потенциальным является вектор

, а значит он равен градиенту скалярной функции, т. е., а значит. (23.4)

Второе слагаемое в (23.4) означает, что электрическое поле может порождаться неподвижными зарядами, а первое означает, что электрическое поле может порождаться переменным магнитным полем.

Магнитный поток, индукция и закон Фарадея

Индуцированные ЭДС и магнитный поток

Закон индукции Фарадея гласит, что электродвижущая сила индуцируется изменением магнитного потока.

Цели обучения

Объясните взаимосвязь между магнитным полем и электродвижущей силой

Основные выводы

Ключевые моменты
  • Это изменение потока магнитного поля, которое приводит к возникновению электродвижущей силы (или напряжения).
  • Магнитный поток (часто обозначаемый Φ или Φ B ), проходящий через поверхность, является составляющей магнитного поля, проходящего через эту поверхность.
  • В самом общем виде магнитный поток определяется как [латекс] \ Phi _ {\ text {B}} = \ iint _ {\ text {A}} \ mathbf {\ text {B}} \ cdot \ text {d} \ mathbf {\ text {A}} [/ latex]. Это интеграл (сумма) всего магнитного поля, проходящего через бесконечно малые элементы площади dA.
Ключевые термины
  • векторная площадь : вектор, величина которого соответствует рассматриваемой области, а направление перпендикулярно площади поверхности.
  • гальванометр : аналоговое измерительное устройство, обозначенное буквой G, которое измеряет ток, используя отклонение стрелки, вызванное силой магнитного поля, действующей на провод с током.

Индуцированная ЭДС

Аппарат, использованный Фарадеем для демонстрации того, что магнитные поля могут создавать токи, показан на следующем рисунке. Когда переключатель замкнут, магнитное поле создается в катушке в верхней части железного кольца и передается (или направляется) на катушку в нижней части кольца.Гальванометр используется для обнаружения любого тока, наведенного в отдельной катушке внизу.

Аппарат Фарадея : Это аппарат Фарадея для демонстрации того, что магнитное поле может производить ток. Изменение поля, создаваемого верхней катушкой, вызывает ЭДС и, следовательно, ток в нижней катушке. Когда переключатель разомкнут и замкнут, гальванометр регистрирует токи в противоположных направлениях. Когда переключатель остается замкнутым или разомкнутым, через гальванометр не течет ток.

Было обнаружено, что каждый раз, когда переключатель замыкается, гальванометр обнаруживает ток в одном направлении в катушке внизу. Каждый раз при размыкании переключателя гальванометр обнаруживает ток в противоположном направлении. Интересно, что если переключатель остается замкнутым или разомкнутым в течение некоторого времени, через гальванометр нет тока. Замыкание и размыкание переключателя индуцирует ток. Это изменение магнитного поля, которое создает ток. Более важным, чем текущий ток, является вызывающая его электродвижущая сила (ЭДС).Ток является результатом ЭДС, индуцированной изменяющимся магнитным полем, независимо от того, есть ли путь для протекания тока.

Магнитный поток

Магнитный поток (часто обозначаемый Φ или Φ B ), проходящий через поверхность, является составляющей магнитного поля, проходящего через эту поверхность. Магнитный поток через некоторую поверхность пропорционален количеству силовых линий, проходящих через эту поверхность. Магнитный поток, проходящий через поверхность с векторной площадью A, составляет

[латекс] \ Phi_ \ text {B} = \ mathbf {\ text {B}} \ cdot \ mathbf {\ text {A}} = \ text {BA} \ cos \ theta [/ latex],

, где B — величина магнитного поля (в Тесла, Тл), A — площадь поверхности, а θ — угол между силовыми линиями магнитного поля и нормалью (перпендикулярно) к A.

Для переменного магнитного поля мы сначала рассмотрим магнитный поток [латекс] \ text {d} \ Phi _ \ text {B} [/ latex] через бесконечно малый элемент площади dA, где мы можем считать поле постоянным:

Изменяющееся магнитное поле : Каждая точка на поверхности связана с направлением, называемым нормалью к поверхности; магнитный поток, проходящий через точку, тогда является составляющей магнитного поля вдоль этого нормального направления.

[латекс] \ text {d} \ Phi_ \ text {B} = \ mathbf {\ text {B}} \ cdot \ text {d} \ mathbf {\ text {A}} [/ latex]

Общая поверхность A затем может быть разбита на бесконечно малые элементы, и тогда полный магнитный поток через поверхность равен интегралу поверхности

[латекс] \ Phi_ \ text {B} = \ iint_ \ text {A} \ mathbf {\ text {B}} \ cdot \ text {d} \ mathbf {\ text {A}} [/ latex].

Закон индукции Фарадея и закон Ленца

Закон индукции Фарадея гласит, что ЭДС, вызванная изменением магнитного потока, равна [латексу] \ text {EMF} = — \ text {N} \ frac {\ Delta \ Phi} {\ Delta \ text {t}} [ / латекс], когда поток изменяется на Δ за время Δt.

Цели обучения

Выразите закон индукции Фарадея в форме уравнения

Основные выводы

Ключевые моменты
  • Минус в законе Фарадея означает, что ЭДС создает ток I и магнитное поле B, которые противодействуют изменению потока Δ, известному как закон Ленца.
  • Закон индукции Фарадея является основным принципом работы трансформаторов, индукторов и многих типов электродвигателей, генераторов и соленоидов.
  • Закон Фарадея гласит, что ЭДС, вызванная изменением магнитного потока, зависит от изменения магнитного потока Δ, времени Δt и количества витков катушек.
Ключевые термины
  • электродвижущая сила : (ЭДС) — напряжение, генерируемое батареей или магнитной силой в соответствии с законом Фарадея.Она измеряется в вольтах, а не в ньютонах, и поэтому на самом деле не является силой.
  • Соленоид : Катушка с проволокой, которая действует как магнит, когда через нее протекает электрический ток.
  • поток : Скорость передачи энергии (или другой физической величины) через данную поверхность, в частности электрического или магнитного потока.

Закон индукции Фарадея

Закон индукции Фарадея — это основной закон электромагнетизма, который предсказывает, как магнитное поле будет взаимодействовать с электрической цепью, создавая электродвижущую силу (ЭДС).Это основной принцип работы трансформаторов, индукторов и многих типов электродвигателей, генераторов и соленоидов.

Эксперименты Фарадея показали, что ЭДС, вызванная изменением магнитного потока, зависит только от нескольких факторов. Во-первых, ЭДС прямо пропорциональна изменению потока Δ. Во-вторых, ЭДС является наибольшей, когда изменение во времени Δt наименьшее, то есть ЭДС обратно пропорциональна Δt. Наконец, если катушка имеет N витков, будет создаваться ЭДС, которая в N раз больше, чем для одиночной катушки, так что ЭДС прямо пропорциональна N.Уравнение для ЭДС, вызванной изменением магнитного потока, равно

[латекс] \ text {EMF} = — \ text {N} \ frac {\ Delta \ Phi} {\ Delta \ text {t}} [/ latex].

Это соотношение известно как закон индукции Фарадея. Единицы измерения ЭДС, как обычно, — вольты.

Закон Ленца

Знак минус в законе индукции Фарадея очень важен. Минус означает, что ЭДС создает ток I и магнитное поле B, которые противодействуют изменению потока Δ, известному как закон Ленца. Направление (обозначенное знаком минус) ЭМП настолько важно, что оно названо законом Ленца в честь русского Генриха Ленца (1804–1865), который, подобно Фарадею и Генри, независимо исследовал аспекты индукции.Фарадей знал о направлении, но Ленц указал его, поэтому ему приписывают это открытие.

Закон Ленца : (a) Когда стержневой магнит вставляется в катушку, сила магнитного поля в катушке увеличивается. Ток, наведенный в катушке, создает другое поле в направлении, противоположном стержневому магниту, чтобы противодействовать увеличению. Это один из аспектов закона Ленца: индукция препятствует любому изменению потока. (b) и (c) — две другие ситуации. Убедитесь сами, что показанное направление индуцированной катушки B действительно противостоит изменению магнитного потока и что показанное направление тока согласуется с правилом правой руки.

Энергосбережение

Закон Ленца является проявлением сохранения энергии. Индуцированная ЭДС создает ток, который противодействует изменению потока, потому что изменение потока означает изменение энергии. Энергия может входить или уходить, но не мгновенно. Закон Ленца — это следствие. Когда изменение начинается, закон говорит, что индукция противодействует и, таким образом, замедляет изменение. Фактически, если бы индуцированная ЭДС была в том же направлении, что и изменение потока, была бы положительная обратная связь, которая не давала бы нам бесплатную энергию из любого видимого источника — закон сохранения энергии был бы нарушен.

Движение ЭДС

Движение в магнитном поле, которое является стационарным относительно Земли, вызывает ЭДС движения (электродвижущую силу).

Цели обучения

Определить процесс, вызывающий двигательную электродвижущую силу

Основные выводы

Ключевые моменты
  • Закон индукции Фарадея можно использовать для расчета ЭДС движения, когда изменение магнитного потока вызвано движущимся элементом в системе.
  • То, что движущееся магнитное поле создает электрическое поле (и, наоборот, движущееся электрическое поле создает магнитное поле), является частью причины, по которой электрические и магнитные силы теперь рассматриваются как разные проявления одной и той же силы.
  • Любое изменение магнитного потока индуцирует электродвижущую силу (ЭДС), противодействующую этому изменению — процесс, известный как индукция. Движение — одна из основных причин индукции.
Ключевые термины
  • электродвижущая сила : (ЭДС) — напряжение, генерируемое батареей или магнитной силой в соответствии с законом Фарадея. Она измеряется в вольтах, а не в ньютонах, и поэтому на самом деле не является силой.
  • магнитный поток : Мера силы магнитного поля в заданной области.
  • индукция : Генерация электрического тока изменяющимся магнитным полем.

Как было замечено в предыдущих атомах, любое изменение магнитного потока индуцирует электродвижущую силу (ЭДС), противодействующую этому изменению — процесс, известный как индукция. Движение — одна из основных причин индукции. Например, магнит, перемещенный к катушке, индуцирует ЭДС, а катушка, перемещенная к магниту, создает аналогичную ЭДС. В этом Атоме мы концентрируемся на движении в магнитном поле, которое является стационарным относительно Земли, производя то, что в общих чертах называется ЭДС движения.

Движение ЭДС

Рассмотрим ситуацию, показанную на. Стержень перемещается со скоростью v по паре проводящих рельсов, разделенных расстоянием в однородном магнитном поле B. Рельсы неподвижны относительно B и соединены с неподвижным резистором R ( резистором может быть что угодно от лампочки до вольтметра). Учтите площадь, ограниченную подвижным стержнем, направляющими и резистором. B перпендикулярно этой области, и площадь увеличивается по мере перемещения стержня. Таким образом, магнитный поток между рельсами, стержнем и резистором увеличивается.Когда поток изменяется, ЭДС индуцируется согласно закону индукции Фарадея.

ЭДС движения : (a) ЭДС движения = Bℓv индуцируется между рельсами, когда этот стержень перемещается вправо в однородном магнитном поле. Магнитное поле B направлено внутрь страницы, перпендикулярно движущемуся стержню и рельсам и, следовательно, к области, окружающей их. (б) Закон Ленца дает направление индуцированного поля и тока, а также полярность наведенной ЭДС. Поскольку поток увеличивается, индуцированное поле направлено в противоположном направлении или за пределы страницы.Правило правой руки дает указанное направление тока, и полярность стержня будет управлять таким током.

Чтобы найти величину ЭДС, индуцированной вдоль движущегося стержня, мы используем закон индукции Фарадея без знака:

[латекс] \ text {EMF} = \ text {N} \ frac {\ Delta \ Phi} {\ Delta \ text {t}} [/ latex].

В этом уравнении N = 1 и поток Φ = BAcosθ. Имеем θ = 0º и cosθ = 1, так как B перпендикулярно A. Теперь Δ = Δ (BA) = BΔA, поскольку B однородна. Отметим, что площадь, заметаемая стержнем, равна ΔA = ℓx.Ввод этих величин в выражение для ЭДС дает:

[латекс] \ text {EMF} = \ frac {\ text {B} \ Delta \ text {A}} {\ Delta \ text {t}} = \ text {B} \ frac {\ text {l} \ Дельта \ text {x}} {\ Delta \ text {t}} = \ text {Blv} [/ latex].

Чтобы найти направление индуцированного поля, направление тока и полярность наведенной ЭДС, мы применяем закон Ленца, как объяснено в Законе индукции Фарадея: Закон Ленца. Как видно на рис. 1 (b), уровень освещенности увеличивается, так как увеличивается закрытая площадь.Таким образом, индуцированное поле должно противостоять существующему и быть вне страницы. (Правило правой руки требует, чтобы я вращался против часовой стрелки, что, в свою очередь, означает, что вершина стержня положительна, как показано.)

Зависимость электрического поля от магнитного

Между электрической и магнитной силой существует множество связей. То, что движущееся магнитное поле создает электрическое поле (и, наоборот, движущееся электрическое поле создает магнитное поле), является частью причины, по которой электрические и магнитные силы теперь рассматриваются как различных проявлений одной и той же силы (впервые замечено Альбертом Эйнштейном) .Это классическое объединение электрических и магнитных сил в так называемую электромагнитную силу является источником вдохновения для современных усилий по объединению других основных сил.

Обратная ЭДС, вихревые токи и магнитное демпфирование

Обратная ЭДС, вихревые токи и магнитное затухание — все это происходит из-за наведенной ЭДС и может быть объяснено законом индукции Фарадея.

Цели обучения

Объясните взаимосвязь между двигательной электродвижущей силой, вихревыми токами и магнитным демпфированием

Основные выводы

Ключевые моменты
  • Входной ЭДС, которая питает двигатель, может противостоять собственная ЭДС двигателя, называемая обратной ЭДС двигателя.
  • Если ЭДС движения может вызвать токовую петлю в проводнике, ток называется вихревым током.
  • Вихревые токи могут вызывать значительное сопротивление, называемое магнитным демпфированием, при движении.
Ключевые термины
  • электродвижущая сила : (ЭДС) — напряжение, генерируемое батареей или магнитной силой в соответствии с законом Фарадея. Она измеряется в вольтах, а не в ньютонах, и поэтому на самом деле не является силой.
  • Закон индукции Фарадея : Основной закон электромагнетизма, который предсказывает, как магнитное поле будет взаимодействовать с электрической цепью, создавая электродвижущую силу (ЭДС).

Задний ЭДС

Двигатели и генераторы очень похожи. (Прочтите наши атомы в разделах «Электрические генераторы» и «Электродвигатели».) Генераторы преобразуют механическую энергию в электрическую, а двигатели преобразуют электрическую энергию в механическую. Кроме того, двигатели и генераторы имеют одинаковую конструкцию. Когда катушка двигателя поворачивается, магнитный поток изменяется, и возникает электродвижущая сила (ЭДС), соответствующая закону индукции Фарадея. Таким образом, двигатель действует как генератор всякий раз, когда его катушка вращается.Это произойдет независимо от того, поворачивается ли вал под действием внешнего источника, например ременной передачи, или под действием самого двигателя. То есть, когда двигатель выполняет работу и его вал вращается, возникает ЭДС. Закон Ленца говорит нам, что наведенная ЭДС противодействует любому изменению, так что входной ЭДС, питающей двигатель, будет противодействовать самогенерируемая ЭДС двигателя, называемая обратной ЭДС двигателя.

Вихретоковый

Как обсуждалось в разделе «ЭДС движения», ЭДС движения индуцируется, когда проводник движется в магнитном поле или когда магнитное поле движется относительно проводника.Если подвижная ЭДС может вызвать токовую петлю в проводнике, мы называем этот ток вихревым. Вихревые токи могут вызывать значительное сопротивление движению, называемое магнитным затуханием.

Рассмотрим устройство, показанное на, которое раскачивает маятник между полюсами сильного магнита. Если боб металлический, то при входе в поле и выходе из поля он испытывает значительное сопротивление, что быстро гасит движение. Однако, если боб представляет собой металлическую пластину с прорезями, как показано на (b), эффект от магнита будет гораздо меньше.Заметного воздействия на боб из изолятора не наблюдается.

Устройство для исследования вихревых токов и магнитного затухания : Обычное демонстрационное устройство по физике для исследования вихревых токов и магнитного затухания. (а) Движение металлического маятника, раскачивающегося между полюсами магнита, быстро затухает под действием вихревых токов. (b) Имеется незначительное влияние на движение металлического боба с прорезями, что означает, что вихревые токи становятся менее эффективными. (c) На непроводящем бобе также отсутствует магнитное затухание, поскольку вихревые токи чрезвычайно малы.

показывает, что происходит с металлической пластиной, когда она входит в магнитное поле и выходит из него. В обоих случаях он испытывает силу, противодействующую его движению. Когда он входит слева, поток увеличивается, и поэтому возникает вихревой ток (закон Фарадея) в направлении против часовой стрелки (закон Ленца), как показано. Только правая сторона токовой петли находится в поле, так что слева на нее действует беспрепятственная сила (правило правой руки). Когда металлическая пластина полностью находится внутри поля, вихревой ток отсутствует, если поле однородно, поскольку поток остается постоянным в этой области.Но когда пластина покидает поле справа, поток уменьшается, вызывая вихревой ток по часовой стрелке, который, опять же, испытывает силу слева, еще больше замедляя движение. Аналогичный анализ того, что происходит, когда пластина поворачивается справа налево, показывает, что ее движение также затухает при входе в поле и выходе из него.

Проводящая пластина, проходящая между полюсами магнита : более подробный взгляд на проводящую пластину, проходящую между полюсами магнита.Когда он входит в поле и выходит из него, изменение потока создает вихревой ток. Магнитная сила на токовой петле препятствует движению. Когда пластина полностью находится внутри однородного поля, нет ни тока, ни магнитного сопротивления.

Когда металлическая пластина с прорезями входит в поле, как показано на, ЭДС индуцируется изменением магнитного потока, но она менее эффективна, поскольку прорези ограничивают размер токовых петель. Более того, в соседних контурах есть токи в противоположных направлениях, и их эффекты нейтрализуются.Когда используется изолирующий материал, вихревые токи чрезвычайно малы, поэтому магнитное затухание на изоляторах незначительно. Если необходимо избегать вихревых токов в проводниках, они могут быть выполнены с прорезями или состоять из тонких слоев проводящего материала, разделенных изоляционными листами.

Вихревые токи, индуцированные в металлической пластине с прорезями : Вихревые токи, индуцированные в металлической пластине с прорезями, входящие в магнитное поле, образуют небольшие петли, и силы на них имеют тенденцию нейтрализоваться, тем самым делая магнитное сопротивление почти нулевым.

Изменение магнитного потока создает электрическое поле

Закон индукции Фарадея гласит, что изменение магнитного поля создает электрическое поле: [latex] \ varepsilon = — \ frac {\ partial \ Phi_ \ text {B}} {\ partial \ text {t}} [/ latex].

Цели обучения

Опишите взаимосвязь между изменяющимся магнитным полем и электрическим полем

Основные выводы

Ключевые моменты
  • Закон индукции Фарадея — это основной закон электромагнетизма, который предсказывает, как магнитное поле будет взаимодействовать с электрической цепью, создавая электродвижущую силу.
  • Альтернативная дифференциальная форма закона индукции Фарадея выражается в уравнении [latex] \ nabla \ times \ vec {\ text {E}} = — \ frac {\ partial \ vec {\ text {B}}} { \ partial \ text {t}} [/ latex].
  • Закон индукции Фарадея — одно из четырех уравнений Максвелла, управляющих всеми электромагнитными явлениями.
Ключевые термины
  • векторная область : вектор, величина которого соответствует рассматриваемой области и направление которого перпендикулярно плоскости.
  • Уравнения Максвелла : Набор уравнений, описывающих, как электрические и магнитные поля генерируются и изменяются друг другом, а также зарядами и токами.
  • Теорема Стокса : утверждение об интегрировании дифференциальных форм на многообразиях, которое одновременно упрощает и обобщает несколько теорем векторного исчисления.

Мы изучили закон индукции Фарадея в предыдущих атомах. Мы узнали взаимосвязь между наведенной электродвижущей силой (ЭДС) и магнитным потоком.Вкратце, закон гласит, что изменение магнитного поля [латекс] (\ frac {\ text {d} \ Phi_ \ text {B}} {\ text {dt}}) [/ latex] создает электрическое поле [латекс] (\ varepsilon) [/ latex], закон индукции Фарадея выражается как [latex] \ varepsilon = — \ frac {\ partial \ Phi_ \ text {B}} {\ partial \ text {t}} [/ latex], где [латекс] \ varepsilon [/ latex] — это индуцированная ЭДС, а [latex] \ Phi_ \ text {B} [/ latex] — магнитный поток. («N» опущено из нашего предыдущего выражения. Число витков катушки может быть включено в магнитный поток, поэтому коэффициент не является обязательным.) Закон индукции Фарадея — это основной закон электромагнетизма, который предсказывает, как магнитное поле будет взаимодействовать с электрической цепью, создавая электродвижущую силу (ЭДС). В этом Атоме мы узнаем об альтернативном математическом выражении закона.

Эксперимент Фарадея : эксперимент Фарадея, показывающий индукцию между витками проволоки: жидкая батарея (справа) обеспечивает ток, который течет через небольшую катушку (A), создавая магнитное поле. Когда катушки неподвижны, ток не индуцируется.Но когда малая катушка перемещается внутрь или из большой катушки (B), магнитный поток через большую катушку изменяется, вызывая ток, который регистрируется гальванометром (G).

Дифференциальная форма закона Фарадея

Магнитный поток [латекс] \ Phi_ \ text {B} = \ int_ \ text {S} \ vec {\ text {B}} \ cdot \ text {d} \ vec {\ text {A}} [/ латекс], где [латекс] \ vec {\ text {A}} [/ latex] — это векторная площадь над замкнутой поверхностью S. Устройство, которое может поддерживать разность потенциалов, несмотря на протекание тока, является источником электродвижущей силы. .(EMF) Математически определение [латекс] \ varepsilon = \ oint_ \ text {C} \ vec {\ text {E}} \ cdot \ text {d} \ vec {\ text {s}} [/ latex], где интеграл вычисляется по замкнутому циклу C.

Закон Фарадея теперь можно переписать [latex] \ oint_ \ text {C} \ vec {\ text {E}} \ cdot \ text {d} \ vec {\ text {s}} = — \ frac {\ partial} {\ partial \ text {t}} (\ int \ vec {\ text {B}} \ cdot \ text {d} \ vec {\ text {A}}) [/ latex]. Используя теорему Стокса в векторном исчислении, левая часть равна [latex] \ oint_ \ text {C} \ vec {\ text {E}} \ cdot \ text {d} \ vec {\ text {s}} = \ int_ \ text {S} (\ nabla \ times \ vec {\ text {E}}) \ cdot \ text {d} \ vec {\ text {A}} [/ latex].Также обратите внимание, что в правой части [latex] \ frac {\ partial} {\ partial \ text {t}} (\ int \ vec {\ text {B}} \ cdot \ text {d} \ vec {\ текст {A}}) = \ int \ frac {\ partial \ vec {\ text {B}}} {\ partial \ text {t}} \ cdot \ text {d} \ vec {\ text {A}} [ /латекс]. Таким образом, мы получаем альтернативную форму закона индукции Фарадея: [latex] \ nabla \ times \ vec {\ text {E}} = — \ frac {\ partial \ vec {\ text {B}}} {\ partial \ text {t}} [/ latex]. Это также называют дифференциальной формой закона Фарадея. Это одно из четырех уравнений Максвелла, управляющих всеми электромагнитными явлениями.

Электрогенераторы

Электрогенераторы преобразуют механическую энергию в электрическую; они индуцируют ЭДС, вращая катушку в магнитном поле.

Цели обучения

Объясните, как в электрогенераторах индуцируется электродвижущая сила.

Основные выводы

Ключевые моменты
  • Электрический генератор вращает катушку в магнитном поле, индуцируя ЭДС, заданную как функцию времени величиной ε = NABw sinωt.
  • Генераторы поставляют почти всю мощность для электрических сетей, которые обеспечивают большую часть мировой электроэнергии.
  • Двигатель становится генератором, когда его вал вращается.
Ключевые термины
  • электродвижущая сила : (ЭДС) — напряжение, генерируемое батареей или магнитной силой в соответствии с законом Фарадея. Она измеряется в вольтах, а не в ньютонах, и поэтому на самом деле не является силой.
  • турбина : Любая из различных вращающихся машин, которые используют кинетическую энергию непрерывного потока жидкости (жидкости или газа) для вращения вала.

Электрические генераторы — это устройства, преобразующие механическую энергию в электрическую.Они индуцируют электродвижущую силу (ЭДС), вращая катушку в магнитном поле. Это устройство, преобразующее механическую энергию в электрическую. Генератор заставляет электрический заряд (обычно переносимый электронами) проходить через внешнюю электрическую цепь. Возможные источники механической энергии включают в себя поршневой или турбинный паровой двигатель, воду, падающую через турбину или водяное колесо, двигатель внутреннего сгорания, ветряную турбину, ручной кривошип, сжатый воздух или любой другой источник механической энергии.Генераторы поставляют почти всю мощность для электрических сетей, которые обеспечивают большую часть мировой электроэнергии.

Паротурбинный генератор : современный паротурбинный генератор.

Базовая настройка

Рассмотрим схему, показанную на. Заряды в проводах петли испытывают магнитную силу, потому что они движутся в магнитном поле. Заряды в вертикальных проводах испытывают силы, параллельные проводу, вызывая токи. Однако те, кто находится в верхнем и нижнем сегментах, ощущают силу, перпендикулярную проводу; эта сила не вызывает тока.Таким образом, мы можем найти наведенную ЭДС, рассматривая только боковые провода. ЭДС движения задается равной ЭДС = Bℓv, где скорость v перпендикулярна магнитному полю B (см. Наш Атом в «ЭДС движения»). Здесь скорость находится под углом θ к B, так что ее составляющая, перпендикулярная B, равна vsinθ.

Схема электрического генератора : Генератор с одной прямоугольной катушкой, вращающейся с постоянной угловой скоростью в однородном магнитном поле, создает ЭДС, синусоидально изменяющуюся во времени.Обратите внимание, что генератор похож на двигатель, за исключением того, что вал вращается для выработки тока, а не наоборот.

Таким образом, в этом случае ЭДС, индуцированная с каждой стороны, равна ЭДС = Bℓvsinθ, и они направлены в одном направлении. Общая ЭДС [латекс] \ varepsilon [/ latex] вокруг петли тогда:

[латекс] \ varepsilon = 2 \ text {Blv} \ sin {\ theta} [/ latex].

Это выражение действительное, но оно не дает ЭДС как функцию времени. Чтобы найти зависимость ЭДС от времени, предположим, что катушка вращается с постоянной угловой скоростью ω.Угол θ связан с угловой скоростью соотношением θ = ωt, так что:

[латекс] \ varepsilon = 2 \ text {Blv} \ sin {\ omega \ text {t}} [/ latex].

Итак, линейная скорость v связана с угловой скоростью соотношением v = rω. Здесь r = w / 2, так что v = (w / 2) ω, и:

[латекс] \ varepsilon = 2 \ text {Bl} \ frac {\ text {w}} {2} \ omega \ sin {\ omega \ text {t}} = (\ text {lw}) \ text {B } \ omega \ sin {\ omega \ text {t}} [/ латекс].

Учитывая, что площадь петли A = ℓw, и учитывая N петель, мы находим, что:

[латекс] \ varepsilon = \ text {NABw} ~ \ sin {\ omega \ text {t}} [/ latex] — это ЭДС, индуцированная в катушке генератора N витков и площади A, вращающейся с постоянной угловой скоростью в однородное магнитное поле B.

Генераторы

, показанные в этом Atom, очень похожи на двигатели, показанные ранее. Это не случайно. Фактически, двигатель становится генератором, когда его вал вращается.

Электродвигатели

Электродвигатель — это устройство, преобразующее электрическую энергию в механическую.

Цели обучения

Объясните, как сила создается в электродвигателях

Основные выводы

Ключевые моменты
  • Большинство электродвигателей используют взаимодействие магнитных полей и токопроводящих проводов для создания силы.
  • Ток в проводнике состоит из движущихся зарядов. Следовательно, катушка с током в магнитном поле также будет ощущать силу Лоренца.
  • В двигателе катушка с током в магнитном поле испытывает силу с обеих сторон катушки, которая создает крутящую силу (называемую крутящим моментом), заставляющую ее вращаться.
Ключевые термины
  • Сила Лоренца : Сила, действующая на заряженную частицу в электромагнитном поле.
  • крутящий момент : вращательное или скручивающее действие силы; (Единица СИ ньютон-метр или Нм; британская единица измерения фут-фунт или фут-фунт)

Основные принципы работы двигателя такие же, как и у генератора, за исключением того, что двигатель преобразует электрическую энергию в механическую энергию (движение).(Сначала прочтите наш атом об электрических генераторах.) Большинство электродвигателей используют взаимодействие магнитных полей и проводников с током для создания силы. Электродвигатели используются в самых разных областях, таких как промышленные вентиляторы, нагнетатели и насосы, станки, бытовая техника, электроинструменты и дисководы.

Лоренц Форс

Если вы поместите движущуюся заряженную частицу в магнитное поле, на нее будет действовать сила, называемая силой Лоренца:

[латекс] \ text {F} = \ text {q} \ times \ text {v} \ times \ text {B} [/ latex]

Правило правой руки : Правило правой руки, показывающее направление силы Лоренца

, где v — скорость движущегося заряда, q — заряд, а B — магнитное поле.Ток в проводнике состоит из движущихся зарядов. Следовательно, катушка с током в магнитном поле также будет ощущать силу Лоренца. Для неподвижного прямолинейного токоведущего провода сила Лоренца составляет:

[латекс] \ text {F} = \ text {I} \ times \ text {L} \ times \ text {B} [/ latex]

, где F — сила (в ньютонах, Н), I — ток в проводе (в амперах, А), L — длина провода, находящегося в магнитном поле (в м). , а B — напряженность магнитного поля (в теслах, Тл).Направление силы Лоренца перпендикулярно как направлению потока тока, так и магнитного поля, и его можно найти с помощью правила правой руки, показанного на рисунке. Используя правую руку, направьте большой палец в направлении тока, и укажите указательным пальцем в направлении магнитного поля. Ваш третий палец теперь будет указывать в направлении силы.

Крутящий момент : Сила на противоположных сторонах катушки будет в противоположных направлениях, потому что заряды движутся в противоположных направлениях.Это означает, что катушка будет вращаться.

Механика двигателя

И двигатели, и генераторы можно объяснить с помощью катушки, вращающейся в магнитном поле. В генераторе катушка подключена к внешней цепи, которая затем включается. Это приводит к изменению потока, который индуцирует электромагнитное поле. В двигателе катушка с током в магнитном поле испытывает силу с обеих сторон катушки, которая создает крутящую силу (называемую крутящим моментом), заставляющую ее вращаться.Любая катушка, по которой проходит ток, может чувствовать силу в магнитном поле. Эта сила является силой Лоренца, действующей на движущиеся заряды в проводнике. Сила на противоположных сторонах катушки будет в противоположных направлениях, потому что заряды движутся в противоположных направлениях. Это означает, что катушка будет вращаться.

Индуктивность

Индуктивность — это свойство устройства, которое показывает, насколько эффективно оно индуцирует ЭДС в другом устройстве или на самом себе.

Цели обучения

Описание свойств катушки индуктивности с указанием взаимной индуктивности и самоиндукции

Основные выводы

Ключевые моменты
  • Взаимная индуктивность — это влияние двух устройств, индуцирующих друг в друге ЭДС.Изменение тока ΔI 1 / Δt в одном порождает ЭДС ЭДС2 в секунду: ЭДС 2 = -M ΔI 1 / Δt, где M определяется как взаимная индуктивность между двумя устройствами.
  • Самоиндукция — это эффект того, что устройство вызывает саму по себе ЭДС.
  • Устройство, которое демонстрирует значительную самоиндукцию, называется индуктором, и ЭДС, индуцированная в нем изменением тока через него, равна ЭДС = −L ΔI / Δt.
Ключевые термины
  • Закон индукции Фарадея : основной закон электромагнетизма, который предсказывает, как магнитное поле будет взаимодействовать с электрической цепью, создавая электродвижущую силу (ЭДС).
  • трансформатор : статическое устройство, которое передает электрическую энергию от одной цепи к другой с помощью магнитной связи. Их основное назначение — передача энергии между различными уровнями напряжения, что позволяет выбирать наиболее подходящее напряжение для выработки, передачи и распределения электроэнергии по отдельности.

Индукция — это процесс, при котором ЭДС индуцируется изменением магнитного потока. Трансформаторы, например, спроектированы так, чтобы быть особенно эффективными для создания желаемого напряжения и тока с очень небольшими потерями энергии в другие формы (см. Наш Atom в разделе «Трансформаторы.«) Есть ли полезная физическая величина, связанная с тем, насколько« эффективно »данное устройство? Ответ — да, и эта физическая величина называется индуктивностью.

Взаимная индуктивность

Взаимная индуктивность — это влияние закона индукции Фарадея для одного устройства на другое, например, первичная катушка, при передаче энергии вторичной обмотке в трансформаторе. Посмотрите, где простые катушки наводят друг на друга ЭДС.

Взаимная индуктивность катушек : Эти катушки могут вызывать ЭДС друг в друге, как неэффективный трансформатор.Их взаимная индуктивность M указывает на эффективность связи между ними. Здесь видно, что изменение тока в катушке 1 вызывает ЭДС в катушке 2. (Обратите внимание, что «E2 индуцированная» представляет наведенную ЭДС в катушке 2.)

Во многих случаях, когда геометрия устройств фиксирована, магнитный поток изменяется за счет изменения тока. Поэтому мы концентрируемся на скорости изменения тока, ΔI / Δt, как на причине индукции. Изменение тока I 1 в одном устройстве, катушка 1, индуцирует ЭДС 2 в другом.Мы выражаем это в форме уравнения как

[латекс] \ text {EMF} _2 = — \ text {M} \ frac {\ Delta \ text {I} _1} {\ Delta \ text {t}} [/ latex],

, где M определяется как взаимная индуктивность между двумя устройствами. Знак минус является выражением закона Ленца. Чем больше взаимная индуктивность M, тем эффективнее связь.

Природа здесь симметрична. Если мы изменим ток I2 в катушке 2, мы индуцируем ЭДС 1 в катушке 1, которая равна

[латекс] \ text {EMF} _1 = — \ text {M} \ frac {\ Delta \ text {I} _2} {\ Delta \ text {t}} [/ latex],

, где M то же, что и для обратного процесса.Трансформаторы работают в обратном направлении с такой же эффективностью или взаимной индуктивностью M.

Самоиндуктивность

Самоиндукция, действие закона индукции Фарадея устройства на самого себя, также существует. Когда, например, увеличивается ток через катушку, магнитное поле и магнитный поток также увеличиваются, вызывая противоэдс, как того требует закон Ленца. И наоборот, если ток уменьшается, индуцируется ЭДС, которая препятствует уменьшению. Большинство устройств имеют фиксированную геометрию, поэтому изменение магнитного потока полностью связано с изменением тока ΔI через устройство.Индуцированная ЭДС связана с физической геометрией устройства и скоростью изменения тока. Выдается

[латекс] \ text {EMF} = — \ text {L} \ frac {\ Delta \ text {I}} {\ Delta \ text {t}} [/ latex],

где L — самоиндукция устройства. Устройство, которое демонстрирует значительную самоиндукцию, называется индуктором. Опять же, знак минус является выражением закона Ленца, указывающего на то, что ЭДС препятствует изменению тока.

Количественная интерпретация ЭДС движения

A ЭДС движения — это электродвижущая сила (ЭДС), индуцированная движением относительно магнитного поля B.

Цели обучения

Сформулируйте две точки зрения, которые применяются для расчета электродвижущей силы

Основные выводы

Ключевые моменты
  • Движущаяся и наведенная ЭДС — одно и то же явление, только наблюдаемое в разных системах отсчета. Эквивалентность этих двух явлений подтолкнула Эйнштейна к работе над специальной теорией относительности.
  • ЭДС, возникающая из-за относительного движения петли и магнита, определяется как [latex] \ varepsilon _ {\ text {motion}} = \ text {vB} \ times \ text {L} [/ latex] (Eq.1), где L — длина объекта, движущегося со скоростью v относительно магнита.
  • ЭДС можно рассчитать с двух разных точек зрения: 1) с точки зрения магнитной силы, действующей на движущиеся электроны в магнитном поле, и 2) с точки зрения скорости изменения магнитного потока. Оба дают одинаковый результат.
Ключевые термины
  • специальная теория относительности : теория, которая (игнорируя эффекты гравитации) согласовывает принцип относительности с наблюдением, что скорость света постоянна во всех системах отсчета.
  • магнитное поле : Состояние в пространстве вокруг магнита или электрического тока, в котором существует обнаруживаемая магнитная сила и где присутствуют два магнитных полюса.
  • рамка отсчета : система координат или набор осей, в пределах которых можно измерить положение, ориентацию и другие свойства объектов в ней.

Электродвижущая сила (ЭДС), индуцированная движением относительно магнитного поля B, называется ЭДС движения. Вы могли заметить, что ЭДС движения очень похожа на ЭДС, вызванную изменением магнитного поля.В этом атоме мы видим, что это действительно одно и то же явление, показанное в разных системах отсчета.

Движение ЭДС

В случае, когда проводящая петля перемещается в магнит, показанный на (а), магнитная сила, действующая на движущийся заряд в петле, определяется как [латекс] evB [/ латекс] (сила Лоренца, e: заряд электрона).

Петля проводника, движущаяся в магнит : (а) ЭДС движения. Токовая петля переходит в неподвижный магнит. Направление магнитного поля внутрь экрана.(б) Индуцированная ЭДС. Токовая петля неподвижна, а магнит движется.

Из-за силы электроны будут продолжать накапливаться с одной стороны (нижний конец на рисунке), пока на стержне не установится достаточное электрическое поле, препятствующее движению электронов, то есть [латекс] \ text {eE} [/ латекс]. Приравнивая две силы, получаем [латекс] \ text {E} = \ text {vB} [/ latex].

Следовательно, двигательная ЭДС на длине L стороны петли определяется как [latex] \ varepsilon _ {\ text {motion}} = \ text {vB} \ times \ text {L} [/ latex] (Eq .1), где L — длина объекта, движущегося со скоростью v относительно магнита.

Индуцированная ЭДС

Поскольку скорость изменения магнитного потока, проходящего через петлю, равна [latex] \ text {B} \ frac {\ text {dA}} {\ text {dt}} [/ latex] (A: площадь петли что магнитное поле проходит), индуцированная ЭДС [латекс] \ varepsilon _ {\ text {индуцированный}} = \ text {BLv} [/ latex] (уравнение 2).

Эквивалентность движущей и индуцированной ЭДС

Из уравнения. 1 и уравнение. 2 мы можем подтвердить, что двигательная и индуцированная ЭДС дают одинаковый результат.Фактически, эквивалентность двух явлений побудила Альберта Эйнштейна исследовать специальную теорию относительности. В своей основополагающей статье по специальной теории относительности, опубликованной в 1905 году, Эйнштейн начинает с упоминания эквивалентности двух явлений:

«…… например, взаимное электродинамическое действие магнита и проводника. Наблюдаемое явление здесь зависит только от относительного движения проводника и магнита, в то время как обычный взгляд проводит резкое различие между двумя случаями, когда одно или другое из этих тел находится в движении.Ведь если магнит находится в движении, а проводник находится в покое, в окрестности магнита возникает электрическое поле с определенной энергией , производящее ток в местах, где части проводника находятся расположенный. Но если магнит неподвижен, а проводник находится в движении, электрическое поле поблизости от магнита не возникает. В проводнике, однако, мы находим электродвижущую силу, которой сама по себе не соответствует энергия, но которая порождает — при условии равенства относительного движения в двух рассмотренных случаях — электрические токи того же пути и силы, что и создаваемые электрическими силами в первом случае.«

Механические работы и электроэнергия

Механическая работа, совершаемая внешней силой для создания ЭДС движения, преобразуется в тепловую энергию; энергия сохраняется в процессе.

Цели обучения

Применить закон сохранения энергии для описания производственной двигательной электродвижущей силы с механической работой

Основные выводы

Ключевые моменты
  • ЭДС движения, создаваемая движущимся проводником в однородном поле, имеет следующий вид [latex] \ varepsilon = \ text {Blv} [/ latex].
  • Чтобы стержень двигался с постоянной скоростью v, мы должны постоянно прикладывать внешнюю силу F ext к стержню во время его движения.
  • Закон Ленца гарантирует, что движение стержня противоположно, и, следовательно, закон сохранения энергии не нарушается.
Ключевые термины
  • ЭДС движения : ЭДС (электродвижущая сила), индуцированная движением относительно магнитного поля.
  • Закон индукции Фарадея : Основной закон электромагнетизма, который предсказывает, как магнитное поле будет взаимодействовать с электрической цепью, создавая электродвижущую силу (ЭДС).

Мы узнали о двигательной ЭДС ранее (см. Наш Атом в «Двигательной ЭДС»). Для простой схемы, показанной ниже, движущаяся ЭДС [латекс] (\ varepsilon) [/ латекс], создаваемая движущимся проводником (в однородном поле), задается следующим образом:

[латекс] \ varepsilon = \ text {Blv} [/ латекс]

, где B — магнитное поле, l — длина проводящего стержня, а v — (постоянная) скорость его движения. ( B , l и v все перпендикулярны друг другу, как показано на изображении ниже.)

ЭДС движения : (a) ЭДС движения = Bℓv индуцируется между рельсами, когда этот стержень перемещается вправо в однородном магнитном поле. Магнитное поле B направлено внутрь страницы, перпендикулярно движущемуся стержню и рельсам и, следовательно, к области, окружающей их. (б) Закон Ленца дает направление индуцированного поля и тока, а также полярность наведенной ЭДС. Поскольку поток увеличивается, индуцированное поле направлено в противоположном направлении или за пределы страницы. Правило правой руки дает указанное направление тока, и полярность стержня будет управлять таким током.

Сохранение энергии

В этом атоме мы рассмотрим систему с точки зрения энергии . Поскольку стержень движется и пропускает ток и , он ощущает силу Лоренца

.

[латекс] \ text {F} _ \ text {L} = \ text {iBL} [/ latex].

Чтобы стержень двигался с постоянной скоростью v , мы должны постоянно прикладывать внешнюю силу F ext (равную величине F L и противоположную по направлению) к стержню вдоль его движения. .Поскольку стержень движется со скоростью v , мощность P , передаваемая внешней силой, будет:

[латекс] \ text {P} = \ text {F} _ {\ text {ext}} \ text {v} = (\ text {iBL}) \ times \ text {v} = \ text {i} \ варепсилон [/ латекс].

На последнем этапе мы использовали первое уравнение, о котором говорили. Обратите внимание, что это в точности мощность, рассеиваемая в контуре (= ток [латекс] \ умноженное на [/ латекс] напряжение). Таким образом, мы заключаем, что механическая работа, совершаемая внешней силой, чтобы стержень двигался с постоянной скоростью, преобразуется в тепловую энергию в контуре.В более общем смысле, механическая работа, совершаемая внешней силой для создания ЭДС движения, преобразуется в тепловую энергию. Энергия сохраняется в процессе.

Закон Ленца

Из «Закона индукции Фарадея и закона Ленца» мы узнали, что закон Ленца является проявлением сохранения энергии. Как мы видим в примере с этим атомом, закон Ленца гарантирует, что движение стержня противодействует из-за склонности природы противодействовать изменению магнитного поля. Если бы индуцированная ЭДС была в том же направлении, что и изменение потока, возникла бы положительная обратная связь, заставляющая стержень улетать от малейшего возмущения.

Энергия в магнитном поле

Магнитное поле накапливает энергию. Плотность энергии задается как [латекс] \ text {u} = \ frac {\ mathbf {\ text {B}} \ cdot \ mathbf {\ text {B}}} {2 \ mu} [/ latex].

Цели обучения

Выразите плотность энергии магнитного поля в форме уравнения

Основные выводы

Ключевые моменты
  • Энергия необходима для создания магнитного поля как для работы против электрического поля, создаваемого изменяющимся магнитным полем, так и для изменения намагниченности любого материала в магнитном поле.2 [/ латекс].
Ключевые термины
  • проницаемость : Количественная мера степени намагничивания материала в присутствии приложенного магнитного поля (измеряется в ньютонах на ампер в квадрате в единицах СИ).
  • индуктор : Пассивное устройство, которое вводит индуктивность в электрическую цепь.
  • ферромагнетик : Материалы, обладающие постоянными магнитными свойствами.

Энергия необходима для создания магнитного поля как для работы против электрического поля, создаваемого изменяющимся магнитным полем, так и для изменения намагниченности любого материала в магнитном поле.Для недисперсионных материалов эта же энергия высвобождается при разрушении магнитного поля. Следовательно, эту энергию можно смоделировать как «хранящуюся» в магнитном поле.

Магнитное поле, создаваемое соленоидом : Магнитное поле, создаваемое соленоидом (вид в разрезе), описанное с использованием силовых линий. Энергия «хранится» в магнитном поле.

Энергия, запасенная в магнитном поле

Для линейных недисперсных материалов (таких, что B = мкм, H, где мкм, называемая проницаемостью, не зависит от частоты), плотность энергии составляет:

[латекс] \ text {u} = \ frac {\ mathbf {\ text {B}} \ cdot \ mathbf {\ text {B}}} {2 \ mu} = \ frac {\ mu \ mathbf {\ text {H}} \ cdot \ mathbf {\ text {H}}} {2} [/ latex].

Плотность энергии — это количество энергии, хранящейся в данной системе или области пространства на единицу объема. Если поблизости нет магнитных материалов, мкм можно заменить на мкм 0 . Однако приведенное выше уравнение нельзя использовать для нелинейных материалов; необходимо использовать более общее выражение (приведенное ниже).

В общем, дополнительная работа на единицу объема δW , необходимая для того, чтобы вызвать небольшое изменение магнитного поля δ B, составляет:

[латекс] \ delta \ text {W} = \ mathbf {\ text {H}} \ cdot \ delta \ mathbf {\ text {B}} [/ latex].

Когда связь между H и B известна, это уравнение используется для определения работы, необходимой для достижения заданного магнитного состояния. Для гистерезисных материалов, таких как ферромагнетики и сверхпроводники, необходимая работа также зависит от того, как создается магнитное поле. Однако для линейных недисперсионных материалов общее уравнение приводит непосредственно к более простому уравнению плотности энергии, приведенному выше.

Энергия, запасенная в поле соленоида

Энергия, запасаемая индуктором, равна количеству работы, необходимой для установления тока через индуктор и, следовательно, магнитного поля.2 [/ латекс].

Трансформаторы

Трансформаторы преобразуют напряжения из одного значения в другое; его функция определяется уравнением трансформатора.

Цели обучения

Примените уравнение трансформатора для сравнения вторичного и первичного напряжений

Основные выводы

Ключевые моменты
  • Трансформаторы часто используются в нескольких точках систем распределения электроэнергии, а также во многих бытовых адаптерах питания.
  • Уравнение трансформатора
  • гласит, что отношение вторичного напряжения к первичному в трансформаторе равно отношению количества витков в их катушках: [латекс] \ frac {\ text {V} _ \ text {s}} {\ text { V} _ \ text {p}} = \ frac {\ text {N} _ \ text {s}} {\ text {N} _ \ text {p}} [/ latex].
  • Если предположить, что сопротивление незначительно, выходная электрическая мощность трансформатора равна его входной. Это приводит нас к другому полезному вопросу: [latex] \ frac {\ text {I} _ \ text {s}} {\ text {I} _ \ text {p}} = \ frac {\ text {N} _ \ текст {p}} {\ text {N} _ \ text {s}} [/ latex]. Если напряжение увеличивается, ток уменьшается. И наоборот, если напряжение уменьшается, ток увеличивается.
Ключевые термины
  • магнитный поток : мера силы магнитного поля в заданной области.
  • Закон индукции Фарадея : Основной закон электромагнетизма, который предсказывает, как магнитное поле будет взаимодействовать с электрической цепью, создавая электродвижущую силу (ЭДС).

Трансформаторы изменяют напряжение с одного значения на другое. Например, такие устройства, как сотовые телефоны, ноутбуки, видеоигры, электроинструменты и небольшая бытовая техника, имеют трансформатор (встроенный в их съемный блок), который преобразует 120 В в напряжение, соответствующее устройству.Трансформаторы также используются в нескольких точках в системах распределения электроэнергии, как показано на рисунке. Мощность передается на большие расстояния при высоком напряжении, поскольку для данного количества мощности требуется меньший ток (это означает меньшие потери в линии). Поскольку высокое напряжение представляет большую опасность, трансформаторы используются для получения более низкого напряжения в месте нахождения пользователя.

Настройка трансформатора : Трансформаторы изменяют напряжение в нескольких точках системы распределения электроэнергии. Электроэнергия обычно вырабатывается при напряжении более 10 кВ и передается на большие расстояния при напряжениях более 200 кВ, иногда даже до 700 кВ, для ограничения потерь энергии.Местное распределение электроэнергии по районам или промышленным предприятиям проходит через подстанцию ​​и передается на короткие расстояния с напряжением от 5 до 13 кВ. Оно снижено до 120, 240 или 480 В для безопасности на месте отдельного пользователя.

Тип трансформатора, рассматриваемого здесь, основан на законе индукции Фарадея и очень похож по конструкции на устройство, которое Фарадей использовал для демонстрации того, что магнитные поля могут создавать токи (показано на рисунке). Две катушки называются первичной и вторичной катушками.При нормальном использовании входное напряжение подается на первичную обмотку, а вторичная обмотка создает преобразованное выходное напряжение. Мало того, что железный сердечник улавливает магнитное поле, создаваемое первичной катушкой, его намагниченность увеличивает напряженность поля. Поскольку входное напряжение переменного тока, изменяющийся во времени магнитный поток направляется во вторичную обмотку, вызывая ее выходное переменное напряжение.

Простой трансформатор : Типичная конструкция простого трансформатора имеет две катушки, намотанные на ферромагнитный сердечник, ламинированный для минимизации вихревых токов.Магнитное поле, создаваемое первичной обмоткой, в основном ограничивается и увеличивается сердечником, который передает его вторичной обмотке. Любое изменение тока в первичной обмотке вызывает ток во вторичной обмотке. На рисунке показан простой трансформатор с двумя катушками, намотанными с обеих сторон многослойного ферромагнитного сердечника. Набор катушек на левой стороне сердечника обозначен как первичный, и его номер указан как N p. Напряжение на первичной обмотке равно V p. Набор катушек на правой стороне сердечника обозначен как вторичный, и его номер представлен как N s.Напряжение на вторичной обмотке равно В с. Символ трансформатора также показан под диаграммой. Он состоит из двух катушек индуктивности, разделенных двумя равными параллельными линиями, представляющими сердечник.

Уравнение трансформатора

Для простого трансформатора, показанного на, выходное напряжение V s почти полностью зависит от входного напряжения V p и соотношения количества петель в первичной и вторичной катушках. Закон индукции Фарадея для вторичной обмотки дает ее индуцированное выходное напряжение V с как:

[латекс] \ text {V} _ \ text {s} = — \ text {N} _ \ text {s} \ frac {\ Delta \ Phi} {\ Delta \ text {t}} [/ latex],

, где N s — количество витков вторичной катушки, а Δ / Δt — скорость изменения магнитного потока.Обратите внимание, что выходное напряжение равно индуцированной ЭДС (В с = ЭДС с ), при условии, что сопротивление катушки невелико. Площадь поперечного сечения катушек одинакова с обеих сторон, как и напряженность магнитного поля, поэтому / Δt одинаково с обеих сторон. Входное первичное напряжение V p также связано с изменением магнитного потока соотношением:

[латекс] \ text {V} _ \ text {p} = — \ text {N} _ \ text {p} \ frac {\ Delta \ Phi} {\ Delta \ text {t}} [/ latex].

Соотношение этих двух последних уравнений дает полезное соотношение:

[латекс] \ frac {\ text {V} _ \ text {s}} {\ text {V} _ \ text {p}} = \ frac {\ text {N} _ \ text {s}} {\ текст {N} _ \ text {p}} [/ latex].

Это известно как уравнение для трансформатора , которое просто устанавливает, что отношение вторичного напряжения к первичному в трансформаторе равно отношению количества контуров в их катушках. Выходное напряжение трансформатора может быть меньше, больше или равно входному напряжению, в зависимости от соотношения количества витков в их катушках. Некоторые трансформаторы даже обеспечивают переменный выход, позволяя выполнять подключение в разных точках вторичной обмотки.Повышающий трансформатор — это трансформатор, который увеличивает напряжение, тогда как понижающий трансформатор снижает напряжение.

Если предположить, что сопротивление незначительно, выходная электрическая мощность трансформатора равна его входной. Уравнивание входной и выходной мощности,

[латекс] \ text {P} _ \ text {p} = \ text {I} _ \ text {p} \ text {V} _ \ text {p} = \ text {I} _ \ text {s} \ text {V} _ \ text {s} = \ text {P} _ \ text {s} [/ latex].

Комбинируя эти результаты с уравнением трансформатора, находим:

[латекс] \ frac {\ text {I} _ \ text {s}} {\ text {I} _ \ text {p}} = \ frac {\ text {N} _ \ text {p}} {\ текст {N} _ \ text {s}} [/ latex].

Значит, если напряжение увеличивается, ток уменьшается. И наоборот, если напряжение уменьшается, ток увеличивается.

Силовые линии Фарадея и теория электромагнитного поля Максвелла

Примечание редактора: В честь 150-летия создания уравнений Максвелла для нас большая честь представить вам главу из книги Жизнь Джеймса Клерка Максвелла , оригинальная биография 1882 года Джеймса Клерка Максвелла. Эта глава, озаглавленная «Силовые линии Фарадея и теория электромагнитного поля Максвелла», дает уникальное понимание теории электромагнитных полей Максвелла.

Эту статью покойный королевский астроном охарактеризовал как «одно из самых замечательных приложений математики к физике, которые я когда-либо видел».

Но, несмотря на упомянутые выше исследования и множество других оригинальных работ почти по всем отраслям физической науки, именно благодаря своим исследованиям в области электричества и молекулярных наук Максвелл занимает выдающееся место среди ученых нынешнего столетия. После получения степени в 1854 году Максвелл прочитал «Экспериментальные исследования» Фарадея, курс, которым он всегда рекомендовал своим студентам следовать.У Фарадея он обнаружил ум, по сути, своего типа. Тщательно знаком с теорией притяжения, разработанной в «Математических трактатах», и с законами электрического действия, проиллюстрированными сэром Уильямом Томсоном в его статье «Равномерное движение тепла в однородных твердых телах и его связь с математической теорией Электричество »в статье, опубликованной в Cambridge Mathematical Journal в феврале 1842 г., и« О механическом представлении электрических, магнитных и гальванических сил », опубликованной в Cambridge and Dublin Mathematical Journal в январе 1847 г., Максвелл увидел связь между точкой зрения Фарадея. взгляд и метод исследования, принятый математиками.Он имел обыкновение говорить, что у него не был хороший нюх, чтобы нюхать ересь, но все хорошее и правдивое Максвелл обнаружил под массой заблуждений или даже лжи, которые собрались вокруг него и которые вызывали его отторжение почти всеми остальными. без запроса. Фарадеевскую концепцию среды он использовал в качестве руководства в своих электрических исследованиях.

До шестнадцатого века все, что было известно об электричестве, было тем фактом, что янтарь при трении обладает способностью притягивать световые тела.Доктором Гилбертом из Колчестера, который был врачом королевы Елизаветы и который может считаться основателем науки об электричестве, было показано (Physiologia Nova, 1600), что они обладают различными веществами. С этого времени в экспериментальной части науки был достигнут быстрый прогресс, и закон, согласно которому притяжение или отталкивание между двумя маленькими телами, заряженными электричеством, изменяется в зависимости от зарядов, а расстояние между ними, был определен Кулоном с его торсионом. баланс, инструмент, ценность которого для экспериментатора трудно переоценить.Но именно Кавендишу (1771-1781) мы в основном обязаны основанием математической теории электричества и высочайшим экспериментальным доказательством закона электрического действия. Поскольку подготовка к печати книги «Электрические исследования достопочтенного Генри Кавендиша» была последним вкладом Максвелла в науку, работа была опубликована всего за несколько недель до его смерти, мы снова должны будем сослаться на исследования Кавендиша, и нам нужно только указать что его эксперименты убедительно доказали и наилучшим образом, насколько позволяли имеющиеся в его распоряжении инструменты, что притяжение или отталкивание между двумя маленькими заряженными телами изменяется прямо как произведение их зарядов и обратно пропорционально квадрату расстояния между ними, так что закон электрического действия такой же, как закон тяготения Ньютона, за исключением того, что напряжение между одинаково заряженными телами является отталкивающим, а напряжение между разноименно заряженными телами — притягивающим.После времен Кавендиша сравнительно немногое было добавлено к теории статического электричества, если не считать сложных математических исследований отдельных проблем Пуассоном и работ Джорджа Грина, которые до недавнего времени читались немногими и оценивались лишь двумя или тремя людьми. пока Фарадей не занялся этой темой. Большая часть работ Кавендиша оставалась неопубликованной и неизвестной, а некоторые из его результатов были независимо получены Фарадеем. Трудно представить, как бы повлиял на разум Фарадея «мысли Кавендиша об электричестве», а также его собственные отчеты о своих экспериментах.Возможно, для мира будет лучше, если Фарадею позволят работать и думать независимо; Безусловно, как для математиков, так и для физиков было благом, что Максвелл, казалось, разъяснял и развивал, если не совершенствовал, работу обоих.

Математическая теория притяжения до времен Фарадея достигла очень высокой степени развития в руках Лапласа, Лагранжа, Пуассона и других и могла быть применена к решению многих очень интересных задач электричества.Но Фарадея не удовлетворила гипотеза о прямом действии на расстоянии между зарядами электричества, и он считал, что должен существовать какой-то механизм, с помощью которого электрические и электромагнитные воздействия могут передаваться от точки к точке. Не все аргументы, которыми он поддерживал эту точку зрения, являются убедительными, поскольку сила, действующая на наэлектризованное тело, и индуцированная электризация любого проводника будут одинаковыми, независимо от того, принимаем ли мы гипотезу прямого воздействия на расстоянии или передачи электрического воздействия в пространстве. линии, прямые или изогнутые, через промежуточную среду.Но любая точка зрения, независимо от того, являются ли аргументы в ее пользу убедительными или нет, имеет ценность, если она побуждает нас более внимательно исследовать механизм, с помощью которого возникает явление; и, таким образом, концепция Фарадея о силовых линиях, передаваемых через среду и оказывающих напряжение и давление везде, где они могут быть найдены, имеет большую ценность как инструмент интеллектуального исследования, чем теория электромагнетизма Вебера, какой бы совершенной она ни была. с математической точки зрения.

Следующая цитата из предисловия к книге «Электричество и магнетизм» описывает взгляды Максвелла на Фарадея его собственными словами (см. Также статью Максвелла «Фарадей» в Ency.Брит., 9 изд.):

Прежде чем я начал изучать электричество, я решил не читать математику по этому предмету, пока я не прочту «Экспериментальные исследования электричества» Фарадея. Я знал, что должна была существовать разница между подходом Фарадея к пониманию явлений и подходом математиков, так что ни он, ни они не были удовлетворены языком друг друга. Я также был убежден, что это расхождение не возникло из-за того, что одна из сторон ошибалась. Впервые в этом меня убедил сэр Уильям Томсон, советам и помощи которого, а также его опубликованным статьям я обязан большей частью того, что узнал по этому вопросу.

Продолжая изучение Фарадея, я понял, что его метод представления явлений также является математическим, хотя и не представлен в традиционной форме математических символов. Я также обнаружил, что эти методы могут быть выражены в обычных математических формах, и их можно сравнить с методами математиков.

Например, Фарадей мысленным взором видел силовые линии, пересекающие все пространство, а математики видели центры силы, притягивающиеся на расстоянии; Фарадей видел медиума, в котором они не видели ничего, кроме расстояния; Фарадей искал причину явлений в реальных действиях, происходящих в среде, они были удовлетворены тем, что нашли его в силе действия на расстоянии, воздействующем на электрические жидкости.

Предположим, что небольшое положительно наэлектризованное тело начинается с точки, близкой к положительно наэлектризованной поверхности, и предположим, что оно всегда движется в том направлении, в котором его подталкивает действующая на него сила, оно, конечно, будет отталкиваться поверхность, и будет двигаться по некоторому прямому или изогнутому пути и будет продолжать движение бесконечно, сила уменьшаться по мере продвижения, если только он не встретится с отрицательно наэлектризованной поверхностью, которая будет притягивать его, и вступит в контакт с этой поверхностью. прекратится.Путь, проложенный таким маленьким наэлектризованным телом, составляет силовую линию Фарадея, которая, следовательно, является линией, направление которой в любой точке совпадает с направлением равнодействующей силы в этой точке. Такие силовые линии всегда исходят от положительно наэлектризованных поверхностей и заканчиваются отрицательно наэлектризованными поверхностями; или, в противном случае, они должны идти до бесконечности. Силовые линии, исходящие от положительно наэлектризованного тела, помещенного в комнату, если поблизости нет других отрицательно заряженных тел, обычно заканчиваются на стенах, полу и потолке комнаты или на объектах в комнате при электрической связи. с этими.Таким образом, Фарадей задумал все пространство, в котором действует электрическая сила, должно проходить через силовые линии, указывающие в каждой точке направление результирующей силы в этой точке. Но Фарадей пошел дальше этого: он задумал заставить силовые линии также представлять интенсивность силы в каждой точке, так что, когда сила велика, линии могли быть близко друг к другу и далеко друг от друга, когда сила небольшой; и поскольку сила в окрестности небольшого заряженного тела пропорциональна заряду, он попытался достичь этой цели, вычерчивая из каждой положительно наэлектризованной поверхности ряд силовых линий, пропорциональных ее заряду, и создавая такое же количество линий сила оканчивается на каждой отрицательно наэлектризованной поверхности.В статье под названием «О силовых линиях Фарадея», прочитанной в Кембриджском философском обществе 10 декабря 1855 г. и 11 февраля 1856 г., Максвелл показал, что если система линий может быть проведена в соответствии с методом Фарадея, то в силу закона электрического действия, равного обратному квадрату расстояния, количество силовых линий, проходящих через единицу площади любой поверхности, проведенных перпендикулярно направлению силы, пропорционально величине силы в окрестности, и что количество линий, проходящих через единицу площади любой другой поверхности, пропорционально компоненту силы, перпендикулярной этой поверхности.Максвелл поэтому представил положительно наэлектризованные поверхности, от которых линии начали разделяться на области, каждая из которых содержит одну единицу электричества, и силовые линии, проводимые через каждую точку каждой ограничивающей линии. Эти линии, таким образом, делят все пространство на «единичные трубки», границы которых являются силовыми линиями, и Максвелл показал, что в силу «закона обратных квадратов» сила в любой точке в любом направлении обратно пропорциональна силе. площадь сечения единичной силовой трубки плоскостью, перпендикулярной этому направлению.Максвелл далее показал, что на отрицательно наэлектризованной поверхности, на которой заканчиваются эти трубки, каждая трубка будет заключать в себе одну единицу отрицательного электричества, и, следовательно, если металлическая поверхность будет введена так, чтобы разрезать силовые линии, поверхность будет помещена справа. под углом к ​​трубке, единица отрицательного электричества будет индуцироваться на каждой части поверхности, содержащейся в следе силовой трубки; и, следовательно, в любой изотропной среде эти единичные силовые трубки также являются единичными индукционными трубками.Следовательно, если система силовых трубок будет соединена с какой-либо электрифицированной системой и в соответствии с этим планом, все пространство, в котором действует сила, будет разделено на трубки, каждая из которых происходит от единицы положительного электричества и заканчивается на блоке отрицательного электричества, тогда как направление силы в любой точке будет указываться направлением силы трубки, а величина силы будет обратно пропорциональна площади поперечного сечения трубки.Теперь, если бы закон силы был любым, кроме закона обратного квадрата, и трубы были бы нарисованы, начиная с наэлектризованной поверхности, как указано выше, и так, что площадь любого участка трубы обратно пропорциональна силе, действующей через сечения эти трубы либо оставляли бы промежутки между собой, когда они удалялись от поверхности, либо пересекали бы друг друга; так что только для закона обратных квадратов возможна описанная выше система трубок. Фарадей указал, что существует не только напряжение, прилагаемое вдоль каждой силовой линии, но что несколько линий оказывают отталкивание друг к другу, и Максвелл показал, что напряжение вдоль силовых линий сопровождается одинаковым давлением во всех направлениях на под прямым углом к ​​этим линиям, соответствует равновесию среды.Взяв иллюстрацию потока воды в реке, Максвелл указал, что линии потока или пути, по которым текут частицы воды, аналогичны линиям электрической силы, а скорость воды аналогична силе силы. Если предположить, что река разделена на трубы, границы которых представляют собой линии потока, и если эти трубы нарисованы так, что единичный объем воды проходит через определенный участок каждой трубы за секунду, тогда, если поток будет При устойчивом режиме единичный объем воды будет течь через каждую секцию каждой трубки за секунду, поскольку вода не входит и не выходит из трубки, кроме как на ее концах.Такие трубы можно назвать единичными трубками потока, и, если в реку не входят притоки, будет одинаковое количество единичных трубок, пересекающих каждый участок реки. Когда русло расширяется, сечение каждой трубы увеличивается, всегда обратно пропорционально скорости воды, и, следовательно, количество единичных трубок потока, которые пересекают любую единицу площади в поперечном сечении реки, будет пропорционально скорости воды по соседству. Таким образом, такая система трубок будет представлять как направление движения, так и скорость воды в каждой точке и будет точно соответствовать, mutatis mutandis, системе единичных трубок электрической силы.

Следующее письмо было адресовано Максвеллу Фарадеем после получения копии статьи о «Линии силы»:

Albemarle Street, W.,
25 марта 1857 г.
МОЙ ДОРОГОЙ СЕР Я получил вашу газету и большое вам спасибо за нее. Я не говорю, что осмеливаюсь поблагодарить вас за то, что вы сказали о «Линиях силы», потому что я знаю, что вы сделали это в интересах философской истины; но вы должны предположить, что это благодарная мне работа, которая дает мне много сил подумать.Сначала я был почти напуган, когда увидел такую ​​математическую силу, воздействующую на этот предмет, а затем удивился, увидев, что испытуемый так хорошо выдержал это. Посылаю вам этим письмом еще одну бумагу; Интересно, что вы на это скажете. Я надеюсь, однако, какими бы смелыми ни были мысли, вы, возможно, найдете причину смириться с ними. Я надеюсь этим летом провести несколько экспериментов со временем магнитного воздействия, или, скорее, со временем, необходимым для принятия электротонического состояния вокруг провода, по которому проходит ток, что может помочь этому предмету.Вероятно, время должно быть коротким, как время света; но величие результата, если оно положительное, не заставляет меня отчаиваться. Возможно, мне лучше было ничего об этом не говорить, потому что я часто долго не осознаю свои намерения, и неудачное воспоминание против меня.
Ваш самый искренний,
М. ФАРАДЕЙ. Проф. К. Максвелл.

Статья, прочитанная в Кембриджском философском обществе и опубликованная в т. Икс. их трудов, по общему признанию, является лишь переводом идей Фарадея на математический язык с иллюстрациями и расширениями, и в нем не делается попыток объяснить природу действия в диэлектрике или механизм, с помощью которого возникают наблюдаемые эффекты.Примерно пять лет спустя, в серии из трех статей, опубликованных в Philosophical Magazine в 1861 и 1862 годах, профессор Максвелл дал простой набросок системы механизма, способного вызывать не только упомянутые выше электростатические эффекты, но и объяснять их. магнитное притяжение, действие электрических токов друг на друга и на магниты и электромагнитная индукция; но прежде чем дать отчет об этих работах, необходимо будет кратко упомянуть основные явления, объяснение которых требовалось.

Обычные явления магнетизма, включая притяжение между несхожими полюсами и отталкивание между подобными полюсами, а также еще более известные явления притяжения мягкого железа магнитным полюсом слишком хорошо известны, чтобы требовать более чем беглого упоминания. Кулон показал, что закон обратных квадратов получается одинаково для магнитного отталкивания и для электрического, так что напряжение между двумя магнитными полюсами пропорционально произведению сил полюсов и обратно пропорционально квадрату расстояния между ними при условии, что Сталь, из которой состоят магниты, достаточно тверда, чтобы предотвратить воздействие магнитов друг на друга, изменяющее силу их полюсов.

Если лист бумаги опереться горизонтально над полюсами магнита и на бумагу посыпать железные опилки, каждая опилка намагнитится за счет индукции в направлении результирующей магнитной силы в той точке, где она расположена, и если по бумаге осторожно постучать, чтобы преодолеть трение, взаимное притяжение разноименных полюсов в опилках заставляет их склеиваться в нити или нити, северный полюс одной опалубки присоединяется к южному полюсу соседней опилки и т. д. все точки крепления лежат вдоль силовой линии.Таким образом, опилки образуют графическое изображение линий магнитной силы, и именно этот эксперимент впервые подсказал Фарадею идею физического существования таких линий; и поскольку ему было трудно представить себе изогнутые силовые линии, возникающие из-за «прямого действия на расстоянии» (Exp. Kes.1166), он считал, что должна существовать некоторая среда, которая является проводником как магнитных, так и электрических сил, и что такие силы распространяются от частицы к частице среды.Фарадей также предположил, что та же среда может служить средством передачи света. Исследование свойств среды, необходимых для объяснения наблюдаемых электрических и магнитных воздействий, объяснение этих воздействий и определение скорости света из чисто электромагнитных соображений на основе гипотезы о существовании такой среды составляет Величайший вклад Максвелла в науку об электричестве. Воздействие электрического тока на магнит впервые наблюдал Эрстед.Говорят, что он делал много попыток в своей лаборатории обнаружить действие между магнитом и проводом, передающим ток, но во всех своих попытках он осторожно размещал провод под прямым углом к ​​магнитной игле и не мог обнаружить никакого эффекта. При попытке повторить эксперимент в присутствии своего класса он поместил проволоку параллельно игле, и последняя немедленно повернулась и в конце концов остановилась почти под прямым углом к ​​проволоке. Всякий раз, когда Северный полюс (т.е.Северный полюс) магнита приближается к проводу, по которому течет ток, полюс стремится огибать провод в определенном направлении, в то время как южный (или южный) полюс магнита стремится огибать провод в определенном направлении. в противоположном направлении, и, следовательно, если магнит будет свободно вращаться вокруг своего центра, он остановится под прямым углом к ​​проводу. Было дано много memoriae technicce для определения того, как магнит будет вести себя по соседству с током. Правило Максвелла было следующим: предположим, что винт с правой резьбой продвигается в направлении тока и обязательно вращается по мере продвижения, как если бы он протыкал твердое тело.Северный полюс магнита всегда будет стремиться двигаться по проводу, передавая ток в том направлении, в котором вращается такой винт, в то время как южный полюс будет двигаться в противоположном направлении.

Таким образом, мы можем предположить, что каждый провод, передающий ток, окружен магнитными силовыми линиями, которые образуют замкнутые кривые вокруг провода, а направление силы такое, в котором правосторонний винт будет вращаться, двигаясь вперед с током. В случае прямого провода бесконечной длины эти кривые, конечно, представляют собой окружности.Поскольку действие и противодействие равны и противоположны, отсюда следует, что какой бы ни была механическая сила, оказываемая током на полюс магнита, последний всегда будет оказывать равную и противоположную силу на провод или другой проводник, по которому проходит ток. Чтобы показать это, было проведено множество экспериментов. Максвелл очень просто иллюстрировал это. Прикрепив кусок изолированной медной проволоки к небольшой круглой медной пластине, он поместил пластину на дно небольшого стакана. Затем вырезали диск из листового цинка такого размера, чтобы он свободно помещался в химическом стакане, оставляя прикрепленным к нему небольшой «хвостик» цинка; он был изогнут и соединен с медной проволокой над верхней частью стакана, в то время как цинковая пластина была подвешена в горизонтальном положении на дюйм или два над медной пластиной.Стакан наполняли разбавленной серной кислотой и помещали на один полюс электромагнита, опилки или порошкообразную смолу помещали в жидкость, чтобы показать ее движение. При возбуждении магнита жидкость вращалась в одном направлении, а при изменении полярности магнита направление вращения менялось. Если пластины подвешены на веревке так, чтобы они могли легко вращаться в стакане вокруг вертикальной оси, действие магнита на ток в вертикальном проводе заставит пластины всегда поворачиваться в направлении, противоположном направлению вращения. жидкость.

Законы механического действия проводников, передающих токи, на магниты и друг на друга были исследованы Ампером в серии экспериментов, которые были одновременно убедительными и исчерпывающими. Профессор Максвелл в высшей степени упомянул об этих экспериментах. Любое их описание было бы здесь неуместным, и мы говорим о них только как о предоставлении экспериментальных доказательств для следующих утверждений.

Мы уже описали способ, которым магнитные силовые линии могут окружать провод, по которому проходит ток.Теперь позвольте такой проволоке согнуть в замкнутую кривую или кольцо, которое не обязательно должно быть круглым. Силовые линии, которые сами по себе образуют замкнутые кривые вокруг провода, будут проходить в одном и том же направлении через кольцо, образованное проводом, передающим ток, как если бы они были нанизаны на провод, и, следовательно, северный полюс магнита будет стремятся пройти сквозь кольцо в направлении силовых линий; и мгновенное размышление покажет, что это направление, в котором правый винт продвинется вперед, если вращаться в направлении тока в проводе.Следовательно, если северный полюс магнита поднести близко к такой маленькой замкнутой цепи, с одной стороны, он будет притягиваться и будет стремиться пройти через цепь; с другой стороны это будет отталкиваться. Южный полюс магнита будет действовать прямо противоположным образом. Следовательно, если небольшая магнитная игла подвешена внутри катушки с проводом, по которой проходит ток, она будет иметь тенденцию располагаться под прямым углом к ​​плоскости катушки. Такое устройство представляет собой гальванометр.

Теперь предположим, что у нас есть небольшой стальной диск того же размера и формы, что и кольцо, образованное проволокой, и что этот диск намагничен, так что одна сторона является северным полюсом, а другая — южным.Такой диск будет действовать на внешние магниты таким же образом, как и ток, если он намагничен, так что правосторонний винт, вращающийся с током, войдет с южной стороны и выйдет на северной стороне. Такой намагниченный диск называется магнитной оболочкой, и, конечно, на него будет воздействовать магнит с силами, точно равными и противоположными тем, с которыми он действует на магнит. Магнитные силовые линии, исходящие от контура, передающего электрический ток, поэтому такие же, как и исходящие от описанной выше магнитной оболочки, при этом сила намагничивания отрегулирована должным образом; Другими словами, магнитное поле вокруг такой цепи такое же, как и вокруг магнитной оболочки, и, следовательно, из этого следует, что две цепи, каждая из которых передает электрические токи, будут действовать друг на друга так же, как две магнитные оболочки, окружности которых совпадают. с проводами, которые намагничены, как описано выше.

Теперь, если оболочки параллельны и намагничены в одном направлении, их противоположные грани будут обращены друг к другу и будут притягиваться друг к другу. Если они намагничены в противоположных направлениях, они будут отталкиваться друг от друга. Точно так же две параллельные цепи будут притягиваться друг к другу, если токи проходят в одном направлении в обоих, и будут отталкивать друг друга, если они идут в противоположных направлениях. Также два параллельных провода, которые можно рассматривать как части таких цепей, будут притягивать друг друга, когда токи в них идут в одном направлении, и отталкивать друг друга, если они идут в противоположных направлениях.Правило Максвелла для определения того, как цепь, передающая ток, будет вести себя в присутствии других токов или магнитов, является очень простым выражением результатов Фарадея. Определив положительное направление цепи как направление, в котором правый винт будет двигаться, если вращаться с током, он сформулировал правило следующим образом:

Если провод, передающий ток, может свободно перемещаться в магнитном поле, он будет стремиться устроиться так, чтобы через цепь в положительном направлении могло проходить максимально возможное количество линий магнитной силы.

Поскольку магнитное поле может создаваться либо магнитами, либо самими электрическими токами, как описано выше, это правило в сочетании с принципом, согласно которому действие и противодействие равны и противоположны, будет служить для определения характера воздействия либо на цепи, передающие токи, либо на магниты во всех возможных случаях, которые могут возникнуть, и, по сути, воплощает великолепные результаты исследований Ампера в этом вопросе.

До опытов Фарадея индукция электрических токов была неизвестна.Основным явлением, зависящим от этого действия, которое наблюдалось и которому не было предложено удовлетворительного объяснения, было явление вращающегося диска Араго. В этом эксперименте медный диск заставляли быстро вращаться в своей собственной горизонтальной плоскости над стрелкой компаса, когда наблюдалось, что игла следует за диском и вращается на его вертикальном штифте. Этот эксперимент впоследствии повторили сэр Джон Гершель и мистер Бэббидж, которые использовали диски из различных веществ и обнаружили, что результат Араго был получен только тогда, когда диски были хорошими проводниками электричества.Фарадей в первой серии своих экспериментальных исследований описывает эксперимент, в котором медный диск заставляли вращаться между полюсами электромагнита, в то время как один электрод
гальванометра был соединен с осью диска, а электрод другой — проводом, который удерживался в контакте с краем диска, причем край был объединен для обеспечения хорошего соединения. При вращении диска сразу же получался ток, направление которого менялось на противоположное по направлению вращения.Этот эксперимент можно рассматривать как отправную точку для динамо-машин Уайльда, Грамма, Сименса и других, которым, кажется, суждено сыграть столь важную роль в цивилизованной жизни будущего.

Фарадей также показал, что, когда две цепи расположены рядом друг с другом, если в одной цепи запускается ток, в соседней цепи возникает мгновенный ток в противоположном направлении, тогда как при остановке «первичного» тока возникает переходный ток. то же направление, что и первичное, происходит в другом или «вторичном» контуре.Этот эксперимент положил начало хорошо известной теперь индукционной катушке. Опять же, когда ток в первичной цепи постоянно протекал, если вторичная цепь была приближена к ней, во вторичной цепи индуцировался ток в направлении, противоположном направлению в первичной цепи, и продолжался во время приближения цепей. При удалении вторичной цепи возник переходный ток в том же направлении, что и в первичной.

Здесь мы не можем уделить место, чтобы проследить развитие законов индуцированных токов.Во всех случаях характер действия может быть выведен из очень краткого заявления Ленца, обычно цитируемого как закон Ленца, и которое может быть выражено следующим образом:

Если проводник движется в магнитном поле, в проводнике будет индуцирована электродвижущая сила, которая будет стремиться произвести ток в таком направлении, что механическая сила, действующая на проводник, будет препятствовать его движению.

Этот закон, взятый вместе с утверждениями, сделанными выше относительно механического воздействия магнитного поля на проводник, передающий ток, служит для определения характера индуцированного тока всякий раз, когда проводник движется в окрестности магнитов или электрических токов.Более того, включение тока в соседней цепи должно иметь такое же воздействие на провод, как если бы проводник был внезапно перенесен с бесконечного расстояния в положение, которое он фактически занимает. Следовательно, закон Ленца будет применяться ко всем случаям наведенных токов.

Утверждение

Максвелла выражает законы индуцированных токов как количественно, так и качественно. Это выглядит следующим образом:

Всякий раз, когда количество силовых линий магнитного поля, проходящих через замкнутый контур, изменяется, вокруг контура возникает электродвижущая сила, представленная скоростью уменьшения количества силовых линий, проходящих через контур в положительном направлении.

Если тогда количество магнитных силовых линий, проходящих через цепь, уменьшается, вокруг цепи будет электродвижущая сила в том направлении, в котором правосторонний винт будет вращаться, если продвигаться вдоль силовых линий; предполагается, что силовая линия всегда проводится в направлении, в котором северный магнитный полюс имеет тенденцию двигаться вдоль нее. Если количество силовых линий, проходящих через цепь, увеличивается, электродвижущая сила будет в противоположном направлении.Этот закон может быть выведен из закона, который выражает механическое воздействие на цепь, передающую ток, когда она помещена в магнитное поле, вместе с принципом сохранения энергии. Чтобы это могло быть численно верным, все задействованные величины должны быть выражены в терминах электромагнитной системы единиц.

Телефон — прекрасный пример применения этого закона. Каждое движение железного диска перед полюсом магнита изменяет количество магнитных силовых линий, проходящих через катушки с проволокой, окружающей полюс, и, следовательно, индуцирует в катушке ток в том или ином направлении, который, увеличение или уменьшение силы магнетизма в приемном телефоне вызывает соответствующее движение в железном диске приемника, который, следовательно, издает звуки, аналогичные звукам, падающим на приемный инструмент.

Из того, что было сказано, очевидно, что движение проводника будет производить ток в нем только тогда, когда проводник движется в магнитном поле, то есть в части пространства, через которую проходят магнитные силовые линии. Фарадей предположил, что в этих условиях проводник был переведен в особое состояние, которое он назвал «электротоническим состоянием», и что ток индуцировался всякий раз, когда это состояние менялось. Максвелл показал, что это электротонное состояние, от изменений которого зависит индуцированный ток в цепи, соответствует количеству магнитных силовых линий, проходящих через цепь.Поскольку каждое изменение этой величины связано с действием электродвижущей силы, а ее отношение к электродвижущей силе такое же, как отношение количества движения к силе в динамике, он назвал эту величину электромагнитным импульсом. Представление Максвелла о физической природе этой величины будет описано ниже.

Определение законов самоиндукции в электрических токах — еще один из многих вкладов Фарадея в науку об электричестве. После одной из вечерних лекций в пятницу в Королевском институте некий мистер Билл.Дженкин сообщил Фарадею, что, когда он разорвал соединение цепи в своем электромагните, разделив два куска провода, которые он держал в руках, он почувствовал сильный шок. Фарадей сказал, что это единственное предложение из очень большого числа, сделанное ему обычными членами популярной аудитории, которое когда-либо приводило к какому-либо результату. Исследуя этот вопрос, Фарадей обнаружил, что, когда в катушке с проволокой течет ток, если аккумулятор удален, существует тенденция к продолжению тока после извлечения батареи, и что эта тенденция усиливается при увеличении количества витков проволоки в катушке, и тем более, вставив мягкое железо в центр катушки.Эта тенденция зависит не столько от длины проволоки, сколько от относительного положения ее частей, и если проволоку сначала сложить вдвое, а затем намотать в катушку, тенденция исчезнет. Если несколько ячеек Гроув пропускают ток через короткий прямой кусок провода, и цепь разрывается, при размыкании будет наблюдаться очень слабая искра, но если в цепь ввести большой электромагнит, при размыкании контакта появится очень яркая искра. , хотя ток, посылаемый батареей, слабее.Таким образом, когда в такой катушке течет ток, его поведение напоминает нам поведение воды, протекающей в трубе, которая, когда внезапно возникает препятствие, чтобы остановить поток, оказывает огромное давление на короткое время на трубу и препятствие. в силу количества движения, которое приобрела вода; но то, что действие происходит не из-за импульса, которым действительно обладает движущееся электричество, показывает тот факт, что он зависит от конфигурации провода.
Это свойство катушки называется самоиндукцией.Если полюса электромагнита соединить проводом с большим сопротивлением, а также батареей, при удалении батареи через провод будет протекать значительный ток. Этот ток Фарадей называл экстра-током. Обычно его называют током самоиндукции.

Аналогичное действие происходит при подключении батареи к катушке. Сила тока не сразу приобретает полное значение, но в течение короткого времени продолжает неуклонно возрастать; самоиндукция катушки, заставляющая ее вести себя так, как если бы ток в ней обладал значительной массой, которая в первую очередь должна быть приведена в движение.Все эти действия являются непосредственными следствиями закона индуцированных токов, изложенного на стр. 526. (введите правильную информацию)

Существует хорошо известный эксперимент Фарадея, в котором образец его тяжелого стекла, или бората свинца, помещался между полюсами мощного электромагнита, и луч плоскополяризованного света пропускался через стекло в направлении магнитной силы. Фарадей обнаружил, что когда свет проходил от северного полюса магнита к южному, плоскость поляризации поворачивалась на угол в том же направлении, что и правосторонний винт, проникая в твердое тело и продвигаясь со светом.Когда свет проходил в противоположном направлении, вращение плоскости поляризации происходило в том же направлении по отношению к магниту и, следовательно, было обратным по отношению к пути света. В этом отношении тяжелое стекло под воздействием магнита ведет себя иначе, чем раствор сахара, который всегда поворачивает плоскость поляризации света в том же направлении, что и направление его пропускания. Это был первый эксперимент, который показал какую-либо связь между светом и магнетизмом и показал, что среда, которая служит проводником света, светоносный эфир, должна, по крайней мере, зависеть от наличия магнитной силы, хотя тот факт, что присутствие весомой материи необходимо для создания этого вращения, и то, что направление вращения зависит от природы материи, делает сомнительным, насколько сильно магнитная сила влияет непосредственно на эфир.

Все прозрачные твердые тела и жидкости в разной степени проявляют одинаковое действие на свет. Если трубку с водой с концами из листового стекла поместить в катушку из проволоки, через которую проходит электрический ток, и через трубку будет проходить плоскополяризованный свет, плоскость поляризации будет повернута на угол в направлении, в котором ток циркулирует, и этот угол будет пропорционален току,

Верде показал, что в случае прозрачного (пара-) магнитного вещества вращение происходит в направлении, противоположном направлению тока.

Любопытное влияние магнита на световой разряд в вакуумной трубке и недавние эксперименты доктора Керра могут указывать на другие отношения между светом, электричеством и магнетизмом. Таким образом, очень кратко упомянув основные явления магнетизма и электромагнетизма, мы можем перейти к краткому объяснению среды или механизма, с помощью которого Максвелл объяснил эти явления и их взаимозависимость.

Из хорошо известных законов распространения света Максвелл принял «в качестве данных, полученных из области науки, не зависящей от того, с чем мы имеем дело, существование пронизывающей среды небольшой, но реальной плотности, способной к приводиться в движение и передавать движение от одной части к другой с большой, но не бесконечной скоростью.Поскольку эта среда может передавать волны с конечной скоростью, отсюда следует, что она обладает свойством, аналогичным массе, так что ее движение подразумевает кинетическую энергию; помимо упругости, в силу которой его деформация подразумевает потенциальную энергию.

Хорошо известно, что если тело вращается вокруг фиксированного центра, будет возникать натяжение по любому радиусу, проведенному в плоскости вращения. Форма, которую Земля приняла бы только под действием силы тяжести, если бы не было вращения, была бы сферой.Суточное вращение имеет тенденцию вызывать сокращение полярной оси и увеличение экваториального диаметра; и это действие продолжалось бы бесконечно, если бы на определенной ранней стадии оно не уравновешивалось притяжением гравитации, и, таким образом, Земля принимает почти сферическую форму, в которой полярная ось короче экваториального диаметра.

Возвращаясь снова к случаю Земли, из фундаментальных законов и принципов динамики видно, что если бы материя переносилась из экваториальных областей к полюсам и осаждалась там так, чтобы удлинить полярную ось за счет экваториальной диаметр, скорость вращения Земли увеличится, а продолжительность дня уменьшится; в то время как, если бы Земля стала более сжатой, ее скорость вращения уменьшилась бы.Фактически, если какое-либо тело находится во вращении и на него не действуют внешние силы, или если силы, действующие на него, не влияют на его вращение, и если форма системы изменяется под действием внутренних напряжений или иным образом, так что его момент инерции относительно оси вращения увеличивается, угловая скорость будет уменьшена, и, в случае превращения сферы в сплюснутый сфероид, скорость на окружности также будет уменьшена, а если момент инерции будет уменьшен, имеет место обратный эффект.

Максвелл предположил, что любая среда, которая может служить проводником магнитной силы, состоит из огромного числа очень маленьких тел или клеток, способных вращаться, и которые мы можем считать сферическими или почти такими в их нормальном состоянии, пока мы не есть основания полагать, что они имеют какую-то другую форму. Когда магнитная сила передается средой, предполагается, что эти тела вращаются вокруг силовых линий магнитного поля в качестве оси и со скоростью, зависящей от интенсивности силы.Чтобы закрепить наши идеи, он предположил, что вращение происходит в том направлении, в котором будет вращаться правосторонний винт, если он продвигается в направлении силы. Таким образом, мы имеем магнитное поле, заполненное «молекулярными вихрями», вращающимися в одном направлении вокруг магнитных силовых линий в качестве осей. Как мы видели, эти вихри будут стремиться сжиматься в направлении своих осей вращения и расширяться под прямым углом к ​​этому направлению, так что если изначально они представляют собой упругие сферы, они будут иметь тенденцию становиться сплюснутыми сфероидами, подобными Земле.Эта тенденция будет включать в себя напряжение в среде вдоль силовых линий, которые являются линиями, вдоль которых имеет место сжатие, и это будет сопровождаться одинаковым давлением во всех направлениях, перпендикулярных силовым линиям, из-за тенденция вихрей к экваториальному расширению.

Теперь предположим, что у нас есть северный магнитный полюс и южный магнитный полюс, расположенные рядом друг с другом. Силовые линии будут проходить от Северного полюса, обычно по изогнутым линиям, к Южному полюсу.Пространство в окрестностях полюсов будет заполнено молекулярными вихрями, которые будут наиболее энергичными вдоль линии, соединяющей полюсы, и скорости вихрей будут уменьшаться по мере того, как мы переходим в более слабые части поля. Напряжение вдоль силовых линий, стремящихся сблизить Северный и Южный полюса, дает достаточное объяснение очевидного притяжения между полюсами; кинетическая энергия молекулярных вихрей составляет потенциальную энергию разделенных полюсов, которую мы, таким образом, предполагаем на самом деле кинетической энергией, хотя и обладает средой между кажущимися притягивающими телами, а не самими телами.(Возможно, все примеры так называемой потенциальной энергии мы когда-нибудь обнаружим как действительно кинетическую энергию, которой обладает среда, свойства которой мы до сих пор не знали.) Когда полюса приближаются друг к другу, поле, которое занимает протяженность вихрей уменьшается, и хотя скорость вихрей увеличивается, вся энергия поля уменьшается, и разница расходуется на работу, совершаемую над приближающимися магнитами. Если полюса имеют одинаковую силу и могут полностью совпадать, поле разрушается, все вихри останавливаются, и вся энергия, которой они обладают, расходуется на работу, выполняемую с магнитами.

Если два одинаковых полюса, например, северных полюса, расположить рядом друг с другом, силовые линии, исходящие от одного, вместо того, чтобы идти к другому, будут повернуты в сторону, и если полюса будут равной силы, плоскость Разделение пополам под прямым углом линии, соединяющей полюса, отделит силовые линии, обусловленные одной, от линий, обусловленных другой, так что никакая линия не будет пересекать плоскость (рис. 10). Силовые линии, таким образом, проходят почти параллельно друг другу, давление, оказываемое молекулярными вихрями во всех направлениях под прямым углом к ​​силовым линиям, вызовет кажущееся отталкивание между полюсами.

Чтобы объяснить передачу вращения в одном и том же направлении от одного молекулярного вихря к другому, Максвелл предположил, что между ними существует ряд чрезвычайно мелких сферических тел, которые катятся без скольжения, контактируя с поверхностями вихрей. Эти тела служат той же цели, что и «холостые колеса» в машинах, которые, проходя между водителем и ведомым, передают движение первого второму в неизменном направлении. Эти крошечные сферические частицы Максвелл предположил, что они составляют электричество.Они катятся по ячейкам или вихрям, как если бы соприкасающиеся поверхности были совершенно шероховатыми или имели зубцы, зацепляющиеся друг с другом, и, таким образом, какие бы силы ни были приложены, скольжение невозможно. Предполагается, что то, что мы обычно считаем молекулами вещества, очень велико по сравнению с молекулярными вихрями и, следовательно, тем более с частицами электричества. В изоляторе или диэлектрике предполагается, что электрические частицы не могут переходить от молекулы к молекуле тела, но в проводнике они могут это сделать, однако проходу препятствует трение, так что выделяется тепло. и энергия, рассеиваемая при передаче.

Теперь предположим, что у нас есть электрический ток, протекающий по проводнику. Остановимся сначала на центральной линии частиц. Они, когда они текут, заставят все ячейки, которых они касаются, вращаться вокруг осей, перпендикулярных линии потока, так что поток частиц будет окружен кольцами вихрей. Каждое кольцо вихрей будет вести себя как кольцо зонта из натурального каучука, когда мы проводим им по пальцу или палке зонтика. Вместо того, чтобы скользить на свое место, оно совершает перекатывающее движение, непрерывно выворачивая себя наизнанку, как бы каждая круговая часть кольца или разрывается, вращаясь вокруг своего собственного центра.Теперь это движение вихрей будет иметь тенденцию заставлять слой электрических частиц за их пределами двигаться в направлении, противоположном центральному потоку, и эту тенденцию, на которую мы снова будем ссылаться, когда мы будем говорить об индукции, можно преодолеть, только заставив следующее кольцо ячеек должно вращаться в том же направлении, что и внутреннее кольцо, когда частицы могут просто катиться между коаксиальными кольцами вихрей, не перемещаясь ни назад, ни вперед. Но если слой частиц вынужден двигаться вперед, как внутренний поток, слой вихрей, окружающий его, должен вращаться быстрее, чем слой внутри него, и так далее, каждая последующая оболочка вихрей вращается быстрее, пока мы не достигнем крайнего слоя, содержащего внутри проводящего провода.Оболочка вихрей, ограничивающая проводник, должна с помощью того же механизма создавать молекулярные вихри в диэлектрике, причем движение передается в постоянно расширяющихся кругах на неограниченное расстояние. Из этого не следует, что это сообщение движения является мгновенным. Ячейки могут состоять из упругого материала, который не принимает своего конечного состояния движения, как только на него действует касательное действие электрических частиц, но сначала начинает претерпевать деформацию, время, необходимое для установления данного вращения в каждый в зависимости от его плотности и эластичности.Следовательно, электромагнитная индукция, так называется действие, которое мы сейчас обсуждаем, будет распространяться в пространстве с конечной скоростью, но об этом мы должны сказать больше в дальнейшем.

Из сказанного следует, что, когда в проводе течет постоянный (т. Е. Постоянный) ток, в окружающем диэлектрике возникают молекулярные вихри, причем ось вращения каждого вихря перпендикулярна плоскости, проходящей через провод. и вихрь. Оси, вокруг которых вращаются вихри, образуют круги, окружающие проволоку, в то время как сами вихри образуют вихревые кольца, вращающиеся с очень большой скоростью, как упомянутое выше внутреннее каучуковое кольцо, или кольца дыма, которые иногда можно увидеть. выйти из курительной трубки.Но линии, вокруг которых вращаются молекулярные вихри, являются магнитными силовыми линиями, поскольку вдоль этих линий в среде существует напряжение, а давление повсюду перпендикулярно им. Следовательно, прямая линия, по которой проходит электрический ток, будет окружена магнитными силовыми линиями, образуя круги с их центрами на оси провода, и поскольку направление магнитной силы такое, в котором правый винт двигался бы, если бы вращался. с вихрями следует, что направление магнитной силы вокруг провода будет таким, в котором правый винт вращался бы, двигаясь вперед с током.Среда будет подвергаться растяжению по кругу вокруг проволоки и давлению в плоскостях, проходящих через проволоку, напоминая нам цилиндры пистолета Армстронга.

Если проволоку изогнуть, то будет то же самое, но линии больше не будут точными кругами. Все магнитные силовые линии проходят через замкнутую цепь в направлении, в котором правый винт продвигался бы, если бы вращался в направлении тока. На рис.11, взятом из статьи в Philosophical Magazine, показаны отношения между током, линиями магнитной силы и направлением движения вихрей, стрелки E E ‘представляют ток, SN указывает направление движения. магнитная сила, а стрелки V V ‘показывают направление вращения вихрей.

Теперь предположим, что провод, по которому проходит ток, помещается в магнитное поле под прямым углом к ​​силовым линиям. Пусть S N (рис. 12) представляет собой силовые линии, A — сечение проводника, а ток течет от считывающего устройства через бумагу. В пространстве непосредственно над проводом молекулярные вихри из-за магнитной силы, изначально находящейся в поле, будут вращаться в направлении, в котором их подталкивает ток A, тогда как в пространстве под проводником будет иметь место обратное.Следовательно, скорость вихрей над проволокой будет увеличиваться током, в то время как скорость вихрей ниже проволоки уменьшается. Следовательно, давление среды под прямым углом к ​​силовым линиям будет выше над проводом, чем под ним, и провод будет направлен вниз под прямым углом к ​​силовым линиям и к своему собственному направлению.

Опять же, предположим, что два параллельных провода находятся рядом друг с другом и передают токи в противоположных направлениях. Сила тока определяет разницу между скоростями молекулярных вихрей по разные стороны от него, причем электрические [частицы связаны с вихрями так же, как дифференциальное колесо в механизме; но вихри на одной стороне движущегося потока электрических частиц могут быть остановлены, если скорость вихрей на другой стороне удвоится, при этом ток останется прежним, хотя сами электрические частицы теперь должны будут вращаться, но это заставляет нет разницы.Следовательно, когда параллельные провода передают токи в противоположных направлениях, вихри между ними, заставляемые обоими токами вращаться в одном направлении, будут вращаться быстрее, чем вихри на противоположных сторонах проводов, и сжиматься, как и они, с силой, пропорциональной квадратов их окружных скоростей, провода будут раздвинуты, как если бы они отталкивались друг от друга.

Когда два параллельных провода передают токи в одном направлении, они имеют тенденцию заставлять ячейки в пространстве между ними вращаться в противоположных направлениях, и, следовательно, скорости молекулярных вихрей там будут меньше, чем на другой стороне проводов.Таким образом, давление среды между проводами будет меньше, чем в пространстве за ними, и провода будут сдвинуты вместе, как если бы они притягивались друг к другу.

Теперь предположим, что электрический ток течет по проводу. Молекулярные вихри будут созданы в непосредственной близости от проволоки, и эти вихри, действующие на электрические частицы по другую сторону от проволоки, будут стремиться привести их в движение в направлении, противоположном току в проводе. провод.Но если среда является диэлектриком, частицы не могут перемещаться на ощутимое расстояние. Поэтому они будут заставлены вращаться и запускать другой, более крупный слой вихрей, окружающих проволоку, и поэтому движение будет распространяться, как объяснено выше. Но предположим, что на определенном расстоянии расположен еще один провод, параллельный первому и составляющий часть замкнутой цепи, в которой не течет ток. На частицы электричества в этом проводе будут воздействовать так же, как на частицы в диэлектрике, но, встречая очень небольшое сопротивление их движению по проводу, им легче пройти через провод, чем сразу передать вихрь. движение к упругим телам по другую сторону от них.Но когда ведущий и ведомый соединены дифференциальным колесом, если ведомый тормозится только своей собственной инерцией, сколь бы малым сопротивлением дифференциальному колесу ни было его поступательное движение, оно в конечном итоге заставит ведомый поворачиваться одновременно. оцените как водителя, и сама перестанет двигаться. Следовательно, сопротивление проводника на длине останавливает электрические частицы и заставляет их передавать вихревое движение клеткам за их пределами. Таким образом, когда ток запускается в проводе, переходные токи в противоположном направлении будут индуцироваться в соседних проводниках, в то время как электрическое напряжение будет создаваться в диэлектрике, причем упругие ячейки, движение которых составляет молекулярные вихри, сначала деформируются касательным напряжением электрические частицы, но как индуцированные токи, так и напряжение полностью прекратятся, как только все молекулярные вихри будут в полном разгаре.

Перед тем, как поддерживать ток в первичном проводе, должны быть должным образом запущены молекулярные вихри в окружающем поле, а это требует затрат работы вследствие массы тел, составляющих вихри. Следовательно, для конечной электродвижущей силы невозможно запустить конечный ток за бесконечно короткое время, точно так же, как конечная сила не может мгновенно вызвать конечную скорость в материальном теле, и, как и в динамике, мы иногда говорят о реакции тела на ускорение, как если бы это была сила, противодействующая приложенной силе, поэтому мы иногда говорим о соответствующем действии в случае тока, как если бы это была сила, противодействующая батарее или другому электродвигателю. и говорят об этом как о электродвижущей силе самоиндукции.Однако, поскольку это зависит не просто от тока в проволоке, а от молекулярных вихрей в окружающей среде, ясно, что самоиндукция проволоки будет зависеть от энергии этих вихрей, а это должно зависеть от отношения нескольких частей проволоки друг к другу и к среде, а также от плотности среды. Плотность среды Максвелл отождествлял с ее магнитной проницаемостью. Это больше в (пара-) магнитных веществах, чем в воздухе или вакууме; больше всего в железе.Фактически, в случае железа он настолько велик, что Максвелл предположил, что частицы самого железа принимают участие в вихревом движении. Следовательно, энергия поля и, следовательно, самоиндукция провода тем больше, чем больше магнитная проницаемость окружающей среды, а наличие железного сердечника в катушке значительно увеличивает ее самоиндукцию и энергию, соответствующую заданный ток, протекающий в катушке.

Если после того, как в проводе установился ток, цепь разомкнута или электродвижущая сила снята, молекулярные вихри отказываются останавливаться, пока они не израсходуют свою энергию.Единственным выходом для этой энергии является ток в проводе, поскольку нет возможности выполнять работу в непроводящей среде, где не может быть проскальзывания между элементами механизма. Следовательно, вихри поддерживают движение электричества в проводе после того, как батарея была удалена, до тех пор, пока они не израсходуют всю свою энергию на работу против сопротивления провода.

Но если в поле есть другой проводник, параллельный или слегка наклоненный к первому, есть еще один частичный выход для энергии системы, и «вторичный» ток будет установлен во втором проводе в том же направлении, что и ток в первичной обмотке, тогда как в первичной обмотке будет меньше, чем был бы, если бы вторичная цепь не существовала.Таким образом, гипотеза о молекулярных вихрях дает объяснение как взаимной индукции двух цепей, так и самоиндукции одной.

Предположим, что провод помещают в магнитное поле под прямым углом к ​​силовым линиям, а затем перемещают так, чтобы разрезать линии под прямым углом, мы должны ожидать, что перед движущимся проводом силовые линии или нити вихрей будут сжаты друг с другом в поперечном направлении, но вытянуты в направлении своей длины, примерно так же, как на эластичные струны воздействует проволока, прежде чем они разорвутся и позволят ей пройти.За проволокой будет сброшено боковое давление, вихри будут сжиматься в направлении своих осей и расширяться экваториально. Но мы видели, что эффект растяжения вращающегося упругого тела в направлении его оси вращения и сжатия его под прямым углом к ​​этому направлению увеличивает скорость вращения, так что фактическая скорость каждой точки на поверхности увеличивается. ; в то время как сокращение тела вдоль оси вращения уменьшает скорость. Следовательно, пока проволока движется поперек силовых линий, скорость вихрей перед проволокой будет больше, чем скорость вихрей позади нее, и электрические частицы в проволоке, как они это делают, проходят между двумя группами. вихрей, вращающихся с разными скоростями, будут течь потоком по проволоке.Направление тока в проводе будет таким, при котором вихри спереди будут вращаться быстрее, чем сзади, и, следовательно, будут оказывать большее давление на провод; другими словами, ток будет индуцироваться в таком направлении, чтобы препятствовать движению провода. Мы приходим к аналогичному результату, если предположим, что силовые линии пересекаются под углом, а не под прямым углом. Таким образом, закон Ленца является следствием гипотезы о молекулярных вихрях. Если мы предположим, что магнитная сила действует с юга на север по горизонтали, провод находится вертикально и движется с запада на восток, у нас есть магнитная сила, действующая с юга на север, механическая сила, действующая с востока на запад и противодействующая движению провода. проволока, и электродвижущая сила, действующая в проволоке вертикально вверх.

Предположим, что по всей определенной области электричество продвигается вперед на очень небольшое расстояние вдоль нормали, так что оно не переходит от молекулы к молекуле вещества, а в каждой молекуле претерпевает смещение от задней части к передней. Электрические частицы, прижимающиеся по касательной к стенкам эластичных ячеек, не могут заставить их вращаться, потому что каждая ячейка действует одинаково со всех сторон в направлении, в котором имеет тенденцию двигаться электричество, и поэтому вещество ячейки подвергается деформации сдвига. которому сопротивляется его упругость, а состояние деформации ячеек распространяется через диэлектрик за счет смещения электрических частиц, которые ведут себя как совершенно несжимаемые тела.Когда сила, вызывающая исходное смещение, устраняется, ячейки возвращают свою первоначальную форму в силу своей упругости, электрические частицы возвращаются в свое нормальное положение, а энергия напряженных упругих ячеек расходуется на работу, совершаемую во время электрического разряда. Таким образом, та же среда, которая служит проводником магнитной силы и порождает все явления электромагнетизма, также служит для передачи силы между зарядами статического электричества и как резервуар энергии из-за электростатических зарядов.Если диэлектрик разделен на ячейки с помощью единичных силовых трубок и эквипотенциальных поверхностей, нарисованных для каждой единичной разности потенциалов, каждая ячейка будет содержать одинаковое количество энергии (см. Элементарный трактат об электричестве профессора Джеймса Клерка Максвелла, опубликованный Clarendon Press, 1881.). Следующие цитаты из статьи в Philosophical Magazine объясняют применение гипотезы молекулярных вихрей к статическому электричеству собственными словами Максвелла:

Согласно нашей теории частицы, которые образуют перегородки между ячейками, составляют материю электричества.Движение этих частиц составляет электрический ток; тангенциальная сила, с которой частицы прижимаются веществом ячеек, является электродвижущей силой, а давление частиц друг на друга соответствует напряжению или потенциалу электричества.

Проводящее тело можно сравнить с пористой мембраной, которая оказывает большее или меньшее сопротивление прохождению жидкости; в то время как диэлектрик подобен эластичной мембране, которая может быть непроницаемой для жидкости, но передает давление с одной стороны на [жидкость] с другой.

В диэлектрике, находящемся под индукцией, мы можем представить, что электричество в каждой молекуле смещено так, что одна сторона становится положительной, а другая отрицательно электрической, но что электричество остается полностью связанным с молекулой и не переходит от одной молекулы к другой. Другой.

Эффект этого воздействия на весь диэлектрик заключается в общем смещении электричества в определенном направлении. Это смещение не составляет тока, потому что, когда оно достигает определенного значения, оно остается постоянным, но это начало тока, и его изменения составляют токи в положительном или отрицательном направлении, в зависимости от того, увеличивается или уменьшается смещение. .. . . Когда мы находим электродвижущую силу, вызывающую смещение в диэлектрике, и когда мы обнаруживаем, что диэлектрик восстанавливается из своего состояния электрического смещения с такой же электродвижущей силой, мы не можем не рассматривать явления как явления упругого тела, поддающегося давлению и восстанавливающего свое состояние. образуются при снятии давления.

Предположим, у нас есть положительно наэлектризованное тело. Это означает, что смещение электричества в среде происходит во всех направлениях вокруг тела и от его поверхности.Таким образом, ячейки подвергаются деформации сдвига, уменьшающейся с увеличением расстояния, поскольку поверхность, по которой происходит смещение, увеличивается, линейное смещение электричества пропорционально уменьшается, а частицы электричества ведут себя как совершенно несжимаемая жидкость. Поскольку среда изотропна, линии электрического смещения совпадают с линиями электрического напряжения, которое везде пропорционально смещению. Искажение, которое испытывают клетки под действием давления электрических частиц, вызывает упругое давление во всех направлениях, перпендикулярно направлению смещения, так что давление в среде оказывается перпендикулярно силовым линиям.

Теперь предположим, что у нас есть два положительно заряженных тела в поле, которые, как мы можем предположить, обладают одинаковыми зарядами. Каждый производит смещение среды наружу от себя, но электрические частицы ведут себя как несжимаемая жидкость, ясно, что не может быть линий смещения от одной к другой, но что между телами линии смещения будут изогнутые так, чтобы избегать друг друга так же, как линии потока, исходящие из двух труб, каждая из которых подает воду в резервуар, будут изогнуты круглыми и будут избегать друг друга.Линии смещения и, следовательно, совпадающие с ними силовые линии будут изгибаться точно так же, как магнитные силовые линии, представленные на рис.10, и давление в среде под прямым углом к ​​этим линиям будет вызвать явное отталкивание тел.

При одинаковом смещении, то есть при одинаковых зарядах маленьких тел, отталкивание будет пропорционально упругости среды. Он также пропорционален произведению зарядов или, поскольку они равны, квадрату одного из них.Предположим, что среда заменена на среду с большей эластичностью. Если мы хотим сохранить отталкивание между телами одинаковым, смещения и, следовательно, заряды должны быть уменьшены, причем произведение этих зарядов, то есть квадрат любого заряда, должно быть обратно пропорционально упругости среды. Следовательно, величина каждого заряда должна изменяться обратно пропорционально квадратному корню из упругости среды при изменении диэлектрика. Следовательно, если мы определим электростатическую единицу электричества как «то количество положительного электричества, которое, действуя на равное количество на единичном расстоянии, отталкивает его с единичной силой», то из этого следует, что единица будет меняться в зависимости от характера диэлектрика, будучи обратно пропорциональной пропорционально квадратному корню из его эластичности.

Но притяжение или отталкивание между двумя заданными электрическими зарядами изменяется обратно пропорционально удельной индуктивной емкости диэлектрика, так что электростатическая единица электричества изменяется прямо как квадратный корень из удельной индуктивной емкости, и, таким образом, удельная индуктивная емкость является величиной величина, которая изменяется обратно пропорционально эластичности среды.

Предположим, у нас есть два параллельных провода, передающих одинаковые электрические токи в одном направлении. При остальном без изменений скорость молекулярных вихрей в любой точке пропорциональна силе токов.Мы знаем, что притяжение между проводами пропорционально произведению силы токов, то есть квадрату одного из них. Давление, возбуждаемое вихрями, cœteris paribus пропорционально их плотности и квадрату их скорости. Предположим, мы сохраняем притяжение между проводами неизменным, но изменяем плотность среды. Тогда скорость вихрей в любой точке должна изменяться обратно пропорционально квадратному корню из плотности среды. Но скорость вихрей пропорциональна силе токов.Следовательно, сила каждого тока должна изменяться обратно пропорционально квадратному корню из плотности среды. Если тогда электромагнитная единица тока будет определена как тот ток, который, протекая в определенном проводе, притягивает равный ток в другом данном проводе с единичной силой, то единица тока и, следовательно, единица электричества, которая представляет собой количество, протекающее на секунды через любой участок провода, по которому проходит единичный ток, будет изменяться обратно пропорционально квадратному корню из плотности среды.

Следовательно, отношение электромагнитной к электростатической единице электричества будет пропорционально отношению квадратного корня из упругости к квадратному корню из плотности среды.Но, как известно, это скорость, с которой поперечная вибрация распространяется через среду. Следовательно, соотношение этих единиц представляет собой конкретную скорость и пропорционально скорости распространения электромагнитного возмущения или вышеописанных вихревых движений через диэлектрик. Если единицы выбраны в соответствии с обычной системой, их соотношение не только пропорционально, но и идентично этой скорости.

В статье, опубликованной в журнале Phil. Trans , за 1868 год, профессор Максвелл представил отчет об эксперименте по определению соотношения электростатических и электромагнитных единиц электричества, где воздух является диэлектриком.Принцип метода заключался в уравновешивании притяжения между двумя наэлектризованными дисками за счет отталкивания между двумя витками проволоки, в которых токи текли в противоположных направлениях. Один из дисков и одна катушка были размещены на одном конце балки торсионных весов, другой диск и катушка были закреплены, но третья катушка, передающая тот же ток, что и две другие, была размещена на другом конце балансира. балка, чтобы исключить магнитное действие земли и подвешенной катушки. Аппарат сейчас находится в Кавендишской лаборатории.Результат эксперимента дал для отношения единиц скорость 288 000 000 метров, или 179 000 статутных миль в секунду. Результат получен другим методом по ММ. Вебер и Кольрауш — 310 740 000 метров в секунду. Для электростатических зарядов использовалась батарея М. Гассио на 2600 ячеек, заряженных коррозионным сублиматом. Точность этого результата зависит от точности единицы сопротивления B.A., скорость фактически представлена ​​как
28 ∙ 8 Ом.

Итак, согласно теории волн, свет состоит из поперечных колебаний упругого вещества, пронизывающего пространство и все тела, а скорость света, определенная Фуко, составляет 298 000 000 метров в секунду, или очень близко к среднему значению, полученному Максвеллом. , Вебером и Кольраушем для скорости распространения электромагнитных возмущений.Если так будет всегда, очевидно, что одна и та же среда будет служить для объяснения явлений электростатики и электромагнетизма, а также для распространения света, который, следовательно, должен иметь природу электромагнитного возмущения. Если в идеальном изоляторе возникает электромагнитное возмущение, то мы видели, что оно должно передаваться на неограниченное расстояние, поскольку между электрическими частицами и ячейками не может происходить скольжения, и поскольку сами частицы не могут перемещаться, кроме как путем индуцирования При соответствующем упругом напряжении в среде нет выхода для энергии возмущения, которая, следовательно, должна передаваться от ячейки к ячейке без ограничений.Но если среда является проводником, то есть если электрические частицы могут претерпевать постоянное смещение, переходя от молекулы к молекуле против сопротивления трения и без какой-либо тенденции к возврату, энергия электромагнитного возмущения будет постепенно рассеиваться; поскольку электрические частицы, вместо того, чтобы сообщать все движение одного слоя клеток следующему, сами будут приводиться в движение, и часть энергии будет рассеиваться в виде тепла, а не передаваться внешнему слою клеток.Таким образом, нарушение будет постоянно уменьшаться по мере распространения, пока очень скоро не станет незаметным. Поведение такое же, как у ведущего и ведомого, соединенных дифференциальным колесом, эпициклическое движение которого замедляется силами, имеющими характер трения. Следовательно, электромагнитные возмущения не могут распространяться в проводниках электричества, и поэтому мы делаем вывод, что все истинные проводники непрозрачны для света.

Прозрачность электролитов, таких как солевые растворы и тому подобное, не представляет трудностей перед этим выводом, поскольку передача электричества в них происходит посредством процесса, полностью отличного от истинной проводимости и более похожего на конвекцию тепла, но Максвелл указал, что прозрачность сусального золота намного выше, чем можно было бы предположить в теории.Таким образом, сопротивление отдельного куска сусального золота было таким, что оно должно было пропускать только 10-50 падающего на него света, что было бы совершенно незаметно, в то время как количество зеленого света, фактически передаваемого им, было легко восприниматься. Этот результат профессор Максвелл мог согласовать с теорией, только предположив, что «меньше потери энергии, когда электродвижущие силы меняются на противоположную с быстротой колебаний света, чем когда они действуют в течение разумного времени, как в наших экспериментах.”

Мы видели, что скорость распространения электромагнитного возмущения в любой среде выражается частным из квадратного корня из упругости, деленного на квадратный корень из плотности диэлектрика. Мы узнали, что эластичность обратно пропорциональна удельной индуктивной емкости среды, в то время как плотность соответствует магнитной проницаемости. Отсюда мы заключаем, что скорость распространения электромагнитного возмущения изменяется обратно пропорционально квадратному корню из удельной индуктивной емкости, а также обратно пропорционально квадратному корню из магнитной проницаемости диэлектрика, и это должно быть верно для скорости света, если свет быть электромагнитным возмущением.Магнитная проницаемость большинства прозрачных сред, таких как стекло, кварц, сера, углеводороды и т. Д., Существенно не отличается от проницаемости вакуума, и, следовательно, в этих веществах скорость света должна быть обратно пропорциональна квадрату корень их удельной индуктивной способности; или, поскольку показатель преломления среды — это отношение скорости света в вакууме к его скорости в этой среде, отсюда следует, что показатель преломления должен быть прямо пропорционален квадратному корню из удельной индуктивной способности.Поскольку все наши измерения удельной индуктивной емкости относятся к действию электродвижущих сил, которые продолжаются в течение гораздо большего времени, чем продолжительность световой вибрации, следует ожидать, что последнее указанное соотношение будет наиболее близко соответствовать эксперименту, чем больше длина волны излучения. света или, как иногда утверждают, удельная индуктивная способность диэлектрика должна быть равна квадрату его показателя преломления для «света бесконечной длины волны».

Результаты измерений удельной индуктивной емкости некоторых жидкостей по Силову, а также газов, серы, парафина и смолы, согласуются с этой теорией, как и следовало ожидать.Больцман также обнаружил, что удельные индуктивные способности кристаллической серы вдоль ее трех кристаллографических осей различны, причем эти различия совпадают с разностями квадратов показателей преломления для света, проходящего вдоль этих трех направлений.

Доктор Хопкинсон ( Phil. Trans . Part II. 1881) недавно измерил удельную индуктивную способность скипидара, бензола, нефти, смазочного масла озокерита, касторового масла, масла спермы, оливкового масла и масла для ног.Углеводороды дают результаты, которые полностью согласуются с теорией Максвелла, но жирные масла, которые представляют собой соединения глицерина с жирными кислотами, обладают слишком большой индуктивной способностью. То же самое, по-видимому, происходит со всеми разновидностями стекла, испытанными Гопкинсоном, удельная индуктивная способность которого варьируется от 6 ∙ 61 в случае очень легкого кремня до 9 ∙ 896 для «двойного сверхплотного» кремня. В случае твердого парафина результат Хопкинсона почти совпадает с результатом Больцмана и теорией Максвелла.В случае стекла, как и в случае жирных масел, высокая удельная индуктивная способность связана со сложным химическим составом стекла, состоящим в основном из силикатов металлов, включая силикаты щелочных и щелочно-земельных металлов. Измерение удельной индуктивной емкости стекла сопряжено с большими трудностями из-за явления, обычно известного как остаточный заряд или поглощение электричества, то есть очевидного проникновения электричества в вещество стекла.Это предмет, которым Максвелл проявил очень большой интерес, и в своей работе по электричеству и магнетизму он дал механическую иллюстрацию действия, исходя из предположения, что оно происходит из-за отсутствия однородности в стекле, некоторые части которого он должны проводить электричество лучше, чем другие, хотя в лучшем случае плохо. Форма эксперимента, очень красивая по своему замыслу, была разработана Максвеллом для измерения удельной индуктивной емкости и была проведена г-ном Дж. Э. Х. Гордоном, который смог обратить электрическое напряжение в стекле вспять 12 000 раз в секунду; но это, конечно, не приближение к быстрому чередованию «волн» света.Однако с использованием используемого аппарата обработка наблюдений сопряжена с большими математическими трудностями, и поэтому к результатам следует относиться с осторожностью, независимо от того, рассматриваем ли мы их как поддерживающие теорию или как противоположные ей.

Применяя гипотезу молекулярных вихрей к действию магнитного поля на поляризованный свет, Максвелл «обнаружил, что единственный эффект, который вращение вихрей будет оказывать на свет, — это заставить плоскость поляризации вращаться в том же направлении. как вихри, через угол, пропорциональный

  1. к толщине вещества.
  2. к разрешенной части магнитной силы, параллельной лучу.
  3. к показателю преломления луча.
  4. обратно квадрату длины волны в воздухе.
  5. к среднему радиусу вихрей.
  6. к емкости по магнитной индукции ».

Соотношение (E) между величиной вращения и размером вихрей показывает, что разные вещества могут различаться по вращательной способности независимо от любых наблюдаемых различий в других отношениях.Мы ничего не знаем об абсолютном размере вихрей; и по нашей гипотезе оптические явления, вероятно, единственные данные для определения их относительного размера в различных веществах.

Теперь, независимо от действия магнитного поля на поляризованный свет, все явления диамагнетизма могут быть объяснены на основе гипотезы о том, что магнитная проницаемость диамагнитных веществ меньше, чем у вакуума, поэтому они ведут себя как парамагнитные вещества. погружен в среду более магнитную, чем она сама.Но Максвелл указал, что «поскольку М. Верде обнаружил, что магнитные вещества действуют на свет, противоположное действию диамагнитных веществ, из этого следует, что вращение молекул должно быть противоположным в этих двух классах веществ».

Следовательно, мы больше не можем рассматривать диамагнитные тела как те, у которых коэффициент магнитной индукции меньше, чем у пространства, свободного от грубой материи. Мы должны признать, что диамагнитное состояние противоположно парамагнетику; и что вихри или, по крайней мере, большинство из них в диамагнитных веществах вращаются в том направлении, в котором вращается положительное электричество в намагничивающей катушке, тогда как в парамагнитных веществах они вращаются в противоположном направлении.

Возможно, мы не сможем завершить этот отчет о гипотезе молекулярных вихрей лучше, чем процитировав собственные слова Максвелла:

Я думаю, у нас есть веские доказательства того, что в магнитном поле происходит какое-то явление вращения; что это вращение выполняется большим количеством очень маленьких частей материи, каждая из которых вращается вокруг своей оси, причем эта ось параллельна направлению магнитной силы, и что вращения этих различных вихрей зависят друг от друга. посредством какого-то механизма, соединяющего их.

Попытка, которую я [предпринял] вообразить работающую модель этого механизма, должна рассматриваться не более, чем она есть на самом деле, демонстрации того, что механизм можно представить способным производить соединение, механически эквивалентное действительному соединению частей электромагнитное поле. Задача определения механизма, необходимого для установления заданного вида связи между движениями частей системы, всегда допускает бесконечное число решений. Некоторые из них могут быть более неуклюжими или более сложными, чем другие, но все они должны удовлетворять условиям механизма в целом.

Однако следующие результаты теории имеют большее значение:

  1. Магнитная сила — это эффект центробежной силы вихрей.
  2. Электромагнитная индукция токов — это действие сил, возникающих при изменении скорости вихрей.
  3. Электродвижущая сила возникает из-за нагрузки на соединительный механизм.
  4. Электрическое смещение возникает из-за упругой податливости соединительного механизма.

В статье, озаглавленной «Динамическая теория электромагнитного поля», зачитанной перед Королевским обществом 8 декабря 1864 года, Максвелл вывел все вышеперечисленные результаты чисто механическими рассуждениями, только предполагая существование среды, способной принимать и хранить увеличивают потенциальную и кинетическую энергию и, следовательно, способны выполнять работу по «восстановлению после смещения в силу своей упругости», в то время как части среды соединены «сложным механизмом, способным к самым разнообразным движениям, но в то же время так связаны, что движение одной части зависит, в соответствии с определенными соотношениями, от движения других частей, причем эти движения сообщаются силами, возникающими из относительных перемещений соединенных частей в силу их упругости.«Для существования такой среды у нас есть свидетельства, не зависящие от электрических воздействий. Что касается механизма, то в статье не делается попыток придать ему какой-либо определенный строй. Эта статья считается величайшим вкладом Максвелла в науку об электричестве, но большинство полученных в ней результатов уже упоминалось.

Ниже приводится хороший образец юмористической иронии Максвелла, множество примеров которой есть в его научных работах. Он обсуждает некоторые разработки Бернхарда Римана Лоренцо, теории электромагнетизма Вебера и Неймана, которая основана на предположении, что действие между двумя величинами электричества является прямым действием на расстоянии и зависит не только от расстояния между ними. зарядов, но при их относительном движении.

Исходя из предположений, содержащихся в обеих этих статьях, мы можем сначала сделать выводы, что действие и противодействие не всегда равны и противоположны; и, во-вторых, это устройство может быть сконструировано для создания любого объема работы за счет собственных ресурсов.

Я думаю, что этих замечательных выводов из последних разработок теории Вебера и Неймана можно избежать, только признав действие среды в электрических явлениях.

Находясь в Кавендишской лаборатории, Максвелл сконструировал механическую модель, которая очень красиво иллюстрирует основные явления индуцированных токов.Как часть механизма, это просто дифференциальный поезд, который часто используется в качестве динамометра для измерения мощности, потребляемой машиной. Схема устройства показана на рис. 13. Рифленое колесо P прикреплено к тому же валу, что и коническое колесо A, которое поэтому вращается вместе с ним, и вращение этой детали представляет собой первичный ток. Второе коническое колесо D свободно вращается на рычаге C D, который является одним из четырех рычагов (из которых только два показаны на рисунке), образующих крест, который может свободно вращаться на центральном валу в точке C.Скользящие грузы M M ’и т. Д. Могут быть закреплены в любом желаемом положении на этих рычагах для изменения момента инерции крестовины, которая является дифференциальной деталью в механизме. Третье коническое колесо B прикреплено шпонкой к тому же полому валу, что и колесо S, которое аналогично P, и вращение детали B S представляет ток во вторичной цепи. Поскольку вал B S полый и свободно движется по валу A C, колеса A и B могут вращаться совершенно независимо друг от друга, за исключением случаев, когда они соединены колесом D.P ’- это указатель, прикрепленный к внутреннему валу и вращающийся с помощью P. Петля из веревки навешена на каждое из колес P и S и несет небольшой груз. Эти струны действуют как фрикционные тормоза для колес, а трение представляет собой сопротивление первичного и вторичного контуров соответственно. Момент инерции нагруженного крестовины или дифференциальной детали представляет собой моменты инерции ячеек, которые составляют молекулярные вихри в диэлектрике. Его кинетическая энергия при вращении представляет собой энергию вихрей, а его угловой момент пропорционален электромагнитному импульсу системы.Моменты инерции других частей механизма очень малы по сравнению с моментом инерции нагруженной крестовины. Движению крестовины и колеса D препятствует минимально возможное трение.

Предположим, что колесо P вращается, представляя ток в первичном проводе; тяжелый крест сначала не будет двигаться, но колесо D будет вращаться и сообщать движение B, который, с помощью S, будет вращаться в направлении, противоположном направлению P, представляя ток во вторичной цепи, противоположный этому направлению. в первичной.Но движению S препятствует фрикционный тормоз, и поэтому конечная сила должна быть приложена D к B, чтобы привести его в движение. Реакция B вместе с «силой, прилагаемой A, будет постоянно стремиться к тому, чтобы крест вращался в том же направлении, что и P, а скорость креста постоянно увеличивается, и в настоящее время он будет вращаться со скоростью, вдвое меньшей, чем P». , а затем D обернется вокруг B, который вместе с S останется в покое. Затем деталь BS будет оставаться в состоянии покоя до тех пор, пока вращение P остается постоянным, что соответствует прекращению тока во вторичной цепи, в то время как вращение в первичной остается неизменным, но если P ускоряется, S будет вращаться в противоположном направлении. к движению П.Теперь предположим, что P внезапно остановился. Кинетическая энергия креста заставит его продолжать вращаться до тех пор, пока он не выполнит соответствующее количество работы, преодолевая сопротивление, и, находясь в состоянии покоя, D будет катиться по нему и заставит B с помощью S вращаться в том же направлении, что и крест, то есть в том же направлении, в котором раньше вращался P, и каким бы ни было сопротивление движению S, он будет преодолен, и S будет вращаться до тех пор, пока работа, выполненная против сопротивления, не станет равной кинетической энергии, которой изначально обладала крестом.Это соответствует току, индуцируемому во вторичной катушке при остановке тока в первичной обмотке, причем ток идет в том же направлении, что и первичный ток, и продолжается до тех пор, пока энергия вращения молекулярных вихрей не будет израсходована на работу, выполняемую против электрического тока. сопротивление.

Если один оператор держится за колесо S и пытается удержать его в покое, в то время как другой прикладывает постоянную силу к P, движение P будет ускоряться намного медленнее, чем если бы к нему была приложена такая же сила, и S был свободен, потому что P может двигаться, только приводя в движение крест с его большим моментом инерции.Если оператор, который теперь поворачивает P, внезапно остановит его, машина будет испытывать сильный удар, и колесо S выскользнет из захвата другого оператора, как бы крепко он его ни держал. Сила, приложенная к S, может соответствовать воздушному разрыву во вторичной обмотке, и этого достаточно для предотвращения искры, когда ток батареи запускается в первичной обмотке, но за счет внезапной остановки первичного тока, как в катушке Кухмкорфа, разрушающего разряда или искра проходит по воздуху между выводами вторичного провода.(Если оператор, который пытается удерживать колесо S в состоянии покоя, неопытен, эффект для него очень поразителен).

Если вставить штифт на лицевую сторону колеса S и один конец пружины прижать к штифту, а другой конец прикреплен к раме устройства, мы получим представление вторичной катушки, в которой цепь сломан, и лейденская банка вставлена ​​с ее покрытиями в концы проволоки. Когда движение P изменяется, S начинает двигаться и отклоняет пружину, что соответствует току во вторичной катушке, заряжающей лейденскую банку.Если пружина допускает очень большой прогиб, так что над ней необходимо проделать большой объем работы, прежде чем она выскользнет из штифта, первичный ток может достичь своей полной силы до того, как произойдет скольжение. Это соответствует тому, что емкость лейденской банки слишком велика, чтобы позволить ее зарядить до потенциала, достаточного для образования искры. В этом случае искры не происходит, но когда сила между колесами D и B уменьшается из-за уменьшения ускорения P, пружина снимает напряжение, заставляя колесо S двигаться назад, и лейденская банка при соответствующих обстоятельствах тихо разряжается через вторичную обмотку, изменяя процесс зарядки в обратном направлении.Но если штифт выскользнет из пружины, колесо S будет вращаться, и пружина отлетит назад, что соответствует разрушительному разряду через воздух, и если ускорение P будет продолжаться достаточно долго, может произойти несколько таких разрушительных разрядов.

Мы, конечно, должны быть осторожны, чтобы не пытаться извлечь из такой модели уроки, которым она не была предназначена, и мы должны помнить, что поведение механизма не отражает электрическое действие во всех отношениях.

На протяжении многих лет Максвелл оказывал ценную услугу Британской ассоциации, особенно в области электротехники.Некоторые отчеты о собраниях, на которых он присутствовал, можно найти в письмах, напечатанных в другой части этой работы, и хотя в последние несколько лет его жизни другие занятия мешали ему посещать ежегодные собрания, он всегда выказывал признаки острого удовольствия, когда обсуждая «британских задниц». В 1862 году он был назначен членом «Комитета по стандартам электрического сопротивления». В отчете, выпущенном в 1863 г., Приложение «Об элементарных взаимосвязях между электрическими измерениями» носит имя профессора Максвелла в сочетании с именем профессора Флиминга Дженкина, в то время как общее описание метода, используемого для определения Ом или Б.А. Единица сопротивления, вместе с математической теорией и деталями экспериментов, написаны пером Максвелла. В 1863–1843 годах Максвелл снова работал над тем же предметом в лаборатории Королевского колледжа, и большая часть «спинов» проводилась под его собственным наблюдением. В 1869 году результаты экспериментов Максвелла об отношении электромагнитного поля к электростатической единице электричества, описанные выше, были воплощены в Отчете Британской ассоциации на собрании в Данди, и это составляет последний из отчетов Комитета.

В 1874 году профессор Максвелл был избран членом комитета, назначенного Британской ассоциацией с целью исследования закона Ома. Большая часть работы, выполненной этим комитетом, была выполнена профессором Кристал в Кавендишской лаборатории под наблюдением и по предложению профессора Максвелла. Отчет об исследованиях будет найден в отчете, представленном Ассоциации на собрании в Глазго в 1876 году.

Прежде чем закончить наше сообщение о вкладе Максвелла в науку об электричестве, мы должны упомянуть о подготовке к печати книги «Электрические исследования достопочтенного Генри Кавендиша», опубликованной в 1879 году, всего за несколько недель до смерти редактора.Объем труда, который профессор Максвелл вложил в эту работу в течение последних пяти лет своей жизни, может быть известен только тем, кто постоянно находился в его компании. Практически все файлы MS. он записывал собственноручно, большая часть копировалась после полуночи, пока он наблюдал за миссис Максвелл во время долгой болезни, о которой упоминалось в другом месте. Каждый неясный отрывок или намек был предметом долгого и тщательного исследования; и многие из них были написаны библиотекарем Королевского общества, а также научным и литературным друзьям в разных частях страны, чтобы получить информацию о значении устаревших слов и символов или об истории отдельных людей.Но помимо этого и сравнения результатов Кавендиша с результатами, полученными последующими исследователями, Максвелл повторил многие эксперименты Кавендиша почти в их первоначальной форме, используя только современные инструменты для измерения. Введение и приложения к работе свидетельствуют о большом труде, терпеливых исследованиях и очень обширном знакомстве с литературой, имеющей отношение к этой теме. Максвелл ни в коем случае не относился к тому классу «мыслителей», которые читали только свои собственные сочинения; его знакомство не только с научной литературой, но и почти со всеми другими классами книг было поразительным; и если ему предлагали какой-либо вопрос физики, он обычно мог дать отчет почти обо всем, что было сделано по этому предмету.В этом отношении он походил на покойного профессора В. Х. Миллера, с которым кембриджские люди консультировались обо всем.

Ссылка
Кэмпбелл, Льюис и Уильям Гарнетт. Жизнь Джеймса Клерка Максвелла: с выборкой из его переписки и случайных писем и очерк его вклада в науку . Лондон: Macmillan and Co. 1882. 513-556.

электромагнетизм | Определение, уравнения и факты

Электромагнетизм , наука о заряде, а также о силах и полях, связанных с зарядом.Электричество и магнетизм — два аспекта электромагнетизма.

Британская викторина

Викторина «Все о физике»

Кто был первым ученым, проведшим эксперимент по управляемой цепной ядерной реакции? Какая единица измерения для циклов в секунду? Проверьте свою физическую хватку с помощью этой викторины.

Электричество и магнетизм долгое время считались отдельными силами.Только в 19 веке они стали рассматриваться как взаимосвязанные явления. В 1905 году специальная теория относительности Альберта Эйнштейна без всяких сомнений установила, что оба аспекта являются аспектами одного общего явления. Однако на практике электрические и магнитные силы ведут себя по-разному и описываются разными уравнениями. Электрические силы создаются электрическими зарядами, находящимися в состоянии покоя или в движении. С другой стороны, магнитные силы создаются только движущимися зарядами и действуют исключительно на движущиеся заряды.

Электрические явления происходят даже в нейтральной материи, потому что силы действуют на отдельные заряженные составляющие. В частности, электрическая сила отвечает за большинство физических и химических свойств атомов и молекул. Он невероятно силен по сравнению с гравитацией. Например, отсутствие только одного электрона на каждый миллиард молекул у двух 70-килограммовых (154-фунтовых) людей, стоящих на расстоянии двух метров (двух ярдов) друг от друга, оттолкнет их с силой в 30 000 тонн. В более привычном масштабе электрические явления ответственны за молнии и гром, сопровождающие определенные штормы.

Электрические и магнитные силы могут быть обнаружены в областях, называемых электрическими и магнитными полями. Эти поля имеют фундаментальную природу и могут существовать в космосе вдали от заряда или тока, которые их породили. Примечательно, что электрические поля могут создавать магнитные поля и наоборот, независимо от внешнего заряда. Как обнаружил в своей работе английский физик Майкл Фарадей, изменяющееся магнитное поле создает электрическое поле, лежащее в основе производства электроэнергии. И наоборот, изменяющееся электрическое поле создает магнитное поле, как пришел к выводу шотландский физик Джеймс Клерк Максвелл.Математические уравнения, сформулированные Максвеллом, включают световые и волновые явления в электромагнетизм. Он показал, что электрические и магнитные поля путешествуют вместе в пространстве как волны электромагнитного излучения, при этом изменяющиеся поля взаимно поддерживают друг друга. Примерами электромагнитных волн, распространяющихся в пространстве независимо от материи, являются радио- и телевизионные волны, микроволны, инфракрасные лучи, видимый свет, ультрафиолетовый свет, рентгеновские лучи и гамма-лучи. Все эти волны движутся с одинаковой скоростью, а именно скоростью света (примерно 300 000 километров или 186 000 миль в секунду).Они отличаются друг от друга только частотой, с которой колеблются их электрическое и магнитное поля.

Получите подписку Britannica Premium и получите доступ к эксклюзивному контенту. Подпишитесь сейчас

Уравнения Максвелла по-прежнему обеспечивают полное и элегантное описание электромагнетизма вплоть до субатомного масштаба, но не включая его. Однако интерпретация его работ в 20 веке расширилась. Специальная теория относительности Эйнштейна объединила электрические и магнитные поля в одно общее поле и ограничила скорость всей материи скоростью электромагнитного излучения.В конце 1960-х физики обнаружили, что у других сил в природе есть поля с математической структурой, подобной структуре электромагнитного поля. Эти другие силы представляют собой сильное взаимодействие, ответственное за энергию, выделяемую при ядерном синтезе, и слабое взаимодействие, наблюдаемое при радиоактивном распаде нестабильных атомных ядер. В частности, слабые и электромагнитные силы были объединены в общую силу, называемую электрослабой силой. Цель многих физиков объединить все фундаментальные силы, включая гравитацию, в одну великую единую теорию, до сих пор не достигнута.

Важным аспектом электромагнетизма является наука об электричестве, которая занимается поведением агрегатов заряда, включая распределение заряда в материи и движение заряда с места на место. Различные типы материалов классифицируются как проводники или изоляторы в зависимости от того, могут ли заряды свободно перемещаться через составляющие их вещества. Электрический ток — это мера потока зарядов; законы, управляющие токами в материи, важны в технологиях, особенно в производстве, распределении и контроле энергии.

Понятие напряжения, как и понятия заряда и тока, является фундаментальным в науке об электричестве. Напряжение — это мера склонности заряда перетекать из одного места в другое; положительные заряды обычно имеют тенденцию перемещаться из области высокого напряжения в область более низкого напряжения. Распространенная проблема в электричестве — это определение взаимосвязи между напряжением и током или зарядом в данной физической ситуации.

Эта статья стремится дать качественное понимание электромагнетизма, а также количественную оценку величин, связанных с электромагнитными явлениями.

Закон Фарадея

Закон Фарадея
Далее: Электрический скалярный потенциал? Up: Зависящие от времени уравнения Максвелла Предыдущая: Введение История развития физики человечеством можно рассматривать как историю синтеза идей. Физики продолжают находить, что очевидно несопоставимые явления можно понимать как разные аспекты некоторых более фундаментальное явление. Этот процесс продолжается до сегодняшнего дня. явления могут быть описаны в терминах трех фундаментальных сил: силы тяжести , силы тяжести Электрослабая сила и сильная сила .Одна из главных целей современной физики состоит в том, чтобы найти способ объединить эти три силы, чтобы все физики можно описать в терминах единой объединенной силы. Этот, по сути, это цель теории струн.

Первый великий синтез идей в физике произошел в 1666 году, когда Исаак Ньютон понял, что сила, заставляющая яблоки падать вниз, такая же, как у сила, которая удерживает планеты на эллиптических орбитах вокруг Солнца. Второй великий синтез, который мы собираемся изучить более подробно, произошел в 1830 г., когда Майкл Фарадей открыл, что электричество и магнетизм — это два аспекты одного и того же, обычно называемые электромагнетизм .Третий великий синтез, который мы обсудим в настоящее время произошло в 1873 году, когда Джеймс Клерк Максвелл продемонстрировал этот световой и электромагнетизм тесно связаны. Последний (но, надеюсь, не последний) великий синтез произошел в 1967 году, когда Стив Вайнберг и Абдус Салам показал, что электромагнитная сила и слабое ядерное взаимодействие (, то есть , которое отвечает за распады) можно комбинировать для создания электрослабой силы. К сожалению, работа Вайнберга выходит далеко за рамки этого курса лекций.

Давайте теперь рассмотрим эксперименты Фарадея, поместив их в надлежащее положение. исторический контекст. До 1830 года единственный известный способ изготовления электрического ток через проводник должен был соединить концы провода с положительное и отрицательное клеммы аккумуляторной батареи. Мы измеряем способность батареи выдавать ток вниз по проводу с точки зрения его напряжения , под которым мы понимаем разницу напряжений между его положительным и отрицательным выводами. Чему соответствует напряжение в физику? Что ж, вольт — это единицы измерения электрического скалярного потенциала, поэтому, когда мы Говоря о батарее 6 В, мы действительно говорим о том, что разница в электрический скалярный потенциал между его положительным и отрицательным выводами составляет шесть вольт.Это понимание позволяет нам писать

(370)

где — напряжение аккумуляторной батареи, обозначает положительный полюс, отрицательный вывод, и является элементом длины вдоль провод. Конечно, приведенное выше уравнение является прямым следствием . Ясно, что разница напряжений между двумя концами провода прикрепленный к батарее подразумевает наличие электрического поля, которое проталкивает заряды через провод. Это поле направлено от положительной клеммы аккумулятора к отрицательной. терминал, и, следовательно, таков, что заставляет электроны течь через провод от отрицательного к положительному выводу.Как и ожидалось, это означает, что Чистая положительный ток течет от положительной клеммы к отрицательной. Дело в том, что — консервативное поле гарантирует, что разность напряжений не зависит от путь провода. Другими словами, два разных провода, подключенных к одной и той же батарее развиваются одинаковые разности напряжений.

Давайте теперь рассмотрим замкнутый контур провода (без батареи). Напряжение вокруг такой петли, которую иногда называют электродвижущей силой . force или эл.м.ф. , это

(371)

Это прямое следствие уравнения поля . Итак, поскольку это консервативное поле, то электродвижущая сила вокруг замкнутый контур провода автоматически равно нулю, и ток по проводу не течет. Кажется, все это имеет смысл. Однако Майкл Фарадей собирается бросить гаечный ключ в наших работах! В 1830 году он обнаружил, что изменяющееся магнитное поле может вызвать протекание тока вокруг замкнутого шлейф из провода (при отсутствии аккумулятора).Ну, а если по проводу течет ток, значит должен быть электродвигатель. сила. Так,
(372)

что сразу означает, что это не консервативная область, и что . Ясно, что нам придется изменить некоторые наших идей относительно электрических полей.

Фарадей продолжил свои эксперименты и обнаружил, что другой способ создания электродвижущей силы вокруг проволочной петли состоит в том, чтобы поддерживать постоянное магнитное поле и переместите петлю.В конце концов, Фарадей смог сформулировать закон, который объяснял все его эксперименты. Э.д.с. генерируется вокруг проволочной петли в магнитном поле, пропорционально скорость изменения потока магнитного поля через петлю. Так, если обозначена петля, и это некоторая поверхность, прикрепленная к петле, то фарадеевский эксперименты можно резюмировать, написав

(373)

где — коэффициент пропорциональности.Таким образом, изменяющийся поток магнитного поля через петлю создает электрическое поле, направленное вокруг петли. Этот процесс известен как магнитная индукция .

единиц S.I. были тщательно отобраны, чтобы приведенное выше уравнение. Единственное, что нам теперь нужно решить, это то, или же . Другими словами, в каком направлении вокруг петли возникает наведенная ЭДС. хотите погонять ток? У нас есть общий принцип, который позволяет нам решать подобные вопросы.Это называется Принцип Ле-Шателье . Согласно принципу Ле Шателье, каждое изменение вызывает реакцию, которая пытается минимизировать изменение. По сути, это означает что Вселенная устойчива к малым возмущениям. Когда этот принцип применяется к частному случаю магнитная индукция, его обычно называют законом Ленца . По словам Ленца закон, ток, индуцированный вокруг замкнутого контура всегда такова, что создаваемое магнитное поле пытается противодействовать изменение магнитного потока, создающего электродвижущую силу.Из рис. 34 видно, что если магнитное поле увеличивается, и ток циркулирует по часовой стрелке (как видно сверху), затем он создает поле, которое противодействует увеличению магнитного потока через петлю, в в соответствии с законом Ленца. Направление тока противоположно смысл токовой петли (при условии, что поток через петля положительна), поэтому это означает, что в уравнении. (373). Таким образом, фарадеевский закон принимает форму

(374)

Рисунок 34:

Экспериментально установлено, что закон Фарадея правильно предсказывает е.м.ф. ( то есть , ) генерируется в любом проводном шлейфе, независимо от положение или форма петли. Резонно предположить, что та же ЭДС. было бы генерируется в отсутствие провода (конечно, ток не будет течь в таком случае). Таким образом, уравнение. (374) действительно для любого замкнутого контура. Если Фарадея закон должен иметь какой-то смысл, тогда он также должен быть верным для любой поверхности, прикрепленной к петля . Ясно, что если поток магнитного поля через петлю зависит от поверхность, на которой это оценивается, то закон Фарадея будет предсказывать разные е.m.f.s для разных поверхностей. Поскольку нет предпочтительной поверхности для обычная некопланарная петля, это не имело бы особого смысла. Условие для потока магнитного поля, , зависеть только на петле, к которой прикреплена поверхность, а не на натуре самой поверхности, является

(375)

для любой закрытой поверхности.

Закон Фарадея, Ур. (374), можно преобразовать в уравнение поля, используя Теорема Стокса.Мы получаем

(376)

Это последнее уравнение Максвелла. Он описывает, как изменяющееся магнитное поле может генерировать или индуцировать электрическое поле. Теорема Гаусса применима к уравнению. (375) дает знакомое уравнение поля
(377)

Это гарантирует, что магнитный поток через петлю является четко определенной величиной.

Расхождение уравнения.(376) дает

(378)

Таким образом, уравнение поля (376) фактически требует, чтобы дивергенция магнитное поле должно быть постоянным во времени для самосогласования (это означает что поток магнитного поля через петлю не обязательно должен быть четко определенным количество, пока его производная по времени хорошо определена). Однако постоянный несоленоидное магнитное поле может быть создано только магнитными монополями, а магнитных монополей не существует (насколько нам известно).Следовательно, . Отсутствие магнитных монополей это факт наблюдения: его не может предсказать никакая теория. Если магнитные монополи будут открыты завтра, это не вызовет у физиков какие-то проблемы. Мы знаем, как обобщить уравнения Максвелла, чтобы включить как магнитные монополи, так и токи магнитных монополей. В этом обобщенном формализма, уравнения Максвелла полностью симметричны относительно электрические и магнитные поля, а также . Тем не мение, дополнительный член (включающий ток магнитных монополей) должен быть добавлен к правая часть уравнения.(376), чтобы сделать его самосогласованным.

Далее: Электрический скалярный потенциал? Up: Зависящие от времени уравнения Максвелла Предыдущая: Введение
Ричард Фицпатрик 2006-02-02

Закон Фарадея

Закон Фарадея
Далее: Закон Ленца Up: Магнитная индукция Предыдущий: Магнитная индукция Явление магнитной индукции играет решающую роль в три очень полезных электрических устройства: электрогенератор , электрический двигатель , и трансформатор .Без этих устройств современная жизнь была бы невозможно в нынешнем виде. Магнитная индукция была открыта в 1830 г. Английский физик Майкл Фарадей. Американский физик Джозеф Генри независимо друг от друга сделал то же открытие примерно в одно и то же время. Оба физиков заинтриговал тот факт, что электрический ток, протекающий вокруг цепь может генерировать магнитное поле. Наверняка, рассуждали они, если электрический ток может генерировать магнитное поле, тогда магнитное поле должно каким-то образом быть способным генерировать электрический ток.Однако потребовалось много лет бесплодных экспериментов. прежде, чем они смогли найти необходимый ингредиент, который позволяет магнитное поле для генерации электрического тока. Этот ингредиент — , изменение во времени .

Рассмотрим плоскую петлю из токопроводящего провода соответствующей площади поперечного сечения. Поместим эту петлю в магнитное поле, напряженность которого приблизительно равна равномерный по всей длине петли. Предположим, что направление магнитное поле образует угол с нормальным направлением к петля.Магнитный поток через петлю равен определяется как произведение площади петли и составляющей магнитное поле, перпендикулярное петле. Таким образом,

(191)

Если петля оборачивается вокруг себя раз (, т.е. , если петля имеет витков ), то магнитный поток через петлю просто умножить на магнитный поток за один виток:
(192)

Наконец, если магнитное поле неоднородно по петле или петля не лежать в одной плоскости, тогда мы должны оценить магнитный поток как поверхностный интеграл
(193)

Вот какая-то поверхность, к которой прикреплена.Если петля имеет витки, то поток в несколько раз превышает указанное выше значение. Единица измерения магнитного потока в системе СИ — вебер (Вб). Одна тесла эквивалентна один вебер на квадратный метр:
(194)

Фарадей обнаружил, что если магнитное поле через петлю из проволоки изменяется во времени. , тогда вокруг контура индуцируется ЭДС. Фарадей смог наблюдать этот эффект, потому что ЭДС вызывает ток, циркулирующий в петле.Фарадей обнаружил, что величина ЭДС прямо пропорциональна скорости изменения магнитного поля во времени. Он также обнаружил, что ЭДС генерируется, когда петля провода перемещается на . из области низкой напряженности магнитного поля в область высокой напряженности магнитного поля, и наоборот . ЭДС прямо пропорциональна скорость, с которой петля перемещается между двумя областями. Ну наконец то, Фарадей обнаружил, что ЭДС генерируется вокруг петли, которая на поворачивается на . в однородном магнитном поле постоянной напряженности.В этом случае ЭДС прямо пропорциональна скорости вращения петли. Фарадей в конце концов может предложить единый закон, который мог объяснить все его многочисленные и разнообразные наблюдения. Этот закон, известный как Закон магнитной индукции Фарадея выглядит следующим образом:

ЭДС, наведенная в цепи, пропорциональна скорости изменения во времени магнитный поток, связывающий эту цепь.
Единицы СИ были зафиксированы таким образом, чтобы константа пропорциональности в этом закон единицы .Таким образом, если магнитный поток через цепь изменяется на сумму во временном интервале тогда генерируемая в цепи ЭДС равна
(195)

Есть много разных способов, которыми магнитный поток, связывающий электрическая цепь может менять. Может изменяться либо напряженность магнитного поля, либо направление магнитного поля. поле может измениться, или положение цепи может измениться, или форма цепь может измениться, или ориентация цепи может измениться.Закон Фарадея гласит, что все эти способы полностью эквивалент в части генерации ЭДС вокруг цепь касается.



Далее: Закон Ленца Up: Магнитная индукция Предыдущий: Магнитная индукция
Ричард Фицпатрик 2007-07-14

Фарадей и электромагнитная теория света

Майкл Фарадей (22 сентября 1791 — 25 августа 1867), вероятно, наиболее известен своим открытием электромагнитной индукции, его вкладом в электротехнику и электрохимию или тем фактом, что он отвечал за введение концепции поля в физике. описать электромагнитное взаимодействие.Но, возможно, не так хорошо известно, что он также внес фундаментальный вклад в электромагнитную теорию света .

В 1845 году, всего 170 лет назад, Фарадей обнаружил, что магнитное поле влияет на поляризованный свет — явление, известное как магнитооптический эффект или эффект Фарадея. Чтобы быть точным, он обнаружил, что плоскость вибрации луча линейно поляризованного света, падающего на кусок стекла, вращалась, когда магнитное поле было приложено в направлении распространения луча.Это было одним из первых указаний на связь электромагнетизма и света. В следующем году, в мае 1846 года, Фарадей опубликовал статью Мысли о вибрациях лучей , пророческую публикацию , в которой он предположил , что свет может быть вибрацией электрических и магнитных силовых линий.

Майкл Фарадей (1791-1867) / Источники: Wikipedia

Случай Фарадея нечасто встречается в истории физики: хотя его обучение было очень простым, законы электричества и магнетизма в гораздо большей степени связаны с экспериментальными открытиями Фарадея, чем с любыми другими учеными.Он открыл электромагнитной индукции , что привело к изобретению динамо-машины, предшественницы электрического генератора. Он объяснил электролиз с точки зрения электрических сил, а также ввел такие концепции, как поле , и силовые линии , , которые не только были фундаментальными для понимания электрических и магнитных взаимодействий, но и легли в основу дальнейших достижений в физике.

Майкл Фарадей родился в Южном Лондоне в скромной семье.Единственное базовое формальное образование, которое он получил в детстве, — это чтение, письмо и арифметика. Он бросил школу, когда ему было тринадцать, и начал работать в переплетном магазине. Его страсть к науке была пробуждена описанием электричества , которое он прочитал в копии Британской энциклопедии , которую он переплетал, после чего он начал экспериментировать в импровизированной лаборатории. 1 марта 1813 года Фарадей был нанят в качестве лаборанта Хэмфри Дэви в Королевском институте в Лондоне, членом которого он был избран в 1824 году и где он проработал до своей смерти в 1867 году, сначала помощником Дэви, затем его сотрудником и, наконец, , после смерти Дэви, как его преемник.Фарадей произвел на Дэви такое впечатление, что когда последнего спросили о его величайшем открытии, Дэви ответил: «Моим величайшим открытием был Майкл Фарадей». В 1833 году он стал первым фуллеровским профессором химии в Королевском институте. Фарадей также признан великим популяризатором науки. В 1826 году Фарадей основал в Королевском институте «Пятничные вечерние лекции», которые являются каналом связи между учеными и непрофессионалами. В следующем году он запустил Рождественские лекции для молодежи, которые ежегодно транслируются по национальному телевидению, серию, цель которой — представить науку широкой публике.Многие из этих лекций читал сам Фарадей. Оба они продолжаются по сей день.

Майкл Фарадей читал рождественскую лекцию в Королевском институте в 1856 г. / Источники: Википедия

Фарадей сделал свое первое открытие электромагнетизма в 1821 г. Он повторил эксперимент Эрстеда , поместив небольшой магнит вокруг токоведущего провода и убедившись, что сила, прилагаемая ток на магните был круговым. Как он объяснил много лет спустя, провод был окружен бесконечной серией круговых концентрических силовых линий , которые он назвал магнитным полем тока. Он взял за отправную точку работы Эрстеда и Ампера о магнитных свойствах электрических токов и в 1831 году получил электрический ток от изменяющегося магнитного поля, явление, известное как электромагнитная индукция . Он обнаружил, что когда через катушку пропускают электрический ток, в соседней катушке генерируется еще один очень короткий ток. Это открытие ознаменовало решающую веху в прогрессе не только науки, но и общества , и сегодня оно используется для производства электроэнергии в больших масштабах на электростанциях.Это явление открывает кое-что новое об электрических и магнитных полях. В отличие от электростатических полей, создаваемых электрическими зарядами в состоянии покоя, циркуляция которых по замкнутому пути равна нулю (консервативное поле), циркуляция электрических полей, создаваемых магнитными полями, происходит по замкнутому пути, отличному от нуля. Эта циркуляция, которая соответствует индуцированной электродвижущей силе, равна скорости изменения магнитного потока, проходящего через поверхность, граница которой представляет собой проволочную петлю ( закон индукции Фарадея ).Фарадей изобрел первый электродвигатель, первый электрический трансформатор, первый электрический генератор и первую динамо-машину, поэтому Фарадея можно без всяких сомнений назвать отцом электротехники .

Фарадей отказался от теории жидкости для объяснения электричества и магнетизма и ввел концепции поля , и силовых линий , отойдя от механистического объяснения природных явлений, таких как действия Ньютона на расстоянии. Введение Фарадеем концепции поля в физику, возможно, является его наиболее важным вкладом и было описано Эйнштейном как великое изменение в физике , потому что оно обеспечило электричество, магнетизм и оптику общей рамкой физических теорий.Однако силовые линии Фарадея не были приняты до тех пор, пока несколько лет спустя не появился Джеймс Клерк Максвелл.

Как отмечалось в начале этой статьи, другим и, возможно, менее известным эффектом, обнаруженным Фарадеем, было влияние магнитного поля на поляризованный свет, явление, известное как эффект Фарадея или магнитооптический эффект . Пытливый ум Фарадея не удовлетворился простым открытием взаимосвязи между электричеством и магнетизмом. Он также хотел определить, влияют ли магнитные поля на оптические явления. Он верил в единство всех сил природы, в частности света, электричества и магнетизма. 13 сентября 1845 г. г. он обнаружил, что плоскость поляризации линейно поляризованного света поворачивается, когда этот свет проходит через материал, к которому приложено сильное магнитное поле в направлении распространения света. Фарадей написал в абзаце № 7504 своего Dairy :

.

«Сегодня работал с магнитными силовыми линиями, пропуская их через разные тела (прозрачные в разных направлениях) и в то же время пропуская через них поляризованный луч света (…) на поляризованном луче производился эффект, и, таким образом, магнитный доказано, что сила и свет связаны друг с другом ».

Это, безусловно, было первым явным указанием на то, что магнитная сила и свет связаны друг с другом, а также показало, что свет связан с электричеством и магнетизмом. В связи с этим явлением Фарадей также писал в том же абзаце:

.

«Этот факт, скорее всего, окажется чрезвычайно плодотворным и очень ценным при исследовании обоих условий естественной силы».

Он не ошибся. Этот эффект является одним из краеугольных камней электромагнитной теории света.

Вращение поляризации из-за эффекта Фарадея / Источники: адаптировано из Википедии

В выступлении королевского института в пятницу вечером, проведенном в апреле 1846 года года, Фарадей предположил, что свет может быть некоторой формой возмущения, распространяющегося вдоль силовых линий . На самом деле именно в эту пятницу Чарльз Уитстон должен был выступить с докладом о своем хроноскопе. Однако в последнюю минуту Уитстона охватил приступ страха перед сценой, и Фарадей выступил с речью Уитстона.Поскольку он закончил раньше времени, он заполнил оставшиеся минуты, раскрывая свои мысли о природе света . Выступление Фарадея было опубликовано в том же году в журнале Philosophical Magazine под заголовком Мысли о лучевых вибрациях . Фарадей даже осмелился подвергнуть сомнению существование светоносного эфира — научная ересь в то время — который должен был быть средой для распространения света, как так элегантно Френель описал в своей волновой теории света.Он предположил, что свет может быть не результатом вибраций эфира, а вибрацией физических силовых линий. Фарадей попытался исключить эфир, но он сохранил вибрации. Почти извиняющимся тоном Фарадей заканчивает свой доклад, в котором говорится:

.

«Я думаю, что вполне вероятно, что я сделал много ошибок на предыдущих страницах, потому что даже для меня мои идеи по этому поводу кажутся только тенью спекуляции ».

Однако эта идея Фарадея была воспринята со значительным скептицизмом и отвергалась всеми до тех пор, пока в 1865 году не была опубликована статья Максвелла под названием Динамическая теория электромагнитного поля .В этой статье Максвелл не только описывает свою основополагающую электромагнитную теорию света — одну из вех, отмеченных в Международный год света 2015 — но также приписывает идеи, которые в конечном итоге легли в основу его теории, мыслям Фарадея о лучевых вибрациях . На странице 466 своей статьи со скромностью, всегда свойственной Максвеллу, он ссылается на статью Фарадея 1846 года следующим образом:

«Концепция распространения поперечных магнитных возмущений за исключением нормальных четко изложена профессором Фарадеем в его« Мыслях о лучевых колебаниях ».Электромагнитная теория света, предложенная им [Фарадеем], по сути та же, что и та, которую я начал развивать в этой статье, за исключением того, что в 1846 году не было данных для расчета скорости распространения ».

И на странице 461 своей статьи 1865 года Максвелл также упоминает о магнитооптическом эффекте, заявляя:

«Фарадей обнаружил, что когда плоско поляризованный луч пересекает прозрачную диамагнитную среду в направлении линий магнитной силы, создаваемых соседними магнитами или токами, плоскость поляризации вращается».

Всего Майкл Фарадей цитируется шесть раз и трижды упоминается в статье Максвелла 1865 года. Однако это неудивительно, учитывая, что большая часть работ Максвелла основана на работах Фарадея, и Максвелл математически смоделировал большинство открытий Фарадея по электромагнетизму в теорию, которую мы знаем сегодня.

Электромагнитные волны, о существовании которых Фарадей размышлял в 1846 году в своих мыслях о лучевых колебаниях , , и которые были математически предсказаны Максвеллом в 1865 году, наконец, были получены в лаборатории Герца в 1888 году.Остальное уже история. Ясно, что Максвелл открыл дверь в физику двадцатого века, но не менее ясно, что Фарадей дал Максвеллу некоторые из ключей, которые он использовал.

В 1676 году Ньютон послал своему сопернику Гуку письмо, в котором написал: «Если я и видел дальше, то это было то, что он стоял на плечах гигантов» (*). Двести пятьдесят лет спустя, во время одного из визитов Эйнштейна в Кембридж, Великобритания, кто-то заметил: «Вы сделали великие дела, но стоите на плечах Ньютона». Эйнштейн ответил: «Нет, я стою на плечах Максвелла».Если бы кто-то сказал то же самое Максвеллу, он, вероятно, сказал бы, что он стоял на плечах Фарадея .

(*) Хотя это предложение интерпретируется некоторыми авторами как саркастическое замечание, направленное на горбатую внешность Гука, в настоящее время эта фраза обычно используется в положительном ключе. Комментарий Ньютона — это заявление о том, что наука представляет собой серию постепенных достижений, в основе которых лежат уже достигнутые ранее (см., Например, книгу Стивена Хокинга под названием « На плечах гигантов »).

Аугусто Белендес

Профессор прикладной физики Университета Аликанте (Испания) и член Королевского физического общества Испании

Библиография

  • A. Díaz-Hellín, Faraday: El gran cambio en la Física (Nívola. Madrid, 2001).
  • Ордоньес, В. Наварро и Х. М. Санчес Рон, Historia de la ciencia (Espasa Calpe. Madrid, 2013).
  • Форбс и Б. Махон, Фарадей, Максвелл и электромагнитное поле: как два человека революционизировали физику (Prometheus Books.Нью-Йорк, 2014).
  • Зайонц, Улавливая свет: переплетенная история света и разума (Oxford University Press, Нью-Йорк, 1995)
  • Хокинг, На плечах гигантов: великие труды по физике и астрономии (Running Press. Philadelphia, 2002)
  • Мансурипур, Классическая оптика и ее приложения (Издательство Кембриджского университета, Кембридж, 2002)

Электромагнетизм — закон Фарадея или сила Лоренца

Студент моего класса физики дал в группе неправильный ответ на вопрос.

Ситуация была такой: самолет имеет провод, протянутый между концами крыльев, и он летит через магнитное поле перпендикулярно, ускоряясь.

Возник вопрос: каким будет наведенный ток?

Его ответ был 0, и он объяснил, что, поскольку самолет летит через однородное поле, не будет никаких изменений в плотности магнитного потока, поэтому никаких изменений в потокосцеплении и, в конечном счете, не будет индуцированного тока.

Я заявил, пояснив здесь сегодня вопрос, что вместо того, чтобы обращаться к законам Фарадея, он должен подумать о правиле левой руки Флеминга.

Мой учитель сказал мне, что я ошибался, и сказал, что, когда самолет ускоряется, он с большей скоростью сокращает силовые линии, поэтому согласно закону Фарадея будет индуцировано напряжение, но он также говорит, что это напряжение будет увеличиваться. .. Даже если бы это были законы Фарадея, поскольку это ускорение было постоянным, разве напряжение не было бы постоянным?

Мой следующий длинный ответ:

Закон Фарадея диктует пропорциональность между наведенной ЭДС и скоростью изменения потоковой связи.Правило проволоки, прорезающей линии поля, напротив, является результатом правила левой руки Флеминга, где магнитная сила действует на электроны внутри проволоки, которая разрезает поле, в направлении проволоки; это приводит к разности потенциалов между обеими сторонами провода или индуцированному току для замкнутой цепи. Кроме того, магнитная связь и, следовательно, закон Фарадея относятся к соленоидам, не так ли? В конечном итоге индуцированное напряжение действительно будет увеличиваться из-за увеличения скорости, с которой провод разрезает поле, но это не связано с законом Фарадея, который связывает изменение магнитной связи в двумерном проводнике. студенты понятное объяснение его ответа о наведенной ЭДС, равной 0, пришло от него, когда он заметил, что потокосцепление имеет постоянную величину, поэтому будет иметь нулевую скорость изменения, независимо от ускорения. Используя правило левой руки и тот факт, что магнитная сила F является суммой BQv, можно увидеть, что электроны будут перемещаться под действием большей силы, что приведет к большей разнице потенциалов, ЭДС или току, так как провод ускоряется.

Я также понял, что при постоянном ускорении напряжение было бы постоянным, если бы это было связано с законом Фарадея, и добавлю это к тому, что я сказал.

Я прошу, потому что хочу помочь этому человеку, не скармливая ему ложную информацию, неправомерно подрывая моего учителя и, в конечном итоге, ставя себя в неловкое положение.

alexxlab

Добавить комментарий

Ваш адрес email не будет опубликован. Обязательные поля помечены *